You are on page 1of 94

1 .

A 75-year-old woman who has had type-2 diabetes mellitus for the last 15
years is admitted for cataract surgery. She is taking metformin 500 mg bid
plain insulin 10 ! at night ramipril 2.5 mg od and bendroflumethia"ide
#bendroflua"ide$ 2.5 mg od. She drinks a glass of whisky to help her sleep at
night. %n&estigations showed' () *.0 mmol+l, urea 10 mmol+l, creatinine 1-0
.mol+l, /a
0
1*0 mmol+l, 1
0
*.7 mmol+l .
2i&er function tests #234s$ were normal and arterial blood gas measurements
showed a p5 7.* p#672$ 5.1 k8a and 567* 17 mmol+l .
9hich one of the following is the most detrimental in these circumstances and should
be stopped ?
Ramipril
Metformin
Your answer
Insulin
Alcohol
Bendroflumethiazide (bendrofluazide (
Metformin is a biguanide used as a blood glucose lowering agent. The major toicit! of
metformin is lactic acidosis. It should not be used in patients with renal insufficienc! (creatinine
" #$$ %mol&l in males and " #'( %mol&l in females)* an! form of acidosis* congesti+e heart
failure* li+er disease or se+ere h!poia. As it is metabolised in the li+er* it is contraindicated in
patients with li+er disease or a high alcohol inta,e. Ramipril* an angiotensin-con+erting enz!me
inhibitor (A./I)* is indicated for the treatment of diabetic nephropath! with albuminuria. It ma!
potentiate the h!pogl!caemic effect of insulin and oral antidiabetic agents* especiall! during the
first few wee,s of use. It should be used under specialist super+ision if the creatinine
concentration is " #01 %mol&l* and renal function and potassium le+els monitored. This patient
has long-standing diabetes with nephropath!. 2!pogl!caemia is common with nephropath! and
is thought to be due to the decreased metabolism of insulin. 3ecreasing renal function can
cause metabolic acidosis* either of which is a contraindication for treatment with metformin.
Bendroflumethiazide (bendrofluazide) can be safel! continued .
2 . A *5-year-old man with type-1 diabetes mellitus on insulin presents in the
A:; with fe&er cough &omiting and abdominal pain. ;<amination re&eals a
dry mucosa decreased skin turgor and a temperature of *7.= >6. 6hest
e<amination re&eals bronchial breathing in the right lower lobe and a chest ?-
ray shows it to be due to a right lower "one consolidation. 7ther in&estigations
show a blood glucose concentration of *0 mmol+l /a
0
1*0 mmol+l 1
0
5.7
mmol+l urea =.0 mmol+l creatinine 120 @mol+l p5 7.15 567* 12 mmol+l
p#672$ -.A k8a and chloride 10A mmol+l. !rinary ketones are positi&e #000$.
4he patient is admitted to the hospital and treated .
9hich of the following should not be used while treating him ?
4luids* i+
Insulin
5otassium
Your answer
Bicarbonate
.orrect answer
Antibiotics
This patient has diabetic ,etoacidosis (36A)* which is more common in t!pe-I than t!pe-'
diabetes mellitus. Inade7uate insulin causes lipol!sis* leading to ,etosis and acidosis. The
common precipitants include inade7uate insulin administration* infections (pneumonia* urinar!
tract infection (8TI))* m!ocardial infarct (MI)* stro,e* peripheral gangrene* etc. Treatment is
directed at the underl!ing cause and correction of the ,etoacidotic state. 2ere* the underl!ing
cause* pneumonia* is treated with antibiotics. The most important aspect of treatment is to gi+e
i+ fluids* as normal saline* along with regular insulin. 9hen the fluid balance is restored* the
initial high potassium concentration ma! fall and therefore has to be monitored and
supplemented as re7uired .
The role of 2.:$ in 36A is contro+ersial. The acidosis usuall! corrects itself once the fluid and
electrol!te balance is restored. ;ome ph!sicians administer 2.:$ if se+ere acidosis (p2 < =) is
present. 2owe+er* there is a +iew that rapid correction of acidosis ma! impair cardiac function .
3 . A *0-year-old woman is e&aluated in the endocrinology clinic for increased
urine output. She weighs A0 kg and has a 2--hour urine output of *500 ml.
5er basal urine osmolality is 210 m7sm+kg. She undergoes a fluid depri&ation
test and her urine osmolality after fluid depri&ation #loss of weight * kg$ is
*50 m7sm+kg. A subseBuent inCection of subcutaneous DDAE8 #desmopressin
acetate$ did not result in a further significant rise of urine osmolality after 2
hours #*55 m7sm+kg .(
9hich of the following is the likely diagnosis ?
>ormal
5rimar! pol!dipsia
.orrect answer
:smotic diuresis
5ituitar! diabetes insipidus
>ephrogenic diabetes insipidus
Your answer
3iabetes insipidus (3I) is suspected when the urine output is " 01 ml&,g per da! ($111 ml for a
?1-,g female). If the basal urine osmolalit! is " $11 m:sm&,g* it suggests an osmotic diuresis.
If fluid depri+ation causes a urine osmolalit! " $11 m:sm&,g* it suggests ps!chogenic
(primar!) pol!dipsia. If not* it is either pituitar! or nephrogenic 3I. These are differentiated b!
the administration of subcutaneous 33A@5 which causes an increased urine osmolalit! " $11
m:sm&,g in pituitar! 3I .
4 . 9hich of the following is a feature of );/-1 ?
Marfanoid features
5haeochromoc!toma
Medullar! carcinoma of the th!roid
Your answer
5arath!roid h!perplasia
.orrect answer
Mucosal neuromas
M/>-# (multiple endocrine neoplasia t!pe-#A 9ernerBs s!ndrome) in+ol+es the three 5BsC
parath!roid* pituitar!* pancreas .
5arath!roid 2!perparath!roidism is the most common manifestation in M/>-#
5ituitar! 5rolactinomas are most common in the pituitar! gland
5ancreas
;econd most common in+ol+ement in M/>-#. 5ancreatic tumours are associated
with :
) a) 5ancreatic pol!peptide (=0DE0F (
) b) gastrin (GollingerD/llison s!ndrome) D recurrent peptic ulcers
) c) insulinoma D h!pogl!caemia
) d) glucagonoma D h!pergl!caemia and s,in rash (necrol!tic migrator! er!thema (
) e) @I5oma (+asoacti+e intestinal pol!peptide-secreting tumour) D @ernerDMorrison
s!ndrome or water! diarrhoea h!po,alaemia achlorh!dria (932A s!ndrome (
M/>-' usuall! in+ol+es the th!roid and parath!roid glands and phaeochromoc!toma. The mode
of inheritance in M/>-# is autosomal-dominant. M/>-'A is characterised b! medullar! th!roid
carcinoma* parath!roid h!perplasia&adenoma and phaeochromoc!toma. In addition* M/>-'B
has mucosal and gastrointestinal (HI) neuromas and marfanoid features .
5 . A *5-year-old 5%E-positi&e man e&aluated for weight loss and weakness has
been found to ha&e disseminated tuberculosis. 7n e<amination he is
hypotensi&e and has hyperpigmentation of the mucosa elbows and skin
creases. 3urther in&estigations confirm a diagnosis of AddisonFs disease .
9hich of the following is likely in this condition ?
Increased serum >a
Increased serum .l
Increased serum 2.:$
Increased serum 6
Your answer
3ecreased serum .a
In adrenal destruction* serum >a* .l and 2.:$ le+els are reduced* and serum potassium is
ele+ated. The h!ponatraemia is due to both loss of sodium in the urine (due to aldosterone
deficienc!) and to mo+ement into the intracellular compartment. 2!per,alaemia is due to a
combination of aldosterone deficienc!* impaired glomerular filtration (due to h!potension) and
acidosis. Mild to moderate h!percalcaemia occurs in #1D'1F patients* the cause of which is
uncertain .
6 . A A0-year-old man is referred to the endocrine clinic with a complaint that his
shoe si"e has gone up from si"e G to si"e 11 and his wedding ring no longer
fits him. 5e is sweating a lot and his wife complains he is snoring more at
night .
9hich of the tests below is most useful for confirming a diagnosis of acromegaly ?
:HTT with H2 measurements
Your answer
;erum IH4-# le+el
;,ull I-ra!
Random H2 le+el
MRI of the pituitar! fossa
A =0-g oral glucose tolerance test (:HTT) is the Jgold standardB for a diagnosis of acromegal!.
In normal people the growth hormone (H2) le+el suppresses during the test* but in patients
with acromegal! it is not suppressed. This is because insulin and H2 are antagonistic hormones.
Random le+els of H2 and insulin-li,e growth factor-# (IH4-#) ma! be raised* but as there is a
wide range of normal a single measurement is not sufficient to ma,e the diagnosis. A d!namic
test (the :HTT) is re7uired. Both a s,ull I-ra! and an MRI scan are li,el! to be abnormal in
patients with acromegal!. 5ituitar! tumours in acromegal! are usuall! macroadenomas. As
other secretor! and non-secretor! tumours of the pituitar! can also be macroadenomas these
tests would not confirm the patient had acromegal!* though the! would be used in the
diagnostic wor,-up .
7 . A A2-year-old woman presents to her H8 complaining of thirst and polyuria.
5er fasting glucose le&el is G mmol+l and =.2 mmol+l on two separate
occasions confirming she has diabetes mellitus .
9hich of the following additional findings would be least likely to be associated with a
diagnosis of secondary diabetes mellitus ?
A bitemporal +isual field defect
A large goitre
Your answer
2!pertension
Multiple striae and bruises
Maculopath!
.orrect answer
;econdar! causes of diabetes occur in other endocrine conditions where hormones with
antagonistic actions to insulin are present in ecess amounts. 5atients with acromegal!* which
shows ele+ated growth hormone (H2) le+els secondar! to a pituitar! adenoma* can also ha+e a
goitre* +isual field defects and h!pertension and diabetes. Those with .ushingBs s!ndrome* from
glucocorticoid ecess* can ha+e +isual field defects* h!pertension* striae and bruising.
Maculopath! is present in diabetes of long duration and is related to long-term poor gl!caemic
control. It could be present in patients with secondar! diabetes if the! ha+e gone undiagnosed
for some time. 2owe+er* maculopath! is not related to an! of the hormone ecesses seen in
these conditions .
8 . A 2A-year-old woman attends her H8 complaining of feeling tired all the time
for the last few months. She has had no period for A months and has been
feeling di""y first thing in the morning .
9hich of the following clinical signs would the H8 be most likely to find if the
diagnosis was thought to be glucocorticoid deficiency ?
Buccal pigmentation
3iminished bod! hair
5allor
5ostural h!potension
Your answer
:ptic atroph!
2!poadrenalism has man! causes including AddisonKs disease* granulomatous disorders* TB*
tumour (particularl! lung) or related to infection (meningococcal septicaemia). ;econdar!
h!poadrenalism is caused b! h!popituitarism or h!pothalamic disorders. 5allor is present in
h!popituitarism due to normochromic* normoc!tic anaemia and a lac, of melanoc!te-stimulating
hormone (M;2A clea+ed from the precursor of A.T2). Buccal pigmentation is also associated
with glucocorticoid deficienc!* most commonl! from AddisonBs disease. A.T2 is ele+ated in
AddisonBs disease and results in higher le+els of M;2* which causes the s,in and buccal
pigmentation found in this condition. Lac, of bod! hair and amenorrhoea are features of
h!pogonadism in h!popituitarism. 5ostural h!potension is related to glucocorticoid deficienc!
regardless of cause .
9 . A *0-year-old woman presents to her H8 with a history of amenorrhoea and
galactorrhoea. She is keen to become pregnant and has been trying for G
months to concei&e without success. She is of normal weight and has no other
constitutional symptoms .
9hich of the following is most likely to be the reason for her symptoms ?
3rug treatment she is on for asthma
2!poth!roidism
2!perth!roidism
5re+iousl! undiagnosed hepatic impairment
5ituitar! microadenoma
Your answer
2er s!mptoms are consistent with h!perprolactinaemia. Raised prolactin has man! causes
including pituitar! tumours (micro- and macroadenomas)* drug treatment (some
antidepressants li,e tric!clic antidepressants* anti-emetics and cimetidine)* pregnanc!* stress*
h!poth!roidism (because th!rotrophin-releasing hormone (TR2) stimulates T;2 and prolactin in
primar! h!poth!roidism) and renal failure (prolactin accumulates in renal failure but not in
hepatic impairment). 2!perth!roidism is not associated with h!perprolactinaemia and
galactorrhoea* although it can cause amenorrhoea and fertilit! problems. 9hile
h!perprolactinaemia can be present in h!poth!roidism it is usuall! associated with menorrhagia
rather than amenorrhoea .
10 . An 1=-year-old young man presents to his H8 with thirst and polyuria. Some A
months pre&iously he had a significant head inCury as the result of a road
traffic accident. 5e is referred to the local endocrine clinic .
9hich of the following results would be the most useful in confirming a diagnosis of
diabetes insipidus after a water depri&ation test ?
5lasma sodium of #'? mmol&l
5lasma sodium of #01 mmol&l
5lasma osmolalit! of $$0 m:sm&,g and urine osmolalit! of =11
m:sm&,g
5lasma osmolalit! of 'E1 m:sm&,g and urine osmolalit! of =11
m:sm&,g
Your answer
5lasma osmolalit! of $$0 m:sm&,g and urine osmolalit! of '11
m:sm&,g
.orrect answer
3iabetes insipidus (3I) can be due to cranial 3I with complete or partial deficienc! of
antidiuretic hormone (A32* +asopressin)* or renal 3I due to renal tubular defects affecting the
action of A32 on water reabsorption from the renal tubules. The plasma sodium can be normal
or ele+ated* depending on whether the patient can continue to drin, freel! and so pre+ent
deh!dration. A low plasma sodium concentration is associated with ;IA32 (s!ndrome of
inappropriate A32). The diagnostic test to confirm 3I is a water depri+ation test. The inabilit! to
concentrate the urine during the test results in the plasma osmolalit! rises and the urine
osmolalit! remaining dilute.>ormal plasma osmolalit! is 'E0D$10 m:sm&,g. The higher the
urine osmolalit! the more concentrated it is .
11 . A 5A-year-old lifelong smoker presents to his H8 with a history of cough
breathlessness and weight loss. A chest ?-ray is abnormal with a mass at the
right hilum. 9hich of the following results is most likely to suggest the tumour
is a small-cell lung tumour ?
;erum calcium of $.$ mmol&l
;erum sodium of #'$ mmol&l
Your answer
;erum potassium of 0.0 mmol&l
5lasma osmolalit! of $$0 m:sm&,g
8rine osmolalit! of #(0 m:sm&,g
;mall-cell lung tumours can secrete a number of hormones including A32 (+asopressin) and
A.T2. /cess A32 causes ;IA32 (s!ndrome of inappropriate A32) with h!ponatraemia and low
plasma osmolalit! (normal 'E0D$10 m:sm&,g) and concentrated urine (" 011 m:sm&,g).
/cess A.T2 would cause a h!po,alaemic al,alosis and not h!per,alaemia. 2!percalcaemia
occurs in s7uamous-cell lung tumours because of ectopic parath!roid hormone (5T2) secretion .
12 . A 5--year old woman is seen for the first time in the diabetes clinic. She is
obese plethoric and has marked bruising on her limbs and fresh striae o&er
her abdomen. She has a dorsal kyphosis following a &ertebral collapse earlier
in the year .
9hich of the following results will help to pinpoint the diagnosis if you suspect
6ushingFs syndrome secondary to adrenal adenoma ?
>ormal 1M11-h serum cortisol le+el
;erum potassium of '.' mmol&l
0900 - h serum cortisol of '11 nmol&l after o+ernight deamethasone test
Raised urine cortisol&creatine ratio
8ndetectable serum A.T2 le+el
Your answer
.ushingBs s!ndrome can be A.T2-dependent (pituitar!-dri+en .ushingBs* ectopic A.T2
secretion) or A.T2-independent (adrenal adenoma* adrenal carcinoma and adrenal nodular
h!perplasia). The serum cortisol le+el can be normal in all these conditions as it has a wide
range of normal. There is a loss of diurnal +ariation and night-time cortisol is ele+ated. ;erum
potassium is most li,el! to be low in cases of ectopic A.T2 or adrenal carcinoma. The urine
cortisol&creatinine ratio is ele+ated in all causes of .ushingBs and doesnBt help to differentiate
the cause. A normal A.T2 le+el can be found in pituitar!-dri+en .ushingBs and sometime also in
ectopic A.T2 as there is o+erlap between the normal and ele+ated ranges. In adrenal causes of
.ushingBs s!ndrome the A.T2 is suppressed or undetectable. All causes of .ushingBs s!ndrome
would gi+e an unsuppressed 1M11-h cortisol le+el after an o+ernight deamethasone
suppression test .
13 . A A7-year-old man is admitted with a A-week history of pro<imal muscle
weakness. 5e has been ha&ing difficulty climbing stairs and getting up from a
sitting position in a chair. 5e is a lifelong smoker and alcohol intake is *0 units
per week .
9hich of the following results is most likely to point to a diagnosis ?
Abnormal li+er function tests and macroc!tosis
.orrect
answer
A suppressed 1M11-h serum cortisol le+el after an o+ernight
deamethasone suppression test
Your answer
2!percalcaemia
Low serum +itamin B#' le+el
/le+ated T;2 and normal free-th!roine le+els
The s!mptoms he is describing are in ,eeping with a proimal m!opath!. .auses include alcohol
ecess (abnormal L4Ts and raised M.@)* .ushingBs s!ndrome (failure to suppress 1M11-h serum
cortisol le+el after an o+ernight deamethasone suppression test)* osteomalacia (low calcium
and raised al,aline phosphatase le+els) and th!rotoicosis (suppressed T;2 and raised free T(
le+els) or h!poth!drodism (ele+ated T;2 and low 4T(). The T4Ts abo+e are in ,eeping with
compensated h!poth!drodism which isnKt associated with proimal m!opath!. Low +itamin B#'
(eg in pernicious anaemia) causes a number of neurological s!mptoms including peripheral
neuropath! and subacute combined degeneration of the spinal cord* but not proimal m!opath! .
14 . A 51-year-old man presents to A:; with altered consciousness his blood
pressure is =0+50 mm5g his skin is pigmented and he has a past history of
5ashimotoFs thyroiditis. 5is family says he has been tired for se&eral months
and has been losing weight and complaining of abdominal pain .
9hich of the following results is most likely to be found on in&estigation ?
Laborator! glucose le+el of #'.$
mmol&l
;erum sodium le+el of ##? mmol&l
Your answer
5ea,ed T-wa+es on /.H
;erum potassium le+el of '.( mmol&l
;erum urea le+el of $.' mmol&l
The clinical picture is suggesti+e of AddisonBs disease with h!potension* pigmentation and a
prodromal period with s!mptoms in ,eeping with glucocorticoid deficienc! in a man with a
histor! of another autoimmune endocrine disorder. :ther disorders that can be associated are
diabetes and pernicious anaemia. 2!pogl!caemia* h!ponatraemia* h!per,alaemia and an
ele+ated urea are all present during a h!poadrenal crisis due to steroid deficienc! and
subse7uent salt and water loss. Although the potassium is ele+ated it is unusual to find pea,ed
T-wa+es on the /.H in patients with h!poadrenalismC the complees tend to be small* low-
+olume ones .
15 . A *1-year-old man is referred to the local hypertension clinic because of
recently disco&ered hypertension that is labile and difficult to control .
9hich of the following features is most likely to suggest a genetic+familial syndrome
is the cause for his hypertension ?
;erum potassium le+el of $.M mmol&l
Your answer
Random blood glucose le+el of M mmol&l
A serum calcium le+el of #.0 mmol&l
A famil! histor! of uneplained death in childbirth
.orrect answer
A famil! histor! of papillar! th!roid neoplasia
Adrenal causes of h!pertension are .onnBs s!ndrome (primar! h!peraldosteronism)* .ushingBs
s!ndrome (from adrenal adenoma* carcinoma or adrenal nodular h!perplasia) and
phaeochromoc!toma. 2!po,alaemia is a feature of .onnBs and .ushingBs s!ndromes.
2!pergl!caemia can be present in .ushingBs s!ndrome or phaeochromoc!toma. 8neplained
death in childbirth and a famil! histor! of th!roid neoplasia raise the possibilit! of multiple
endocrine neoplasia t!pe-' (phaeochromoc!toma* medullar! th!roid tumours and
h!perparath!roidism) as the cause of his h!pertension. A raised serum calcium le+el would also
point towards M/>' (multiple endocrine neoplasia D t!pe ') but h!pocalcaemia is not associated
with adrenal causes of h!pertension .
16 . A 25-year-old woman presents to a reproducti&e endocrinology clinic with a
history of being unable to concei&e after 2 years of using no contraception. %t
is thought she may ha&e polycystic o&arian syndrome .
9hich of the following is most likely to be associated with this condition ?
A 'E da! menstrual c!cle
/le+ated L2&4;2 ratio
Your answer
>ormal free-androgen inde
Low le+els of circulating insulin
>ormal BMI (bod! mass inde (
5ol!c!stic o+arian s!ndrome is one of the commonest causes of ano+ulator! infertilit!. 5atients
can ha+e a normal menstrual c!cle but are more li,el! to ha+e oligomenorrhoea. It is associated
with a number of biochemical abnormalities* including raised L2 le+els* normal or ele+ated
testosterone but with a low ;2BH (se-hormone-binding globulin) resulting in a high free-
androgen inde. Androstenedione le+els can either be normal or raised. The underl!ing
biochemical defect in patients with 5.:; is recognised to be insulin resistance. This causes high
circulating insulin le+els due to peripheral insulin resistanceC therefore h!perinsulinaemia* and
not low insulin le+els* is characteristic of the condition. The insulin resistance has been shown in
both lean and obese patients with the condition .
17 . A *7-year-old woman presents to the endocrine clinic with a history of
hirsutism acne and oligomenorrhoea. She is ha&ing difficulty losing weight
and has searched the %nternet and thinks she may ha&e polycystic o&arian
syndrome. She wants to discuss the implications of this .
9hich of the following is the most important issue to discuss with her at this stage of
her life ?
/ercise regimens
3oes she want to ha+e children
.orrect answer
2er blood glucose le+el
Treatment for her hirsutism
Your answer
9eight-reduction diets
All the abo+e are rele+ant and each should be discussed. The hirsutism and acne can be +er!
difficult to deal with and can cause distress from a cosmetic point of +iew. There are a +ariet! of
treatment options but 3ianette (c!proterone acetate) is probabl! the most effecti+e* along with
cosmetic treatments li,e waing* sha+ing* pluc,ing or electrol!sis. 2er future ris, of t!pe-'
diabetes and associated cardio+ascular ris, is +er! important and she should be ad+ised about
the need for lifest!le treatments and the need to lose weight and eercise regularl! to reduce
the chance of this happening. It is now recommended that all patients with 5.:; ha+e their
fasting blood glucose le+el measured annuall! to pic, up diabetes at an earlier stage. 2owe+er*
the most important issue in a woman of her age is fertilit!* as women with 5.:; fre7uentl!
re7uire assistance with conception. 2er age is against her if she is going to ha+e problems with
fertilit! and re7uires help to concei+e. The commonest treatment is to induce o+ulation with
clomifene. ;he needs to be ad+ised that pregnanc! also carries an increased ris, of gestational
diabetes .
18 . A 2G-year-old woman presents to her H8 with a history of weight loss heat
intolerance poor concentration and palpitations .
9hich of the following is most likely to be associated with a diagnosis of
thyroiditis associated with &iral infection ?
Bilateral eophthalmos
3iffuse* smooth goitre
Reduced upta,e on th!roid isotope scan
Your answer
>egati+e th!roid peroidase antibodies
5retibial m!oedema
Hra+esB disease is an autoimmune th!roid disorder and is more common in women than men. It
is associated with positi+e T5: (th!roid peroidase antibodies) and T;2 (th!roid stimulating
hormone) receptor antibodies. The goitre in Hra+esB disease is normall! diffusel! enlarged
rather than nodular. 8pta,e of radio-isotope is increased in Hra+esB disease. In other conditions
causing th!rotoicosis* li,e postpartum th!roiditis and +iral th!roiditis* the upta,e is reduced.
Hra+esB disease is associated with a number of other features not found in other causes of
th!rotoicosis* eg eophthalmos* pretibial m!oedema and th!roid acropach! (a condition that
affects the nails and loo,s li,e finger clubbing). The eophthalmos can be unilateral or bilateral .
19 . A *--year-old woman is referred to the endocrine clinic with a history of
thyroto<icosis. At her first appointment she is found to ha&e a smooth goitre
lid lag and bilateral e<ophthalmos with puffy eyelids and conCuncti&al
inCection. She wants to discuss treatment of her thyroid problem as she is
keen to become pregnant .
9hat is the most likely treatment you would ad&ise ?
12 18 months of carbimazole alone
12 18 months of prop!lthiouracil alone
.orrect answer
A combination of antith!roid drugs and th!roine
Radioacti+e iodine (iodine-#$# (
Th!roidectom!
Your answer
All the abo+e treatments are recognised treatments for th!rotoicosis. Antith!roid drugs
(carbimazole and prop!lthiouracil) can be used alone and the dose titrated down or up to ,eep
the patient euth!roid. Treatment is usuall! for #'D#E months. 2owe+er carbimazole has been
associated with aplasia cutis (a scalp defect) in babies born to mothers treated during
pregnanc!. The Jbloc, and replaceB regimen of th!roine and anti-th!roid drugs is not used in
pregnanc! as th!roine does not cross the placenta. ;urger! is an option for people with a large
goitre* or for people who fail on other treatments but would not be first choice in a !oung
otherwise health! woman. Radio-iodine is a +er! effecti+e treatment for th!rotoicosis but is
contraindicated in women of childbearing age who are contemplating pregnanc! within ?
months .
20 . A *0-year-old man and his wife present to a reproducti&e endocrinology clinic
because of infertility. 4he man is tall with bilateral gynaecomastia.
;<amination of the testes re&eals bilateral small firm testes .
9hich of the following in&estigations is most helpful in diagnosing a patient with
1linefelterFs syndrome ?
.T scan of the pituitar! gland
.hromosomal anal!sis
Your answer
Measurement of serum gonadotrophins
Measurement of serum testosterone
;emen anal!sis
6linefelterBs s!ndrome is a genetic disorder with an etra I chromosome* gi+ing a genot!pe
IIY. It is usuall! diagnosed in late pubertal or earl! adult life because of dela!ed seual
de+elopment or infertilit!. It is associated with h!pogonadism gi+ing raised gonadotrophin le+els
and low testosterone le+els. ;emen anal!sis would show azoospermia. H!naecomastia is often
present. .T brain scan will be normal. The other tests are appropriate for in+estigation of male
infertilit!. There are other cause of infertilit! that would gi+e high gonadotrophin and low
testosterone le+elsA an! primar! testicular disorder would gi+e this pattern* eg cr!ptorchidism*
mumps orchitis* haemochromatosis* m!otonic d!stroph! and alcohol abuse. The onl! test that is
specific for 6linefelterBs s!ndrome is a genetic test for the IIY genot!pe .
21 . A 2A-year-old man is referred for gastroscopy because of se&eral months of
dyspepsia. 5e has routine bloods checked and is found to ha&e a serum
calcium le&el of *.2 mmol+l with a &enous bicarbonate le&el of ** mmol+l.
Ienal and li&er function are both normal. 6hest ?-ray is normal .
9hat is the most likely cause of his hypercalcaemia ?
M!eloma
Metastatic malignanc!
Mil,Dal,ali s!ndrome
Your answer
5rimar! h!perparath!roidism
;arcoidosis
All the abo+e diagnoses are causes of h!percalcaemia. ;!mptoms of h!percalcaemia are
commonl! thirst* pol!uria* d!spepsia* malaise* bone pain and constipation. M!eloma and
metastatic malignanc! cause h!percalcaemia either b! direct l!tic lesions of bone or b! the
production of 5T2r5 (parath!roid hormone-related protein). M!eloma would be etremel! rare
in this age group. There is nothing in the histor! to suggest bone pain or a diagnosis of
metastatic malignanc!. 5rimar! h!perparath!roidism is most commonl! found b! chance in
elderl! women. It can be part of the multiple endocrine neoplasia s!ndromes and therefore can
be seen in !ounger people. The normal chest I-ra! suggests that sarcoidosis is not the
diagnosis in this case. The age of the patient* the d!speptic s!mptoms and the raised
bicarbonate le+el suggest the most li,el! diagnosis is the mil,Dal,ali s!ndrome caused b! the
ingestion of antacids .
22 . A 17-year-old young woman is referred to the endocrine clinic with primary
amenorrhoea. She is of normal height and weight. She has moderate
hirsutism. A male cousin was seen in the clinic at the age of = years with
precocious puberty .
9hat is the most likely cause for her primary amenorrhoea ?
.ongenital adrenal h!perplasia
Your answer
2!perprolactinaemia
5ol!c!stic o+arian s!ndrome
TurnerBs s!ndrome
Testicular feminisation s!ndrome
TurnerBs s!ndrome (genot!pe I:) is associated with primar! amenorrhoea and short stature.
2!perprolactinaemia and 5.:; are more commonl! associated with secondar! amenorrhoea.
Testicular feminisation causes primar! amenorrhoea and is caused b! either a partial or
complete androgen-receptor defect. Affected patients are genot!picall! male (IY) but
phenot!picall! female. .ongenital adrenal h!perplasia is most commonl! caused b! '#-
h!dro!lase deficienc!. ;e+ere forms present in infanc! with salt-losing crises and females can
ha+e ambiguous genitalia. Milder forms cause precocious pubert! in bo!s and +irilism* hirsutism
and primar! amenorrhoea in girls. 3iagnosis is b! finding raised serum #=-h!dro!progesterone
le+els that show a h!perresponsi+eness to A.T2. Treatment is with glucocorticoids .
23 . An =1-year-old woman is referred to the thyroid clinic with increasing si"e of a
pre-e<isting goitre. She has had long-standing hypothyroidism and has been
on a dose of thyro<ine of 100 .g daily for many years .
9hich of the following primary thyroid cancers is she most likely to ha&e ?
Anaplastic th!roid cancer
4ollicular th!roid cancer
Medullar! th!roid cancer
5apillar! th!roid cancer
Your answer
Th!roid l!mphoma
.orrect answer
5rimar! th!roid l!mphoma is strongl! associated with l!mphoc!tic th!roiditis (2ashimotoBs
disease)* which is present in E1F of cases. It is predominantl! a disease of elderl! women. 8p
to $1F of patients ha+e a histor! of goitre and ma! be ta,ing th!roine. Anaplastic th!roid
cancer is commonl! found in elderl! patients and usuall! presents with a th!roid mass. It can
occur in a pre-eisting goitre. It is usuall! rapidl! progressi+e and has the worst prognosis of
the th!roid cancers. Medullar! th!roid cancer arises from within the J.B cells of the th!roid* cells
which produce calcitonin. It is commonl! associated with multiple endocrine neoplasia t!pe-'
(M/>') along with primar! h!perparath!roidism and phaeochromoc!toma. 4ollicular th!roid
cancer is t!picall! found in middle-aged to older indi+iduals and usuall! presents as a solitar!
as!mptomatic th!roid nodule. /+entuall! '1F of patients will de+elop distant metastases.
5apillar! th!roid cancer is most commonl! found in the $1D01 age group but can occur at an!
age. It most commonl! metastasises to the local l!mph nodes within the nec,. 5rognosis is
better than the other th!roid cancers .
24 . A 52-year-old woman sees her H8 complaining of a --kg weight gain dry hair
and skin she feels slow and always has the heating up high. She has a small
diffuse goitre. 5er 4S5 le&el is 20 m!+l and free thyro<ine 5 pmol+l. 4hyroid
pero<idase antibody is positi&e at high titres. She has two sisters who ha&e
had thyroid disease .
9hat is the most likely diagnosis ?
3eNuer+ainBs th!roiditis
4ollicular carcinoma
2ashimotoBs th!roiditis
Your answer
Hra+esB disease
>odular goitre
2ashimotoBs th!roiditis is an autoimmune th!roid disorder found more often in women than
men. It is associated with positi+e th!roid antibodies and there ma! be a goitre that tends to be
diffuse rather than multinodular. The gland is infiltrated with l!mphoc!tes and patients can
become h!poth!roid. 3eNuer+ainBs th!roiditis is not associated with positi+e antibodies and
tends to present with painful swelling in the nec,. Hra+esB disease is also an autoimmune
th!roid disorder but patients usuall! present with th!rotoicosis. 4ollicular carcinoma can
present with a th!roid swelling but not h!poth!roidism .
25 . A -5-year-old man presents to his H8 with headaches. 5is (8 is 1AA+G-
mm5g. Ioutine in&estigations re&eal sodium 1-2 mmol+l potassium 2.A
mmol+l chloride 101 mmol+l and normal urea and creatinine le&els. 8lasma
renin is undetectable and aldosterone le&els are raised. 9hat is the most likely
cause for his hypertension ?
.ushingBs s!ndrome
.onnKs s!ndrome
Your answer
5haeochromoc!toma
Renal arter! stenosis
Acromegal!
.onnBs s!ndrome is a condition where there is a benign adrenal adenoma that is secreting
aldosterone. The renin le+el is low* and h!po,alaemia is one of the common findings as well as
h!pertension. .ushingBs s!ndrome* acromegal! and phaeochromoc!toma are all associated with
secondar! h!pertension* but not with low renin and ele+ated aldosterone le+els. Renal arter!
stenosis has high renin le+els as well as high aldosterone le+els and so causes secondar!
h!peraldosteronism. Treatment of .onnBs s!ndrome is with surgical ecision of the adenoma or
with potassium-sparing diuretics .
26 . Jou are called to see a *A-year-old woman on the surgical ward who is 2 daysF
post-thyroidectomy. She is complaining of tingling around her mouth and in
her hands and has de&eloped spasm of her hands .
9hat immediate treatment can you gi&e that is most likely to resol&e her symptoms ?
As, her to breathe into a paper bag
Intra+enous calcium
Your answer
Intra+enous diazepam
Intra+enous glucose
Intra+enous potassium
5ostoperati+e complications of th!roidectom! include recurrent lar!ngeal damage* haemorrhage
and inad+ertent remo+al of the parath!roid glands* although actuall! less than #F of patients
suffer permanent h!pocalcaemia. The s!mptoms described are t!pical of h!pocalcaemia*
although the! can be seen in patients who are h!per+entilating. The treatment of choice to
relie+e the acute s!mptoms is #1 ml of #1F calcium gluconate. 2!pocalcaemia can be transient
after th!roid surger!. If it is permanent then the patient needs long-term therap! with
alfacalcidol to maintain her calcium le+els .
27 . A 2=-year-old woman who is * monthsF postpartum comes to the surgery
complaining of tiredness she has had no periods since the baby was born and
she has been unable to breast-feed because of a lack of milk production. Jou
notice in her case sheet that she reBuired a blood transfusion after deli&ery for
postpartum haemorrhage .
9hat is the most likely diagnosis ?
/mpt! sella s!ndrome
>elsonBs s!ndrome
5rolactinoma
;heehanBs s!ndrome
Your answer
;ippleBs s!ndrome
;heehanBs s!ndrome is h!popituitarism following pregnanc! complicated b! haemorrhage at the
time of deli+er!. The haemorrhage and associated h!potension is thought to cause pituitar!
infarction. 5atients fail to lactate and remain amenorrhoeic postdeli+er!* and also de+elop other
pituitar! hormone deficiencies of the th!roid and adrenal ais. >elsonBs s!ndrome is seen in
patients with pituitar!-dri+en .ushingBs s!ndrome man! !ears after bilateral adrenalectom!.
;uch patients are deepl! pigmented. /mpt! sella s!ndrome is associated with h!popituitarism*
imaging of the pituitar! fossa shows no ob+ious pituitar! tissue. The cause is un,nown. ;ippleBs
s!ndrome is M/>' (h!perparath!roidism* medullar! th!roid tumours and phaeochromoc!toma .(
28 . A *--year-old man presents to the diabetes clinic with a history of thirst
polyuria and a recent *.2 kg #7 lb$ weight loss. 5is urine contains a small
amount of ketones .
9hich of the following would suggest he is most likely to ha&e type-2 rather
than type-1 diabetes ?
A BMI of '$
2igh circulating insulin le+el
Your answer
2LA t!pe 3R-$
5ositi+e islet-cell antibodies
5lasma bicarbonate le+el of E mmol&l
T!pe-# diabetes mellitus is an autoimmune disorder characterised b! the presence* in man!
patients* of autoantibodies to the islet cell (I.A)* insulin (IAA) or glutamic acid deh!drogenase
(HA3). It is associated with 2LA 3R-$ or 3R-(. 5atients with t!pe-' diabetes more commonl!
ha+e a high BMI " '0* as this form of diabetes is associated with insulin resistance and high
insulin le+els rather than low insulin le+els. 4inall!* patients with t!pe-' diabetes mellitus can
ha+e O or OO ,etones in the urine* but donBt usuall! ha+e se+ere acidosis represented b! a
bicarbonate le+el of E mmol&l. This is more common in t!pe-# diabetes* with diabetic
,etoacidosis still a common and sometimes fatal complication .
29 . A A7-year-old man is referred to the hospital diabetes clinic with a new
diagnosis of type-2 diabetes mellitus. 5e has a ()% of 2G. 6reatinine le&el is
150 nmol+l and he has 10 protein on urinalysis. 5e has a past history of heart
failure .
9hich of the following drugs are you most likely to prescribe ?
.hlorpropamide
Hliclazide
Your answer
5ioglitazone
Rosiglitazone
Metformin
Hliclazide is a sulphon!lurea and can be used alone or in combination therap! for t!pe-'
diabetes mellitus. It acts b! increasing insulin release from the pancreatic P-cell. It can be used
in mild to moderate renal failure. 5ioglitazone and rosiglitazone are thiazolidinediones* which
promotes insulin sensiti+it! b! their action on the 55ARQ receptor (peroisome proliferator
acti+ated receptor-Q).The! are associated with fluid retention and are contraindicated in heart
failure. .hlorpropamide* a sulphon!lurea* is rarel! used now and is ecreted b! the ,idne!.
Metformin is a biguanide and* although its mechanism of action is not entirel! clear* it reduces
insulin resistance and hepatic glucose production. It is thought to be able to cause lactic acidosis
in certain circumstances and its use is contraindicated in patients with renal* hepatic or cardiac
failure .
30 . A 27-year-old woman with type-1 diabetes mellitus attends for her routine
re&iew and says she is keen on becoming pregnant .
9hich of the following is the factor most likely to make you ask her to defer
pregnancy at this stage ?
Minor bac,ground retinopath!
2b A#. M.(F
Your answer
;he hasnBt been ta,ing folic acid
Microalbumin le+el of ?-mg ecretion in '( hours
;ensor! neuropath!
5regnanc! in t!pe-# diabetes is still associated with a two- to threefold increase in congenital
abnormalities when compared with the bac,ground population. It is also associated with higher
neonatal morbidit! and mortalit! and higher operati+e deli+er! rates. 5repregnanc! counselling
with the aim of bringing the 2b A#c to near-normal le+els is associated with better pregnanc!
outcomes. 3iabetic retinopath! can progress during pregnanc! and the e!es must be chec,ed at
least once each trimesterA if there is progression that is sight-threatening then the patient
should be referred to an ophthalmologist. Bac,ground retinopath! would not be a
contraindication to pregnanc!. The microalbumin le+el is within normal limits. >ephropath! can
also progress during pregnanc!* but microalbuminuria would not be a contraindication to
pregnanc! and nor would sensor! neuropath!. It is important that patients with diabetes ta,e 0
mg folic acid dail! prepregnanc! (this reduces neural tube defects) and this can be started when
the patient starts tr!ing to concei+e* although it should ideall! be ta,en for $ months
beforehand. The most important thing this woman can do is to impro+e her gl!caemic control
before tr!ing to concei+e .
31 . A 17-year-old young woman with poorly controlled diabetes mellitus presents
with a temperature dehydration and altered consciousness. 5er initial
biochemistry shows sodium 1*0 mmol+l potassium -.5 mmol+l bicarbonate A
mmol+l urea 11.2 mmol+l creatinine 1*5 nmol+l and hydrogen ion =0 .
9hat is the most important immediate treatment ?
Intra+enous antibiotics
Intra+enous bicarbonate
Intra+enous fluids
Your answer
Intra+enous insulin
Intra+enous potassium
The patient has diabetic ,etoacidosis* a condition still associated with mortalit! in patients with
t!pe-# diabetes mellitus. There is a total bod! deficit of fluid and electrol!tes. The most
important initial treatment is intra+enous saline as the h!po+olaemia is the factor most li,el! to
cause the patient to die. Intra+enous insulin is re7uired at an earl! stageA and* although it is
normal* the potassium le+el will rapidl! fall with fluid and insulin treatment and the patient will
re7uire potassium replacement. Intra+enous bicarbonate should not be used immediatel! as it
can cause massi+e fluid shifts and precipitate cerebral oedema. It can be used if h!drogen ion is
greater than #11 and the patient is not responding to initial measures* in which case small
+olumes of a #.'?F bicarbonate solution should be used .
32 . A 5--year-old man newly diagnosed with type-2 diabetes mellitus presents
to the clinic for his first assessment. 5e is found to ha&e changes in his eyes
on fundoscopy .
9hich of the following is most likely to need immediate referral to the
ophthalmologist ?
A few dot and blot haemorrhages
;ome hard eudates " # disc diameter from the fo+ea
.ataract
>ew +essels on the disc
Your answer
Two soft eudates in the temporal field
Bac,ground diabetic retinopath! consists of dot and blot haemorrhages and hard eudates.
5atients do not need to be referred to the ophthalmologist unless these are within # disc
diameter of the fo+ea. This can be monitored annuall! at the routine clinic. .ataracts appear
about #1 !ears earlier in t!pe-' diabetes than in non-diabetic patients. If the +ision is
significantl! affected the patients warrant routine and not urgent referral to the
ophthalmologist. ;oft eudates suggest retinal ischaemia* which would re7uire routine referral
to the ophthalmologist. >ew +essels an!where in the fundus are a feature of proliferati+e
retinopath! and* as new +essels ha+e a ris, of haemorrhage and can threaten sight* the! should
be referred urgentl! to the ophthalmologist .
33 . A 7=-year-old man is admitted to hospital with a left hemiparesis and altered
consciousness. 5e is on aspirin 75 mg bendroflua"ide 2.5 mg ator&astatin 10
mg and glibenclamide 15 mg daily. 5is wife says he has been unwell for a
couple of days and has been off his food. She has still been gi&ing him all his
medication .
9hich of the following tests is going to be most helpful in finding an immediately
re&ersible cause for his symptoms ?
Blood glucose le+el
Your answer
.T brain scan
/.H
;erum creatinine le+el
Troponin le+el
2!pogl!caemia in the elderl! is a not uncommon problem* e+en if onl! treated with oral
h!pogl!caemic agents. Hlibenclamide has a long half-life and should therefore be a+oided in the
elderl!. The reduced food inta,e and the ongoing inta,e of medication in this patient is li,el! to
ha+e caused h!pogl!caemia* which can be associated with neurological s!mptoms in the elderl!.
The neurological s!mptoms will resol+e promptl! with intra+enous detrose. Although a
m!ocardial infarction or a cerebro+ascular accident could ha+e caused his s!mptoms* the!
would not be immediatel! re+ersible .
34 . A -*-year-old woman presents with weight loss palpitations diarrhoea and a
cessation of periods. She has been treated by her H8 for an<iety. ;<amination
re&eals a single nodule on the left of her thyroid about 1.5 cm in diameter.
4hyroid scanning with technetium shows increased uptake within the nodule
with reduced acti&ity throughout the rest of the gland. 4hyroid function tests
showed a free thyro<ine of *0 pmol+l #GK25 pmol+l$ 4S5 L 0.05 m!+l #0.5K5 .(
(ased upon these findings what would be the definiti&e treatment ?
Radioacti+e iodine therap!
.orrect answer
.arbimazole
;urgical ecision
Your answer
5ropanolol therap!
2igh-dose carbimazole therap! with th!roine replacement
Toic th!roid nodules preferentiall! ta,e up radioacti+e iodine. This ma,es them particularl!
amenable to radioacti+e iodine treatment. A dose of $11D011 MB7 is usuall! sufficient to cure
the th!rotoicosis. Radioiodine therap! is contraindicated in children and women who are
lactating or pregnant* and where the safet! of cohabitants from the effects of radioacti+it! canBt
be guaranteed. ;urger! is the net best option for patients in whom radioiodine is
contraindicated or refused. .arbimazole alone or in combination with th!roine replacement is
used in the medical treatment of Hra+eBs diseaseA 5ropanolol is a useful s!mptom relie+er for
th!rotoicosis .
35 . A *7-year-old woman presents to A:; after chasing thie&es who were stealing
her car. 5er blood pressure is noted to be 1=5+110 mm5g on admission. She
admits to episodic headaches and feeling stressed and an<ious. She is of
normal appearance her serum calcium on admission is noted to be 2.G5
mmol+l with normal renal function abdominal ultrasound re&eals a possible
adrenal mass .
9hat is the most likely diagnosis ?
5haeochromoc!toma
Your answer
.arcinoid s!ndrome
M/>-#
M/>-'a
.orrect answer
M/>-'b
M/>-' (multiple endocrine neoplasia D t!pe ') is associated with medullar! th!roid carcinoma
(almost alwa!s)* parath!roid chief-cell h!perplasia (#1D'0F) and phaeochromoc!toma ('1D
01F). M/>-'b is also associated with a marfanoid appearance* whereas M/>-'a is not. The
cause is an autosomal-dominant inherited genetic mutation on the long arm of chromosome #1.
.orrect management of this patient includes appropriate a- and P -bloc,ade prior to surgical
remo+al of the phaeochromoc!toma* screening for medullar! th!roid carcinoma with the i+
pentagastrin test and calcitonin assa!* then li,el! th!roidectom!* and probable
parath!roidectom! in the hands of an eperienced surgeon. The prognosis of M/>-' is +ariable*
but* o+erall* the #1-!ear sur+i+al rate is around ?0F .
36 . A 2--year-old adopted man presents with transient left-sided weakness of his
arm which resol&es after a few hours. 5is only other history of note is a
reduced libido and inability to maintain erections. 7n e<amination he appears
to ha&e a spotty skin pigmentation. Jou notice a heart murmur and there is
suggestion of a left atrial mass on echo. 5is prolactin is ele&ated at 2000
@mol+l .
9hat is the most likely diagnosis ?
Left atrial m!oma
.arne! comple
Your answer
5rolactinoma
5rotein . deficienc!
;omatisation disorder
This man has spott! s,in pigmentation* probable prolactinoma and a probable left atrial
m!oma. .arne! comple is diagnosable with two features out of spott! s,in pigmentation*
m!oma* endocrine tumours (commonest being primar! pigmented nodular adrenocortical
disease)* but it is also associated with ;ertoli-cell tumours* growth hormone- or prolactin-
producing pituitar! adenomas* th!roid adenomas and o+arian c!sts) and psammomatous
melanotic schwannoma (5M;). It is also diagnosable on the presence of one feature and an
affected first-degree relati+e. It is an autosomal-dominant condition caused b! an inacti+ating
mutation of protein ,inase A on chromosome #= .
37 . A 2=-year-old man presents to casualty with a sudden loss of &ision in his
right eye. 5is only past history of note is a pre&ious cerebellar haemorrhage.
7n e<amination he has e&idence of bilateral retinal angiomas and a partial
retinal detachment in his right eye .
9hat is the most likely diagnosis ?
;imple traumatic retinal detachment
.lotting disorder
Bleeding due to h!pertension
+on 2ippelDLindau disease
Your answer
Mc.uneDAlbright s!ndrome
@on 2ippelDLindau disease is characterised b! .>; and retinal haemangioblastomas (presenting
feature in (1F of patients)* renal c!sts and carcinomas (occurring later)* phaeochromoc!toma
('1F of affected families* bilateral in (1F of patients) and pancreatic tumours (01F non-
functioning). The pre+alence is # in $M*111 with a mean age at presentation of '= !ears. Renal
c!sts occur later* with =1F of patients ha+ing them b! age ?1 !ears. Ideall!* genetic testing in
affected families should ta,e place around the age of 0 !ears. Affected indi+iduals then re7uire
!earl! urinal!sis* catecholamine screening* and fluorescein angiograph!* with $-!earl! brain MRI
scanning .
38 . A *2-year-old woman presents with collapse. She works in an office
en&ironment and it has been a particularly hot day. 7n e<amination in casualty
she looks a little dehydrated. (8 is 110+70 mm5g .
4he following serum electrolyte results are obtained ;
/a
0
1*-mmol+l
1
0
2.Gmmol+l
)g
00
0.57 mmol+l #0.75-1.05 (
567*
-
*-mmol+l
9hat is the most likely diagnosis ?
BartterKs s!ndrome
HitelmanKs s!ndrome
.orrect answer
HordonKs s!ndrome
.onnKs s!ndrome
LiddleKs s!ndrome
Your answer
HitelmanBs s!ndrome is due to a mutation in the thiazide-sensiti+e >aD.l transporter in the
distal con+oluted tubule. It is associated with h!po,alaemia* h!pomagnesaemia (a cardinal
feature) and raised serum bicarbonate le+els. There is also h!pocalciuria. Treatment is with
potassium and magnesium replacement with or without potassium-sparing diuretics.
3ifferentiation between HitelmanKs and BartterKs s!ndromes is made b! measuring the urinar!
ecretion of magnesium (which is high in Hitelman s!ndrome and normal in Bartter s!ndrome)
and calcium (which is high in Bartter s!ndrome and low in Hitelman s!ndrome). BartterBs
s!ndrome presents earlier with h!po,alaemic al,alosis and h!percalciuria* due to a mutation in
the bumetanide-sensiti+e >a
O
D6
O
-'.l
-
transporter. Treatment is with potassium replacement*
non-steroidal anti-inflammatories and sometimes A./ inhibitors. LiddleBs s!ndrome is due to a
mutation in the distal nephron sodium channel* and is associated with h!po,alaemic al,alosis
and low renin and aldosterone le+els* but h!pertension still occurs. /ssentiall!* HordonBs
s!ndrome is the opposite of BartterBs s!ndrome* presenting with h!per,alaemia. .onnBs
s!ndrome is associated with h!pertension and is due to aldosterone ecess .
39 . A -5-year-old woman who works in a pharmacy presents with episodes of
tiredness and lethargy. 5er blood pressure is 115+75 mm5g. 5er bloods re&eal
hypokalaemia and a raised serum bicarbonate le&el. !rine collection re&eals
hypercalciuria. 7therwise the findings are unremarkable .
9hat is the likely diagnosis ?
BartterBs s!ndrome
HitelmanBs s!ndrome
Your answer
4rusemide abuse
.orrect answer
.onnBs s!ndrome
LiddleBs s!ndrome
This picture could fit BartterBs s!ndrome* although BartterBs s!ndrome is rare (# per million)*
has an autosomal-recessi+e pattern of inheritance and commonl! presents below the age of 0
!ears. 4eatures of BartterBs s!ndrome include +olume depletion* seizures* tetan! and muscle
wea,ness. This womanBs occupation is the clue* which unfortunatel! ma,es frusemide abuse
+er! much more li,el! .
40 . A -2-year-old man is referred to the hypertension clinic for ad&ice. 5e is
currently taking atenolol bendroflua"ide and ramipril and his blood pressure
is currently 1A5+105 mm5g. 5is potassium is *.0 mmol+l with a serum
bicarbonate concentration of 2= mmol+l .
9hat is the best ne<t management step ?
Measure the aldosteroneCrenin ratio
Your answer
9ash out as man! of his antih!pertensi+e agents as is possible for a period
of ' wee,s* then re+iew
.orrect
answer
Measure his '(-h blood pressure
Arrange '(-h urinar! free-cortisol collection
Add in a further agent and re+iew in #' months
The suspicion here with h!po,alaemia and metabolic al,alosis* and resistant h!pertension on
three agents* is that he has primar! h!peraldosteronism. A./ inhibitors and angiotensin II-
receptor bloc,ers* diuretics* calcium-channel bloc,ers* b -bloc,ers all ideall! re7uire a washout
period of ' wee,s to ma,e the aldosteroneC renin ratio assa! meaningful. 9hilst it ma! seem
dangerous to wash out* a definiti+e diagnosis is impossible if !ou donKt. ;pironolactone re7uires
a washout period of ? wee,s. A high aldosteroneC renin ratio is suggesti+e of primar!
h!peraldosteronism. The blood sample should be ta,en in the morning* standing* and with a
normalised potassium concentration (using supplementation) if possible. 8rinar! potassium
ecretion " $1 mmol&'( h ma! be another useful adjunct in ma,ing the diagnosis. ;urger! is
the treatment of choice for .onnBs adenoma and leads to resolution of h!pertension in around
=1F of patients* mitotane ma! be useful for controlling s!mptoms of adrenal carcinoma*
spironolactone is the medical treatment of choice for adrenal h!perplasia .
41 . A -2-year-old man is referred to the hypertension clinic for ad&ice. 5e is
currently taking atenolol 100 mg bendroflua"ide 2.5 mg and ramipril 10 mg
and his blood pressure is currently 1A5+105 mm5g. ;<amination is otherwise
unremarkable. 5is potassium is *.0 mmol+l with a serum bicarbonate
concentration of 2= mmol+l creatinine =5 @mol+l glucose tolerance is normal .
9hat is the most likely underlying diagnosis ?
.ushingBs disease
5rimar! h!peraldosteronism
Your answer
/ssential h!pertension
Renal arter! stenosis
5haeochromoc!toma
There is e+idence of h!po,alaemic metabolic al,alosis* probabl! associated with primar!
h!peraldosteronism. 2is resistant h!pertension in the presence of mar,ed h!po,alaemia
supports this. 5rimar! h!peraldosteronism accounts for at least 'F and perhaps up to #1F of
h!pertensi+e patients. The aldosteroneC renin ratio is li,el! to be raised if the patient is off
antih!pertensi+e medication* as is urinar! potassium ecretion. It is important* howe+er* to do
the blood testing in the morning* after a period of standing and wash out of this manBs
antih!pertensi+es for a period of ' wee,s if possible. 2!peraldosteronism does respond to some
etent to spironolactone* but surger! is the treatment of choice for .onnBs adenoma. Bilateral
adrenal h!perplasia* glucocorticoid-suppressible h!peraldosteronism and adrenal carcinoma are
other causes .
42 . A 2--year-old student has been reco&ering at home after a period of intensi&e
care and general medical admission for meningococcal septicaemia. - days
after discharge from hospital she &isits her H8 complaining of di""iness on
standing and profound tiredness. 7n e<amination she looks tired and Mwashed
outF and does indeed ha&e postural hypotension. (lood testing re&eals a
sodium concentration of 121 mmol+l potassium of A.7 mmol+l and urea of
15.0 mmol+l. She has mild normochromic normocytic anaemia .
9hat is the most likely underlying diagnosis ?
;econdar! adrenal insufficienc!
9aterhouseD4riderichsen s!ndrome
Your answer
M/
;!ndrome of inappropriate A32
2!poth!roidism
9aterhouseD4riderichsen s!ndrome is bilateral adrenal haemorrhage occurring due to massi+e
septicaemia* often associated with se+ere* life-threatening meningococcal disease. It ma!
present as in this patient with tiredness* letharg! and postural h!potension a short period after
discharge from the precipitating illness. .lassicall!* laborator! findings associated with adrenal
insufficienc! are h!ponatraemia* h!per,alaemia* ele+ated urea* anaemia* ele+ated /;R*
eosinophilia and mild h!percalcaemia. The commonest cause in the western world is
autoimmune disease (=1F)* closel! followed b! malignanc!* although TB is a common
association in the de+eloping world. /mergenc! treatment of adrenal insufficienc! in+ol+es fluid
replacement* I@ h!drocortisone and glucose supplementation* often e+en prior to determining
the underl!ing cause .
43 . A *=-year-old psychiatric patient who is being treated for depression is
admitted with increased confusion. 5is sodium is122 mmol+l with a plasma
osmolality of 2*5 m7smol+kg. !rine osmolality is *00m7sm+kg. 4here is no
e&idence on e<amination of cardiac renal or hepatic failure. Iandom plasma
cortisol and thyroid function are normal .
Hi&en the most likely diagnosis which of the following statements best fits this
condition ?
Reduced renal sodium ecretion is li,el!
Treatment with I@ saline is li,el! to be necessar!
Renal sodium ecretion is li,el! to be normal
Your answer
5s!chiatric drugs are +er! unli,el! to be related to the underl!ing condition
2!poth!roidism is a possible contributor
This is the s!ndrome of inappropriate antidiuretic hormone (;IA32) secretion. .riteria for
diagnosis are h!ponatraemia with an osmolalit! of less than '=1 m:smol&,g. Inappropriatel!
raised urine osmolalit! (" #11 m:smol&l).2igh urinar! sodium of " '1 mmol&l (reflecting
relati+e increased concentration due to reduced water loss)* and normal renal* adrenal and
th!roid function.;odium handling b! the ,idne! is not affected in ;IA32. 5s!chiatric drugs*
particularl! monoamine oidase inhibitors and phenothiazines ma! be possible causes. :ther
possible causes include tumours* such as small-cell lung cancer* other chest patholog!*
intracerebral haemorrhage or trauma* drugs such as carbamazepine or chlorpropamide and
metabolic causes such as h!poth!roidism. Treatment is with fluid restriction or tetrac!cline-li,e
compounds that promote water ecretion .
Another possible diagnostic consideration in a presentation of h!ponatraemia* especiall! gi+en
the ps!chiatric aspect here* is that of ps!chogenic pol!dipsia. 2owe+er* in this condition urine is
maimall! dilute (urine osmolalit! < #11m:sm&,g)* in contrast to ;IA32 .
44 . A ---year-old patient with hypomania is referred for opinion. She is noted to
ha&e a sodium concentration of 1-2 mmol+l with a urea of 12 mmol+l and a
creatinine of 1-0 @mol+l. !rine osmolality is 250 m7smol+l. 3asting plasma
glucose is normal. 4he nurses ha&e monitored her urine output and found it to
be -.- litres in 2- h .
9hich of the following statements best fits with her condition ?
5s!chiatric drugs are unli,el! to ha+e pla!ed a part in her condition
8rine osmolalit! rising to " $10 m:smol&l ( h after desmopressin in the water
depri+ation test is a positi+e result for cranial diabetes insipidus* it is unli,el! to
rise in this case
Your
answer
>ephrogenic diabetes insipidus usuall! has a dominant pattern of inheritance
3esmopressin in addition to her usual drugs is li,el! to be effecti+e
Indometacin is of no +alue
This patient has diabetes insipidus* probabl! renal in origin related to the use of lithium as
treatment for manic depression. 3ecompensated diabetes insipidus has features of deh!dration
(as in this case)* with an inappropriatel! low urine osmolalit!. :b+iousl! diabetes mellitus needs
ecluding as a cause of pol!uria before a diagnosis of diabetes insipidus can be made.
5s!chogenic pol!dipsia is ruled out b! the water depri+ation test* with urine osmolalit! rising
appropriatel! in response to the lac, of oral inta,e. The urine remains dilute in cases of
nephrogenic diabetes insipidus* despite a rising plasma osmolalit! and urine failing to
concentrate after desmopressin. 3esmopressin is unli,el! to be effecti+e in this case*
indometacin ma! be effecti+e. 2owe+er* if possible* the lithium should be withdrawn in
consultation with a ps!chiatrist* and another drug such as lamotrigine substituted. >ephrogenic
diabetes insipidus has an autosomal or I-lin,ed recessi+e pattern of inheritance* whereas
cranial diabetes insipidus is inherited onl! with an autosomal-dominant pattern .
45 . A A2-year-old man presents with bony pain that has been present for some
months particularly affecting his left femur pel&is and lower back. (lood
testing re&eals a normal serum calcium le&el but a raised alkaline
phosphatase. ?-rays of the femur and pel&is re&eal mi<ed lytic and sclerotic
change with accentuated trabecular markings. 6hest ?-ray is normal .
9hat is the likely diagnosis ?
;econdar! carcinoma
Multiple m!eloma
2!perparath!roidism
2!poparath!roidism
5agetBs disease
Your answer
5agetBs disease is thought to be present in 'F of the population abo+e 00 !ears of age* with
M1F being as!mptomatic. It is more common in .aucasian populations and rarer in populations
of African descent. There are said to be three phases associated with the time course of
radiological changes seen in 5agetBs disease. /arl! disease is said to be primaril! l!tic* then
there are mied l!tic and sclerotic changes* progressing later to primaril! sclerotic change with
increasing bon! thic,ening. Hoals of treatment are to normalise bone turno+er* maintain the
al,aline phosphatase le+el within the normal range* minimise s!mptoms and pre+ent long-term
complications. The mainsta! of treatment for this has been use of the bisphosphonates. These
are now often gi+en as intermittent i+ courses a few wee,s apart. Long-term complications
include deafness (in up to 01F of patients with s,ull-base 5agetBs disease)* and +ar! rarel!
osteogenic sarcoma .
46 . A *2-year-old woman presents with amenorrhoea for A months. A pregnancy
test is negati&e. 7&er the past few months she has occasionally been leaking
milk and presents now as this has occurred more and more during stimulation
and intercourse and she is becoming distressed by it. 4hyroid function testing
is normal. She is on no medication. 5er serum prolactin le&el is 2-00 m!+l and
a 64 scan of the pituitary is unremarkable .
9hich of the following best fits her condition ?
;he is li,el! to ha+e a macroprolactinoma
;he should be obser+ed for #' months
.abergoline is effecti+e therap!
Your answer
;urger! is the best option here
A +isual field defect is li,el!
;he most li,el! has a microprolactinoma* which would appear as a h!podense area on MRI
scanning. ;erum prolactin is often in the range #011 m8&l to $111 m8&l in patients with
microadenomasA le+els are usuall! abo+e $111 m8&l in those with macroadenomas. Although
surger! in conjunction with dopamine-agonist therap! is the treatment of choice for patients
with macroadenomas* microadenomas often respond well to cabergoline (E$F normalisation of
prolactin). A +isual field defect is unli,el!. :bser+ation in these patients is not usuall! preferable
to normalisation of prolactin le+els with dopamine agonistsA if this obser+ational approach is
ta,en then ade7uate se-hormone replacement should be underta,en. :f course* normalisation
of prolactin ma! restore fertilit!* and the patient should be warned of this possibilit! .
47 . A 55-year-old woman presents with her husband to the endocrine clinic. She
has distressing symptoms of sweating and her husband noticed increased
prominence of her Caw when he was archi&ing photos from recent years. 2ast
year she was diagnosed with type-2 diabetes. 7ther past history of note is
that she has recently been operated on for carpal tunnel syndrome .
9hich of the following most likely to fit with her condition ?
Random growth-hormone le+el is li,el! to be < # m8&l
Your answer
IH4-# le+els are li,el! to be normal
Hrowth-hormone le+els are li,el! to remain abo+e ' m8&l after a =0-g
glucose load
.orrect
answer
1-25 - :2 +itamin 3 le+el is in+ariabl! normal
2!pertension coeists in '1F of patients with this condition
The diagnosis here is acromegal!. A random growth-hormone le+el of < # m8&l ecludes the
diagnosis* growth-hormone le+els usuall! remain abo+e ' m8&l after an oral glucose tolerance
test. There are increased le+els of #* '0-:2 +itamin 3 in some patients. IH4-# is in+ariabl!
ele+ated in patients with acromegal!. ;weating is present in more than E1F of cases*
h!pertension in (1F* impaired glucose tolerance in (1F and fran, t!pe-' diabetes in '1F.
.oarse facial features including prognathism are often noticed b! patients on comparing old
photographs* enlargement of the hands and feet and soft tissue swellings are also common
features. Hrowth hormone-secreting pituitar! tumours are usuall! identified on MRI scanning
and trans-sphenoidal resection is the treatment of choice .
48 . A *2-year-old woman presents with e<treme lethargy a couple of weeks after
the birth of her third child by emergency caesarean section. 4he reason for the
caesarean section was se&ere blood loss and hypotension. She complained to
the health &isitor of increasing problems some 7 days earlier but was told that
this was to be e<pected after the birth of her child. 7n admission &ia casualty
she was noted to ha&e a sodium concentration of 127 mmol+l a potassium
concentration of 5.0 mmol+l and a urea of 12 mmol+l .
9hat is the likely diagnosis ?
;heehanBs s!ndrome
Your answer
2!poth!roidism
5rimar! adrenal failure
5ostnatal depression
3eh!dration
;he has suffered a period of h!potension and blood loss associated with her emergenc!
caesarean section. This has resulted in pituitar! infarction* and she presents now with
s!mptoms of h!poadrenalism* which include h!ponatraemia and potassium at the upper limit of
normal. ;he* of course* re7uires fluid reh!dration and emergenc! steroid replacement with i+
h!drocortisone. ;he will also ha+e pituitar!-dependent h!poth!roidism and re7uire th!roine
replacement. Restoration of fertilit! is more difficult* pulsed deli+er! of pituitar! se-ais
hormones is usuall! re7uired. 5ostnatal depression and simple deh!dration are somewhat
unli,el! with this set of blood results .
49 . A 25-year-old o&erweight woman presents with hirsutism and
oligomenorrhoea. She has been unable to concei&e for 1= months. 4he
adrenals appear normal on ultrasound scanning but an o&arian ultrasound
scan re&eals numerous small cysts in both o&aries .
9hich of the following is likely to fit best with her diagnosis ?
The L2&4;2 ratio is li,el! to be normal
;e hormone-binding globulin is low in 01F of sufferers with this
condition
Your answer
Testosterone le+els are usuall! normal
4ertilit! is usuall! unaffected b! this condition
32/A; is usuall! normal or low
This is pol!c!stic o+arian s!ndrome (5.:;). The pre+alence of 5.:; is estimated to range from
E to ''F of women. :ligo&amenorrhoea is present in =1F* hirsutism in ?1F* obesit! in $0F
and infertilit! in $1F. L2 is usuall! mar,edl! raised* as is the L2&4;2 ratio. ;e hormone-
binding globulin is low in 01F of sufferers. 3ue primaril! to h!perinsulinaemia.Testosterone and
32/A; (deh!droepiandrosterone) le+els are also usuall! raised. 8p to (1F of women with
5.:; ma! ha+e impaired glucose tolerance* and up to #1F fran, t!pe-' diabetes mellitus. The
insulin-resistant state is also associated with d!slipidaemia. Management is usuall! with weight
loss and lifest!le ad+ice. Metformin is a useful treatment for insulin resistance and ma! help
restore fertilit!* although clomifene ma! also be re7uired for o+ulation induction. Local creams
and electrol!sis ma! ha+e a useful role to pla! if the primar! complaint is one of hirsutism .
50 . A *=-year-old woman presents to the clinic with difficult to treat hypertension.
She is on two agents and currently has a (8 of 155+G5 mm5g. She has noted
that her face has become more rounded o&er the years and she is ha&ing
increasing trouble with both acne and hirsutism. 3asting blood glucose testing
has re&ealed impaired glucose tolerance. 4here has also been increasing
trouble with abdominal obesity and she has noticed some purple stretchmarks
appearing around her abdomen .
9hat is the most likely diagnosis ?
5haeochromoc!toma
Multiple endocrine neoplasia
/ssential h!pertension
;imple obesit!
.ushingBs s!ndrome
Your answer
.ushingBs s!ndrome is associated with a round* plethoric* facial appearance and weight gain D in
particular truncal obesit!* buffalo hump and supracla+icular fat pads* s,in fragilit!* proimal
muscle wea,ness* mood disturbance* menstrual disturbance and reduced libido. 2!pertension is
present in more than 01F of patients* impaired glucose tolerance in $1F. :steopenia and
osteoporosis* and premature +ascular disease are an ine+itable conse7uence in untreated
.ushingBs. The annual incidence is approimatel! ' per million and the disease is commoner in
women. The disease results from h!percortisolaemia and the cause in ?EF of cases is a
pituitar! adenoma producing A.T2. /ctopic A.T2 production is the cause in #'F* adrenal
adenoma in #1F and adrenal carcinoma in EF. 3iagnosis is made on the results of the '(-h
urinar! free-cortisol assa!. 3ifferentiation as to the cause is carried out with the
deamethasone-suppression test and selecti+e +enous sampling in cases when a discrete
tumour cannot be identified on contrast-.T scanning .
51 . A 2--year-old woman presents with 1* months of amenorrhoea. 3or the past
few months she has been e<periencing hot flushes night sweats mood
changes and pain on intercourse. 3S5 has been N -0 m%!+l on two separate
occasions and her serum estradiol le&el is low. 4S5 is normal. 3asting blood
glucose is normal. 8regnancy test is negati&e .
9hat is the most likely diagnosis ?
5remature o+arian failure
Your answer
5ol!c!stic o+arian s!ndrome (5.:; (
Androgen-secreting adrenal tumour
5ituitar! failure
Th!rotoicosis
The blood picture of ele+ated 4;2* low estradiol and prolonged amenorrhoea fits the picture.
The normal T;2 rules out th!rotoicosis* and the raised 4;2 ma,es 5.:; unli,el! and this does
not fit the picture of an androgen-secreting adrenal tumour. Autoimmune disease is responsible
for '1F of cases of premature o+arian failure* and is pre+alent in #1F of women with AddisonBs
disease and '0F of women with autoimmune th!roid disease. ;pontaneous reco+er! of fertilit!
is unli,el!* and put at onl! 0F .
52 . A ---year-old woman has attended A:; on a number of occasions this year
because of renal tract stones. She has also suffered depression during the past
year or two. She is found to ha&e a serum calcium of *.10 mmol+l #2.-K2.A$
creatinine of 1*= @mol+l and albumin of -0 g+l .
9hat is her most likely underlying diagnosis ?
2!perparath!roidism
Your answer
4amilial h!percalcaemic h!pocalciuria
Multiple endocrine neoplasia
2!poparath!roidism
5seudoh!poparath!roidism
This woman most li,el! has h!perparath!roidism. 2!perparath!roidism is a feature in M0F of
patients with M/>-# (multiple endocrine neoplasia D t!pe #) and ma! coeist with prolactin- or
growth hormone-producing pituitar! adenomas* pancreatic islet cell tumours* non-functioning
adrenal adenomas or th!roid adenomas. As part of the M/>-' s!ndrome* there is an association
with phaeochromoc!toma* .ushingBs s!ndrome and medullar! carcinoma of the th!roid. ;hort
metacarpals (usuall! the fourth or fifth) are associated with pseudoh!poparath!roidism* which*
in turn* is associated with h!pocalcaemia due to an abnormalit! of the H-protein receptor for
parath!roid hormone. In the case of parath!roid h!perplasia or parath!roid adenoma* a surgical
solution is the best option where s!mptoms such as renal stones ha+e begun to appear .
53 . A -0-year-old man presents with a serum calcium concentration of *.05
mmol+l and urinary calcium e<cretion of 12 mg+2- h. 4here is no history of
renal stones pancreatitis depression or any prior illness. 5e was referred by a
particularly "ealous H8 and underwent parathyroidectomy .
9hich of the following is true of the disease which underlies this manOs case history ?
2e has h!perparath!roidism that is li,el! to ha+e been cured b! the
surger!
8rinar! calcium ecretion is increased
Your answer
Inheritance is autosomal dominant
.orrect
answer
Acute pancreatitis is commonl! associated with the condition
The bod! has an increased abilit! to sense raised calcium le+els
This is not h!perparath!roidism. This is familial h!pocalciuric h!percalcaemia. It has an
autosomal-dominant pattern of inheritance with +irtuall! complete penetrance. The
heteroz!gotic state is commonl! as!mptomatic* but homoz!gotes present with se+ere
h!percalcaemia soon after birth and re7uire parath!roidectom!. :ccasionall! a subgroup of
adult heteroz!gotes does present with recurrent pancreatitis and ma! re7uire
parath!roidectom!. The mutation causes a reduced abilit! for the calcium sensor to detect
h!percalcaemia* so the bod! tolerates le+els of serum calcium that would usuall! be said to be
outside the normal range .
54 . A 51-year-old woman presents to her H8 with polyuria tiredness and a
random plasma glucose le&el of 1*.0 mmol+l. According to the ADA criteria
what should happen ne<t ?
;he should be reassured that the result is normal
;he ma! ha+e diabetes mellitus and re7uires a fasting blood test the
following da! to confirm the diagnosis
Your
answer
;he probabl! has impaired glucose tolerance and should undergo a '-h
glucose tolerance test
A '-h glucose tolerance test plasma glucose le+el of #1.0 mmol&l would
confirm diabetes mellitus
A fasting plasma glucose of =.' mmol&l the following da! would suggest
impaired fasting glucose
The American 3iabetes Association (A3A) criteria were de+eloped in #MM=* with a suggestion
that a glucose tolerance test should be a+oided* if possible* to ma,e diagnosis simpler and
7uic,er. Three methods of classification of diabetes mellitus were de+elopedC s!mptoms and a
random plasma glucose of greater than or e7ual to ##.# mmol&lA fasting plasma glucose of
greater than or e7ual to =.1 mmol&lA and =0-g oral glucose tolerance test '-h glucose of greater
than or e7ual to ##.# mmol&l. These diagnostic criteria still stand the test of time toda!. Ideall!
random raised fasting plasma glucose should be confirmed with a fasting sample. JImpaired
fasting glucoseB (analogous to Jimpaired glucose toleranceB)* is defined as a plasma glucose le+el
of greater than or e7ual to 0.? mmol&l and less than =.1 mmol&l .
55 . A 2A-year-old woman has been recently diagnosed with type-1 diabetes. She
has read a great deal about the prognosis of renal disease in type-1 diabetes
and has a number of Buestions to ask .
9hich of the following statements best describes the renal disease in patients
with type-1 diabetes ?
Microalbuminuria usuall! occurs within ' !ears of the diagnosis of t!pe-#
diabetes
5ea, incidence of fran, albuminuria is #= !ears after the diagnosis of t!pe-
# diabetes
.orrect
answer
After the serum creatinine le+el reaches '11 Rmol&l* a fall in H4R of 1.0
ml&min per month might be epected
/nd-stage renal failure usuall! occurs within 0 !ears of the onset of
albuminuria
Your
answer
>ephropath! ha+e a #1 times higher mortalit! rate than age- and se-
matched controls
In t!pe-# diabetes microalbuminuria usuall! occurs 0D#0 !ears after the diagnosis of diabetes
mellitus. The pea, incidence of fran, albuminuria is around #= !ears after diagnosis. It is
reasonable to epect a fall in the glomerular filtration rate (H4R) of # ml&min per month once
the serum creatinine le+el has reached around '11 Rmol&traditionall!* end-stage renal failure is
said to occur some =D#1 !ears after a diagnosis of albuminuria* but it is hoped that impro+ed
management of h!pergl!caemia and h!pertension ma! stretch this period a little. 8nfortunatel!*
nephropath! carries a '1D#11 times higher mortalit! than that for age- and se-matched
controlsC (1F of deaths of patients with nephropath! are due to cardio+ascular disease .
56 . A 70-year-old man is brought unconscious to the emergency department, his
blood sugar le&el is 70 mmol+l. 4here is no e&idence of ketoacidosis. 5is chest
?-ray re&eals e&idence of left-sided consolidation .
9hat is the most likely diagnosis ?
;imple pneumonia
A complication of t!pe-# diabetes
A complication of t!pe-' diabetes
Your answer
;tro,e
Hlucagonoma
2e is most li,el! to be suffering from h!perosmolar non-,etotic coma* which is associated with
t!pe-' diabetes and with coeistent infection. The common presentation is often with blood
glucose le+els of o+er 01 mmol&l* a mar,edl! raised serum osmolalit! and a decreased
conscious le+el* without ,etosis. Management is with i+ insulin and isotonic saline infusion*
although the blood glucose le+el often drops 7uic,l! after insulin is initiated. 2!perosmolar
coma ma! be the initial presentation of diabetes mellitus* and two-thirds of sufferers present
with t!pe-' diabetes mellitus for the first time. Mortalit! is high* particularl! in elderl! sufferers.
Hlucagonoma is +anishingl! rare* with an annual incidence of #&'1 million in the 86 .
57 . A A5-year-old obese woman who has had type-2 diabetes for - years presents
to the clinic for her annual re&iew. 5er current medication is metformin 2
g+day. Jou decide to add in glicla"ide =0 mg po bd because of poor control as
her 5b A16 is =.5P .
9hich of the following best describes sulphonylureas ?
In the 8653; stud! the! demonstrated no effects on micro+ascular
outcome
Your answer
In the 8653; stud! the! demonstrated no effect on macro+ascular
outcome
.orrect
answer
In the 8653; stud! the! demonstrated positi+e effects on macro+ascular
outcome
Mild h!pogl!caemia in response to sulphon!lurea therap! is rare
Hliclazide has positi+e effects on insulin resistance
Although sulphon!lurea therap! was pro+en b! the 8653; stud! to ha+e micro+ascular
benefits* no benefit on macro+ascular outcome (cardio+ascular disease) was demonstratedC this
was* of course* in contrast to metformin. ;ulphon!lureas wor, b! stimulating insulin release
from pancreatic beta cells and are associated with mild h!pogl!caemia in up to #?F of users*
although h!pogl!caemia ma! be less fre7uent when short-acting compounds are used. .ertain
groups of people (the elderl! and those with renal impairment) ma! be at a higher ris, of
h!pogl!caemia. ;ulphon!lureas ha+e no effect on insulin resistance .
58 . A 5A-year-old highly insulin resistant type-2 diabetes sufferer has been
taking 200 units total daily dose of S6 insulin per day. 5is weight is increasing
and his control worsening with an 5b A16 of G.1P. Jou add in pioglita"one *0
mg to his insulin. Some - weeks later he presents to the emergency
department in heart failure .
9hich of the following statements best describes glita"one therapy ?
Hlitazones cause heart failure b! eerting a directl! toic effect on the
m!ocardium
Your
answer
Hlitazones ha+e no more peripheral insulin-sensitising effects than
metformin
There is e+idence that all three agents in the glitazone class (troglitazone*
pioglitazone and rosiglitazone) ha+e similar profiles of hepatotoicit!
2eart failure ma! be precipitated in some patients ta,ing glitazones due to
fluid retention
.orrect
answer
Hlitazones act primaril! at the site of the 55AR-S receptor
There are currentl! two licensed glitazones (insulin sensitisers) world-wide* rosiglitazone and
pioglitazone. Both drugs act at the site of the 55AR-Q receptor* promoting the transcription of
genes predominantl! lin,ed to fatt! acid metabolism. 4ibrates* and some newer dual agonists
currentl! in de+elopment* ha+e acti+it! at the 55AR-S receptor site. 4luid retention ma! be
precipitated b! these drugs* which ma! (in less than #F of users) precipitate heart failure.
/perimental e+idence has demonstrated that these drugs actuall! impro+e m!ocardial blood
flow and glucose utilisation. Metformin acts primaril! b! affecting hepatic insulin resistance*
whereas the glitazones act b! impro+ing insulin sensiti+it! and peripheral glucose upta,e
ins,eletal muscle and fat. Troglitazone has been withdrawn due to li+er side-effects .
59 . A 17-year-old young woman presents to the emergency department with
blood glucose of 2G mmol+l. She is known to ha&e type-1 diabetes. 5er p5 is
7.12 with serum bicarbonate of 11 mmol+l. 4here is ketonuria .
9hich of the following statements best fits the predisposing factors in&ol&ed in D1A ?
M!ocardial infarction ma! be the precipitating factor in up to 0F of cases
of 36A
Infection ma! be the precipitating cause in ?1F of cases of 36A
The patient is not pre+iousl! ,nown to ha+e diabetes in $1F of 36A
cases
Your answer
>on-compliance with treatment is the cause in '0F of 36A cases
.orrect
answer
Inappropriate alterations to insulin are the cause in '1F of 36A cases
The commonest precipitant of diabetic ,etoacidosis (36A) is infection ($1D(1F). This is closel!
followed b! non-compliance with treatment ('0F)* alterations to insulin dose (#$F)* newl!
diagnosed diabetes (#1D'1F) and m!ocardial infarction (< #F). The condition is characterised
b! h!pergl!caemia* acidosis and ,etonuria. 36A is common* with M.#F of patients in the
/8R:3IAB stud! reporting hospitalisation o+er the period of a !ear. 36A is caused b! insulin
deficienc! and counter-regulator! hormone ecess. The mortalit! rate is 'D0F* but up to 01F
in more elderl! patients .
60 . A *A-year-old woman who is 2= weeks pregnant attends the midwife clinic for
a pregnancy check. She is noted to ha&e glycosuria and has a ()% of *0 Jou
understand her mother has 4ype-2 diabetes .
9hat is her correct management plan ?
:bser+e and follow up in ( wee,sK time with repeat urine glucose testing
.arr! out a fasting blood glucoseA if it is less than =.1 mmol&l then repeat
the sample in ( wee,sB time
Your
answer
.arr! out a fasting blood glucoseA if it is o+er 0.0 mmol&l then proceed to
administer a =0-g oral glucose tolerance test
.orrect
answer
.arr! out a fasting blood glucoseA it is o+er =.1 mmol&l then proceed to
administer a =0-g oral glucose tolerance test
5roceed directl! to administer an oral glucose tolerance test
A glucose tolerance test is re7uired if the patientBs fasting blood glucose le+el is o+er 0.0
mmol&l. 3iabetes is defined as a fasting le+el of " =.1 mmol&l or " ##.1 mmol&l ' h after a
glucose tolerance test. Hestational impaired glucose tolerance (IHT) is defined as a fasting blood
glucose le+el of ?.1D=.E mmol&l and&or a '-h le+el of MD## mmol&l. Although initial treatment for
gestational diabetes is with dietar! ad+ice* insulin is also re7uired in #1D$1F of cases. Insulin
should be considered for those with fasting le+els abo+e ?.1 mmol&l after dietar! inter+ention*
or postprandial sugar le+els abo+e E.1 mmol&l. 8ntreated gestational diabetes is said to carr! a
perinatal mortalit! rate of (.(D?.(F* compared to 1.0D#.0F in a similarl! matched
normogl!caemic populationC hence* good gl!caemic control is essential for this reason .
61 . A 2=-year-old mature student nurse is admitted from the emergency
department where she is on placement ha&ing suffered a collapse. 5er blood
sugar was noted to be 1.5 mmol+l and she responded to 50 ml of 50P
i&de<trose. After a second collapse her insulin le&el was raised without a
similar rise in c-peptide, her glucose le&el was 1.G mmol+l .
9hat is the most appropriate ne<t in&estigation ?
A $-da! fast in an attempt to precipitate a further attac,
Audit of insulin stoc,s that she has had access to
Your answer
Alcohol histor!
.hest I-ra! to eclude malignanc!
3ischarge the patient
The clue is the fact that her insulin le+el was raised without a corresponding rise in c-peptide.
This essentiall! rules out insulinoma* and suggests that the patient is either injecting ecess
insulin herself or is ta,ing sulphon!lureas. In fact* on further 7uestioning her aunt has t!pe-'
diabetesA she has been struggling with her studies and ta,ing sulphon!lureas to precipitate the
attac,s as a Jcr! for helpB. ;tandard wor,-up for h!pogl!caemia should include laborator! blood
glucose (not test strips) measurement* li+er function tests to rule out significant li+er
d!sfunction* blood alcohol and alcohol histor!* cortisol T ;!nacthen testing* insulin and c-
peptide le+els ta,en during an attac, as well as a chest I-ra! to eclude occult malignanc! .
62 . A A=-year-old man attends his H8 on the insistence of his wife because his left
forefoot has become increasingly unstable and abnormally shaped with bony
swelling. 5e has a long history of diabetes mellitus and his control has been
erratic with 5b A16 in the range of GK10P .
9hich of the following pieces of information best fits the pathology or management of
6harcotFs foot ?
Tight glucose control is unli,el! to be of benefit in this condition
Reduction of oedema is unimportant with respect to pre+enting deterioration
:rthotic shoes are unnecessar! in the management of this condition
5lain radiograph! is alwa!s abnormal* e+en earl! in this condition
The patholog! of this condition is thought to be due to s!mpathetic
d!sfunction* ecessi+e blood flow to the joint and osteoclast acti+it!
Your
answer
This man has a .harcot left foot. This is a rare condition now* with the a+erage general hospital
clinic seeing a maimum of #1 .harcotBs patients. .hronic untreated .harcotBs foot results in
either Jroc,er-bottomB foot due to downward displacement and subluation of the tarsus* or
medial con+eit! due to talona+icular joint displacement or dislocation of the tarsometatarsal
joints. Management in+ol+es tight gl!caemic control* reduction of oedema and orthotic and
chiropod! in+ol+ement. Intra+enous bisphosphonate treatment ma! be of benefit in some
patients. Although plain radiographs ma! be normal in the earl! stages of the disease* the!
later show joint destruction* osteol!sis* joint reorganisation and subluation .
63 . A 5A-year-old patient on glicla"ide for his type-2 diabetes presents with an
acute central crushing chest pain. 5e is diagnosed with myocardial infarction .
9hich of the following best fits the outcome or management of myocardial infarction
associated with type-2 diabetes ?
The mortalit! rate from m!ocardial infarction in patients with t!pe-'
diabetes is the same as that for non-diabetics
Intra+enous insulin followed b! sc insulin after MI reduces mortalit! b!
##F at $.0 !ears compared to controls
Your
answer
8se of A./ inhibition after MI impro+es the ?-wee, mortalit! rate b! 01F
;tatins should alwa!s be started unless the! are contra-indicated
.orrect
answer
Blood pressure target should be #01&E1 mm2g
3iabetics ma! be up to twice as li,el! to die from m!ocardial infarction as non-diabetics* and
are more li,el! to suffer an MI in the first place. The 3IHAMI stud! used i+ insulin for '( h
followed b! sc insulin for patients who had suffered an MI. Ris, reductions e7ualled =.0F at #
!ear and ##F at $.0 !ears compared to controls. There is now significant doubt about this
result as the stud! was repeated in the 3IHAMI-' stud!. This showed no ad+antage in terms of
cardiac outcomes for patients treated for a prolonged period with insulin. 8se of A./ inhibitors
(HI;;I-$ 3iabetic ;ubgroup ;tud!) is associated with a $1F relati+e-ris, reduction in the ?-
wee, mortalit! rate (E.=F +s. #'.(F). Blood pressure reduction should be aggressi+el!
targeted* aiming for #$1&E1 mm2g* and all patients with diabetes who are o+er (1 !ears of age
should be started on statin treatment unless there is a contraindication* (Uoint British ;ocieties
Huidelines .(
64 . A 51-year-old patient on metformin and glicla"ide for his type-2 diabetes
presents for re&iew. 5e has a ()% of *1 and his blood pressure is 15=+G2
mm5g .
9hen considering hypertension in type-2 diabetes which of the following statements
best fits the condition ?
3e+elopment of h!pertension is strongl! lin,ed to a geneticall! inherited
+ariant of red cell sodiumDlithium counter-transport acti+it!
Hood h!pertensi+e control has a wea,er effect on macro+ascular outcomes
than good tight glucose control
2!pertension in t!pe-' diabetes is primaril! associated with h!perinsulinaemia
and insulin resistance
Your
answer
2!pertension affects up to '1F of patients with t!pe-' diabetes
Thiazide diuretics are the treatment of choice in patients with h!pertension
and microalbuminuria
2!perinsulinaemia directl! stimulates s!mpathetic ner+ous s!stem acti+it!* increasing renal
sodium reabsorption and promoting +ascular smooth muscle proliferation. Insulin resistance is
also associated with the reduced acti+it! of nitric oide s!nthase* leading to reduced
+asodilatation in response to +ascular stress. Red cell sodiumDlithium counter-transport acti+it!
abnormalities are associated with the earl! de+elopment of h!pertension in t!pe-# diabetes.
Between $1 and 01F of .aucasians with t!pe-' diabetes ha+e a s!stolic blood pressure abo+e
#?1 mm2g.A./ inhibitors are the treatment of choice for diabetic h!pertension* particularl!
where microalbuminuria coeists. Thiazide diuretics and P-bloc,ers ma! worsen gl!caemic
control .
65 . A 52-year-old man is sent by his H8 for an urgent re&iew. 5e has been
maintained on metformin and glicla"ide for his type-2 diabetes. 3or the past
week or so he has had feelings of se&ere aching pain and paraesthesias in his
upper legs. 5e has felt Moff his foodF during the past week and has begun
losing weight. 4here is also pro<imal muscle weakness .
9hich of the following best fits diabetic amyotrophy ?
75% of patients reco+er full! from this condition
Transference to insulin therap! is the mainsta! of treatment
.orrect answer
Reco+er! from this condition usuall! ta,es o+er # !ear
The condition is much more common in t!pe-# diabetes
9eight loss is unli,el! to be related to the diagnosis
Your answer
3iabetic am!otroph! is said to occur most commonl! in men in their fifties with t!pe-' diabetes
treated with oral h!pogl!caemic agents. It is a mied motor and sensor! proimal neuropath!
said to cause se+ere pain* which is responsible for anoreia and weight loss. ;ome 01F of
patients reco+er full! from this condition* usuall! within $D( months. The mainsta! of treatment
is supporti+e care and transference to insulin therap! .
66 . A A*-year-old man presents to the diabetes clinic for re&iew. 5e attends with
his wife who has accused him of a lack of interest in her as he is no longer
able to sustain an erection .
9hich of the following statements is most strongly associated with impotence in type-
2 diabetes ?
Libido is often also affected in addition to ph!sical patholog!
/stimates sa! that '1F of men with diabetes mellitus of more than ?
!earsB duration are affected
Your answer
:ther autonomic neuropath! is unli,el! to be present
5enile pain is a common association
b-Bloc,ers and thiazide diuretics ma! eacerbate the problem
.orrect
answer
Libido is rarel! affected* which ma,es the condition all the more frustrating for affected men.
Between $1 and 01F of men with diabetes of more than ? !earsB duration are affected. There is
an association with other autonomic neuropathies such as postural h!potension and
gastroparesis. The condition is not usuall! associated with pain* and ma! be eacerbated b!
both P -bloc,ers and thiazide diuretics. 53/-0 inhibitors are now a+ailable as oral therapies for
impotence* but these are often re7uired at high dose. Alprostadil and +acuum de+ices are
alternati+e methods. /stimates put successful treatment at onl! around =1F .
67 . A 5--year-old woman presents to the diabetes clinic for re&iew. She has
suffered from diabetes mellitus for many years and has now progressed from
sulphonylurea treatment to insulin therapy. A past history of gallstones is
noted and she failed to tolerate metformin due to chronic diarrhoea. She has
been slowly losing weight o&er the past few months and has mild anaemia.
!ltrasound scan of her li&er re&eals a suggestion of a number of small
metastases within the li&er .
9hen considering that this woman has a primary pancreatic tumour which of the
following statements best fits the condition ?
;omatostatin* A.T2 and calcitonin ma! all be raised
.orrect
answer
.ontrast spiral-.T scanning is effecti+e in demonstrating the primar!
tumour in M1F of cases
Your answer
.urati+e surger! is possible in the majorit! of cases
There is almost alwa!s an association with M/>-#
Most tumours are detected because of the clinical s!ndrome
The histor! is suggesti+e of a somatostatinoma* the annual incidence of which is estimated at #
in (1 million. There is a M0F association with impaired glucose tolerance (IHT) or diabetes
mellitus* ?EF with gallstones* weight loss '0F and anaemia #(F* there is also an association
with diarrhoea. The tumours are often multisecretor! and A.T2 and calcitonin le+els ma! be
raised in addition to that of somatostatin. There is an association with M/>-# in some (=F).
Man! tumours ma! occur undetected* and not causing the somatostatinoma s!ndrome. .ontrast
spiral-.T scanning is effecti+e in detecting the primar! tumour in onl! 01F of casesA
radiolabelled octreotide or endoscopic ultrasound scanning ma! often be re7uired. Although
surgical cure is rarel! possible due to the presence of metastases* hepatic embolisation ma! be
helpful for s!mptom control .
68 . A 52-year-old woman diagnosed with type-2 diabetes mellitus and losing
weight is referred for an opinion, her H8 is thinking about insulin therapy. A
normochromic normocytic anaemia is noted. 7n e<amination she has angular
stomatitis and a well-demarcated erythematous rash in her groin which
e<tends to her lower limbs buttocks and perineum .
9hat is the ne<t step in her management ?
Refer to nursing colleagues for con+ersion to insulin
Tr! high-dose sulphon!lurea therap!
:bser+e and see again in ? months
Measure plasma glucagon le+els
.orrect answer
Measure plasma somatostatin le+els
Your answer
There is a suggestion that this patient has a glucagonoma* M1F of which are associated with
the characteristic s,in rash D necrol!tic migrator! er!thema. The annual incidence of
glucagonoma is estimated at # in '1 million. :+er =1F of glucagonomas are malignant* but*
because of their indolent presentation* the diagnosis is often o+erloo,ed for man! !ears. 3ue to
insulin antagonism o+er M1F are associated with impaired glucose tolerance* and e+entuall!
weight loss due to protein catabolism. ;urgical cure rate is as low as 0F* due to metastases
man! !ears prior to presentation. :ctreotide impro+es the s,in rash but ma! ha+e a detrimental
effect on glucose controlA combination chemotherap! with streptazocin and 0-fluoruracil (0-48)
gi+es good palliati+e results* as does hepatic embolisation .
69 . A 52-year-old woman is referred for opinion she has been diagnosed with
type-2 diabetes mellitus and is losing weight her H8 is thinking about insulin
therapy. A normochromic normocytic anaemia is noted. 7n e<amination she
has angular stomatitis and a well-demarcated erythematous rash in her groin
e<tending to her lower limbs buttocks and perineum .
9hat is the underlying diagnosis ?
5oorl! controlled t!pe-' diabetes
Latent t!pe-# diabetes
;omatostatinoma
.ushingBs disease
Hlucagonoma
Your answer
The s,in rash* necrol!tic migrator! er!thema* glucose intolerance* a normochromic* normoc!tic
anaemia* weight loss and angular stomatitis are all highl! suggesti+e of glucagonoma. The
annual incidence of glucagonoma is estimated at #&'1 million. :+er =1F of glucagonomas are
malignant* but because of their indolent presentation* the diagnosis is often o+erloo,ed for
man! !ears. 3ue to insulin antagonism o+er M1F are associated with impaired glucose
tolerance* and e+entuall! weight loss due to protein catabolism. ;urgical cure rate is as low as
0F* as metastases occur man! !ears prior to presentation. :ctreotide impro+es the s,in rash
but ma! ha+e a detrimental effect on glucose controlA combination chemotherap! with
streptazocin and 0-fluoruracil (0-48) gi+es good palliati+e results* as does hepatic embolisation .
70 . A *A-year-old man is referred after his first myocardial infarction for your
opinion. 5e has a total cholesterol le&el of G.5 mmol+l with normal
triglycerides. Jou note the presence of corneal arcus and tendon <anthomas .
9hich of the following genetic factors is not likely to be a cause of the
hypercholesterolaemia ?
An ecess of L3L receptors
Your answer
>ot producing an! L3L receptors
4ailure of the L3L receptors to mo+e to the cell surface
Abnormal receptor-binding to L3L
Inabilit! to internalise L3L for metabolism
4amilial h!percholesterolaemia is associated with an autosomal-dominant mutation on the short
arm of chromosome #M* thereb! reducing the number of high-affinit!* low-densit! lipoprotein
(L3L) receptors b! up to 01F. :+er $11 mutations ha+e been described so far and the! ma! be
a combination of an! of options BD/. 5atients present with premature coronar! heart disease
(.23) and ha+e a standardised mortalit! ratio of up to nine times that of age- and se-matched
controls. 2omoz!gotes present with .23 in childhood* heteroz!gotes present with .23 at the
age of $1O !ears .
71 . A *1-year-old man presents with acute pancreatitis. 4his is the third occasion
he has presented in the last 5 years. Jou follow him up in clinic and find he
has a markedly ele&ated triglyceride fraction and hypercholesterolaemia but
with 5D2 and 2D2 cholesterol within the normal range .
9hich of the following statements best fits his abnormal lipid picture ?
8suall! impro+ed b! thiazide diuretics
8suall! impro+ed b! glucocorticoids
8suall! impro+ed b! alcohol
2as an autosomal-recessi+e mode of inheritance
Your answer
Affects up to # in $11 people
.orrect answer
This is familial h!pertrigl!ceridaemia* an autosomal-dominant condition affecting # in $11
people* associated with raised +er!-low-densit! lipoproteins (@L3L) and trigl!ceride le+els. It
ma! be eacerbated b! alcohol* glucocorticoids and thiazide diuretics. A+oidance of alcohol is
ad+ised. .linical features include erupti+e anthomas and lipaemia retinalis .
72 . A A1-year-old non-smoking woman with no pre&ious cardiac history presents
from her H8 at the cardiac risk-factor clinic. 5er total cholesterol is G.0
mmol+l. She is o&erweight and has sleep apnoea. 7n e<amination you notice
her skin is particularly dry and there appears to be some e&idence of hair loss .
9hat is the ne<t step in her management ?
;creening of famil! members for h!percholesterolaemia
Lifest!le ad+ice and reassurance
;tart high-dose statin therap!
.hec, her T;2
Your answer
.hec, her fasting blood glucose
The suspicion with the histor! of dr! s,in* hair loss* obesit! and sleep apnoea is that she has
h!poth!roidism. This would be the cause of her secondar! h!percholesterolaemia. 4ran,
h!poth!roidism is said to occur in (F of patients with d!slipidaemias* and a further #1F of
patients with d!slipidaemia ha+e a raised T;2 le+el* but with a free T( still within the normal
range. Total cholesterol often impro+es somewhat with th!roine therap!* but statins ma! be
re7uired in addition .
73 . Jou re&iew a A7-year-old man in the lipid clinic. 5e is taking pra&astatin -0
mg+day.9hich of the following drugs should be used with caution ?
9arfarin
/r!throm!cin
.orrect answer
3igoin
Antacids
Antifungals
Your answer
9arfarin and digoin ha+e well-recognised interactions with ator+astatin and sim+astatin.
Antacid and antifungal agents are ,nown to interact with ator+astatin. /r!throm!cin has ,nown
interactions with sim+astatin* flu+astatin* ator+astatin and pra+astatin. All statins interact with
ciclosporin and nicotinic acid and should be used with caution in patients on fibrate therap! as
the ris, of rhabdom!ol!sis ma! be increased .
74 . 4he presence of galactorrhoea is )7S4 suggesti&e of which one of the
following conditions ?
TurnerBs s!ndrome
5ol!c!stic o+ar! disease
2!poth!roidism
Your answer
;heehanBs s!ndrome
Bromocriptine therap!
Halactorrhoea is nonpuerperial epression of mil,. 2!perprolactinaemia causes galactorrhoea
and amenorrhoea* which ma! result from a prolactin secreting pituitar! tumour or h!perplasia.
2!poth!roidism is a cause of raised prolactin le+els. In acromegal! one-third of patients ha+e
mild ele+ation of prolactin le+els resulting in galactorrhoea* amenorrhoea and decreased libido.
;imilar ele+ation of prolactin le+els occurs in a small percentage of patients with primar!
h!poth!roidism. ;heehanBs s!ndrome is a primar! h!popituitarism due to ischaemic necrosis of
the pituitar! gland caused b! postpartum haemorrhageA it is characterised b! failure of
postpartum lactation and failure to resume normal c!clic menstruation. TurnerBs s!ndrome
(o+arian d!sgenesis) causes primar! amenorrhoea with poor de+elopment of the breast.
Bromocriptine is a dopaminergic agent that has an inhibitor! effect on prolactin and is
fre7uentl! used to treat h!perprolactinaemia. Halactorrhoea and raised prolactin le+els ma! also
be seen in 5.:; .
75 . 9hich one of the following proteins is most likely to be associated with &ery
high le&els of plasma chylomicrons ?
Apoprotein /
Your answer
Apoprotein .II
.orrect answer
Apoprotein AII
Lipoprotein B
L3L receptor
3ietar! trigl!cerides in cholesterol are pac,aged b! gastrointestinal epithelial cells into large
lipoprotein particles called ch!lomicrons. After secretion into the intestinal l!mph and passage
into the general circulation* ch!lomicrons bind to the enz!me lipoprotein lipase* which is located
on endothelial surfaces. This enz!me is acti+ated b! a protein contained in the ch!lomicron*
apoprotein .II* liberating free fatt! acids and monogl!cerides* which then pass through the
endothelial cells and enter adipoc!te or muscle cells. Therefore* complete inacti+ation of either
lipoprotein lipase or apoprotein .II as a result of the inheritance of two defecti+e copies of the
rele+ant gene results in an accumulation of ch!lomicrons (t!pe I lipoprotein ele+ation) owing to
failure of con+ersion to the ch!lomicron remnant particle. 5atients with familial lipoprotein lipase
deficienc! usuall! present in infanc! with recurrent attac,s of abdominal pain caused b!
pancreatitis. The! also ha+e erupti+e anthomas resulting from trigl!ceride deposition.
Treatment should consist of a low-fat diet that ma! be supplemented b! medium-chain
trigl!cerides* which are not incorporated into ch!lomicrons. The absence of functional
apoprotein .II* with conse7uent failure to acti+ate lipoprotein lipase* presents with a similar
phenot!pe* although the affected patients are t!picall! detected at a somewhat later age than
are patients with familial lipoprotein lipase deficienc! .
76 . A A5-year-old woman known to ha&e chronic low back pain notices se&ere
sharp pain in the left groin after a minor fall and is unable to walk. 2eft neck of
femur fracture is identified on radiological e<amination. Ioutine laboratory
e&aluation discloses a serum calcium concentration of 1.G mmol+l a serum
phosphorus concentration of 0.A= mmol+l and increased serum alkaline
phosphatase acti&ity. 4he serum parathyroid hormone le&el was subseBuently
found to be ele&ated.4he most likely diagnosis is ?
5rimar! h!perparath!roidism
2!per+itaminosis 3
5agetBs disease of bone
:steoporosis
Your answer
@itamin 3 deficienc!
.orrect answer
The combination of h!pocalcaemia and h!pophosphataemia points to the diagnosis of
osteomalacia and +itamin 3 deficienc!. 3ietar! deficienc! and malabsorption are common
causes of +itamin 3 deficienc!. 5rimar! h!perparath!roidism and h!per+itaminosis 3 are
associated with h!percalcaemia rather than h!pocalcaemia. 5agetBs disease is associated with
increased ris, of fracture and increased serum al,aline phosphatase acti+it!* but the serum
calcium is usuall! within normal limits. :steoporosis is the most common cause of fracture of
nec, of femur and is not associated with an! specific abnormalit! in the standard bone
biochemistr! profile .
77 . %n glucagonoma the )7S4 likely associated skin lesion is ?
/r!thema chronicum migrans
Acanthosis nigricans
5anniculitis
Ichth!osis
>ecrol!tic migrator! er!thema
Your answer
Hlucagonoma s!ndrome (diabetes mellitus* weight loss and anaemia) is associated with a
characteristic s,in rash (necrol!tic migrator! er!thema) in =0F of cases. The lesion starts as an
indurated er!thema at the perineum* face and nose. 9ithin a few da!s blisters will co+er the
surface of the s,in which then crust and heal lea+ing h!perpigmented s,in. This process ta,es
=D#( da!s with lesions de+eloping in one area while others are resol+ing .
78 . A 2*-year-old woman presents to her H8 after the birth of her second child.
She complains of e<treme tiredness and a persistent hoarse &oice that she is
ha&ing problems shaking off. Despite breast-feeding her child she is failing to
lose her pregnancy weight. 4hyroid autoantibodies are negati&e. 5er 4S5 is 12
m!+l with a free 4-of 5 pmol+l. 4here is no thyroid tenderness on
e<amination. 5er H8 notes that her pulse is only 52 beats per minute .
9hat diagnosis fits best with this clinical picture ?
2ashimotoBs th!roiditis
5ostpartum th!roiditis
Your answer
Atrophic h!poth!roidism
Iodine deficienc!
2!perth!roidism
This woman is h!poth!roid after the birth of her second child. 2er th!roid autoantibodies are
negati+e and she has no signs of autoimmune disease* which might suggest atrophic
h!poth!roidism were the! present (eg +itiligo). Iodine deficienc! is now rare and tends to occur
in isolated mountain areas. 5ostpartum th!roiditis is usuall! transient* and ma! in+ol+e
h!perth!roidism* h!poth!roidism or the two in se7uence. It is thought to be due to changes in
the immune s!stem after pregnanc! and is histologicall! a l!mphoc!tic th!roiditis. It is usuall!
self-limiting .
79 . A 57-year-old woman presents with a feeling of shortness of breath and
choking on lying down. Some - months earlier she had been diagnosed with
atrial fibrillation and was started on aspirin and digo<in by her H8. 7n
e<amination her H8 could feel goitre. 8lain radiography confirmed retrosternal
e<tension which was presumed to be contributing to her shortness of breath.
5er 4S5 le&el was less than 0.05 m!+l. 4hyroid autoantibodies were negati&e .
9hat diagnosis best fits with this clinical picture ?
2ashimotoBs th!roiditis
Large* toic* multinodular goitre
Your answer
Th!roglossal c!st
Th!roid carcinoma
Hra+eBs disease
Toic multinodular goitre usuall! occurs in women o+er 00 !ears of age and is more common
than Hra+eBs disease in the elderl!. 2er atrial fibrillation ma! well be related to this. 2er goitre
is obstructing* with significant retrosternal etension* and surger! is the treatment of choice.
Initiall!* howe+er* she should be rendered euth!roid with antith!roid drugs such as carbimazole .
80 . A A2-year-old man presents for re&iew some * months after first being
diagnosed with type-2 diabetes. 5is ()% is *0. Despite ha&ing lost about 7 kg
in weight his morning blood sugars are still around G mmol+l, an 5b A16 check
was =.GP. 5e is hypertensi&e and taking ramipril his triglycerides are raised
and his 5D2 cholesterol is low .
9hich therapy for his diabetes would be the best initial choice for his hyperglycaemia ?
Hlibenclamide
Hliclazide
Rosiglitazone
Metformin
Your answer
Acarbose
The best initial therap! for this man* who clearl! has the metabolic s!ndrome* is metformin.
This drug should be introduced at a dose of 011 mg per da!* with a gradual increase in the dose
o+er a few wee,s to around #.0D' g total dail! dose (di+ided into morning and e+ening doses .(
The 8653; (8nited 6ingdom 5rospecti+e 3iabetes ;tud!) showed that for macro+ascular ris,*
metformin was superior to sulphon!lureas or insulin (a statisticall! significant ris, reduction for
m!ocardial infarction compared to con+entional therap!* which was not found in the
sulphon!lurea or insulin group). Metformin is a partial insulin sensitiser that wor,s to reduce
hepatic glucose output and also has some anti-inflammator! action* showing positi+e effects on
plasminogen-acti+ator inhibitor-# (5AI-#) in particular. 5AI-# is associated with an increased
tendenc! to blood clotting and ma! be associated with an increased +ascular ris, in sufferers of
the metabolic s!ndrome .
81 . A 57-year-old man with a ()% of *0 and a history of hypertension
dyslipidaemia and type-2 diabetes presents for re&iew. 5e has tolerated 2
g+day of metformin well but 2 years after the initial diagnosis of diabetes his
blood sugars are still too high in the morning on occasions and a recent 5(
A16 was 7.5P. 5is Cob entails occasional shift work during which he is unable
to eat for long periods .
9hich would be the most appropriate add-in therapy to his metformin treatment ?
Acarbose
Hlargine
5ioglitazone
.orrect answer
.hlorpropamide
Hlibenclamide
Your answer
The choice is between the addition of sulphon!lurea therap!* insulin or a glitazone.
;ulphon!lurea therap! or insulin ma! not be appropriate due to his shift wor, and ris, of
h!pogl!caemia. In addition* he has features of the insulin resistance (metabolic) s!ndrome* and
so a glitazone ma! be the more logical addition .
Rosiglitazone and pioglitazone wor, b! increasing glucose upta,e into s,eletal muscle and fat.
The! bind to the 55AR-gamma nuclear receptor (55AR* peroisome proliferator-acti+ated
receptor)* which promotes the transcription of a number of enz!mes concerned with glucose
and lipid metabolism. .linical trials indicate that b! targeting insulin resistance the! appear to
ha+e positi+e effects on other features of the metabolic s!ndrome such as blood pressure and
23L cholesterol. There is some debate around the cardio+ascular effects of rosiglitazone .
82 . A 5A-year-old man with type-2 diabetes presents with background diabetic
retinopathy. 5is 5( A16 has been consistently abo&e GP for the past 5 years .
9hich of the following factors would most worsen prognosis for his retinopathy ?
Rapid impro+ement in blood glucose le+els
Your answer
Total cholesterol 0.' mmol&l
Long-term impro+ements in blood glucose control
Trigl!ceride le+els of '.# mmol&l
;topping smo,ing
Rapid impro+ement in blood glucose le+els ma! be associated with worsening of diabetic e!e
disease. Both the 3iabetes .ontrol and .omplications Trial (3..T) in t!pe-# diabetes and the
8653; (8nited 6ingdom 5rospecti+e 3iabetes ;tud!) in t!pe-' diabetes ha+e demonstrated the
long-term benefits of reducing blood sugar le+els in controlling future micro+ascular
complications. All diabetic patients should be offered !earl! e!e screening* preferabl! with a
digital retinal camera. Rapidl! deteriorating +isual acuit!* hard eudates encroaching on the
macula* preproliferati+e changes or new-+essel formation are all reasons for earl! referral to an
ophthalmologist .
3iabetic retinopath! has been ,nown for man! !ears to be the leading cause of blindness in the
86 in people of wor,ing age* but progression to blindness is now slowing and tailing off due to
impro+ed ophthalmic care. ;mo,ing has no effect on the prognosis of retinopath! .
83 . A -5-year-old man is to undergo trans-sphenoidal surgery for resection of a
growth hormone-secreting pituitary adenoma. 5e asks about the prospect of a
cure and you e<plain that he will reBuire biochemical monitoring o&er the ne<t
few months and years to assess this .
9hich of the following biochemical tests is the best way to monitor for recurrence ?
Blood glucose
;erum cortisol
IH4-# or growth hormone le+el
Your answer
5rolactin
Th!roid function testing
The blood glucose le+el ma! be a mar,er of recurrence +ia the de+elopment of diabetes
mellitus* but in itself is not the best wa! to monitor. The aim of therap! is to ,eep the growth
hormone le+el below 0 m8&l or insulin-li,e growth factor-# (IH4-#) le+els within normal limits.
Hrowth hormone le+els abo+e 0 m8&l are associated with a worse prognosis. Nuoted failure
rates for surger! are around #$.$F for tumours of < # cm or ##.#F for tumours " # cm
confined to the sella .
Treatment failure ma! be managed with somatostatin analogues or eternal-beam radiotherap!.
.hronic somatostatin analogue use ma! be associated with increased gallstones* and the
response to eternal-beam radiotherap! is often slow* so neither treatment is perfect .
84 . A -2-year-old man presents to his H8 complaining of decreased libido. 5e has
also noticed ha&ing to sha&e less freBuently. 7n the few occasions he has tried
to ha&e se< he has failed to maintain his erection to penetration. 5is &isual
field testing is normal and he has no medication history. Serum prolactin
le&els are =G0 mg+l growth hormone and thyroid function le&els are normal.
9hat diagnosis fits best with this clinical picture ?
Microprolactinoma
Your answer
Macroprolactinoma
5s!chogenic impotence
2!poth!roidism
Acromegal!
Men with prolactinoma tend to present substantiall! later than women* the latter presenting
with cessation of periods and galactorrhoea. Macroprolactinoma is unli,el! gi+en his normal
+isual fields. 5rolactinoma is the most common pituitar! tumourA microadenomas are more
common in women* whereas macroadenomas appear more commonl! in men. Treatment is
surgical (+ia the trans-sphenoidal route) or medical (microprolactinomas) using dopamine
agonists such as bromocriptine or cabergoline to suppress prolactin release. Trans-sphenoidal
cure rates are around 01D=0F. 8p to '1F of microprolactinomas resol+e during long-term
dopamine agonist treatment .
85 . A 1G-year-old student is brought to A:; by his flatmates. 5e had been playing
sBuash that afternoon and while resting #after ha&ing had a pasta meal$
complained of generalised weakness. 5e was unable to stand and had to be
carried in by his friends. 5is potassium le&el was noted to be 2.A mmol+l.
!rine screen for diabetic and la<ati&e abuse was normal. Apparently he has
had similar attacks since his early teenage years. Symptoms were aborted by
potassium chloride .
9hat type of mutation best fits the underlying pathology of this autosomal-dominant
condition in&ol&ing intermittent paralysis ?
Mutation of a muscle +oltage-gated sodium channel
Mutation in a muscle +oltage-gated potassium channel
Your answer
Mutation in a renal potassium channel
Mutation in a renal sodium channel
Mutation in a muscle +oltage-gated calcium channel
.orrect answer
2!po,alaemic periodic paral!sis is related to a muscle calcium-channel mutation (.A.l>#A$). It
is an autosomal-dominant conditionA attac,s usuall! begin in the teenage !ears and ma! remit
after around $0 !ears of age. 5otassium is normall! below $ mmol&l during an attac, and
s!mptoms resol+e with the administration of potassium chloride. Attac,s appear to be
precipitated either b! a high carboh!drate meal or b! a period of rest after etreme eercise.
Loss of function of muscles concerning speech* bulbar or generalised wea,ness ma! occur and
attac,s can last for se+eral hours .
86 . A 2*-year-old woman with type-1 diabetes presents with an unusual lesion on
her shin. %t began as a patch of spreading erythema but now looks yellow and
has begun to ulcerate .
9hat diagnosis best fits this lesion ?
Acanthosis nigricans
3ermatitis herpetiformis
Hranuloma annulare
Your answer
>ecrobiosis lipoidica
.orrect answer
Ianthoma
>ecrobiosis lipoidica occurs in patients with t!pe-# diabetes* beginning as a patch of er!thema
that spreads across the shin* begins to !ellow and then ma! ulcerate. Acanthosis nigricans is
associated with insulin resistance and ma! occur in those with t!pe-' diabetes and is usuall!
found in the aillas. Hranuloma annulare is normall! diffuse in diabetes and occurs as a ring of
papules. 3ermatitis herpetiformis is* of course* associated with coeliac disease (gluten
enteropath! .(
87 . A 71-year-old man with established sarcoidosis presents for re&iew. 5e suffers
from a degree of pulmonary fibrosis due to pre&ious pulmonary infiltration and
has been taking corticosteroids intermittently. Jou are asked for a
consultation as his serum calcium concentration is *.1 mmol+l. 5is renal
function is normal and parathyroid hormone is Cust below the lower end of the
normal range .
9hat is the most likely cause of his hypercalcaemia ?
2!perparath!roidism
2!poparath!roidism
Increased h!dro!lation of @itamin 3
Your answer
.hronic renal failure
Mil,-al,ali s!ndrome
In this case* parath!roid hormone is +irtuall! in the normal range and his renal function is
normal* so there is nothing to suggest a diagnosis of the mil,Dal,ali s!ndrome. 2!percalcaemia
is found in #1F of established cases of sarcoid* and ma! e+entuall! lead to nephrocalcinosis.
The cause is increased # a-h!dro!lation of +itamin 3 b! sarcoid macrophages* in addition to
that ta,ing place in the ,idne! .
88 . Jou are asked to re&iew a 5--year-old psychiatric patient by his H8. 4his
patient has been diagnosed with impaired glucose tolerance. 5e also has a
history of hypertension for which he takes ramipril .
9hich of the following drug classes is most well known as a cause of impaired
glucose tolerance ?
Thiazolidinediones
A./ inhibitors
At!pical antips!chotics
Your answer
Biguanides
;ulphon!lureas
:lder at!pical antips!chotic agents ha+e been implicated as a cause of impaired glucose
tolerance* and are thought to put those ta,ing them at increased ris, of t!pe-' diabetes .
Thiazides and P -bloc,ers ma! also impact negati+el! on the incidence of new diabetes* and this
combination of antih!pertensi+es is no longer a first choice for patients with a high t!pe-'
diabetes ris, .
Ramipril was shown in the 2:5/ (2ealthcare :ptions 5lan /ntitlement) stud! to reduce
progression to t!pe-' diabetesA this is thought to be mediated b! a reduction in insulin
resistance. The LI4/ (Longitudinal Inter+al 4ollow-up /+aluation) stud! using losartan has
demonstrated that angiotensin-receptor bloc,ers also probabl! reduce the incidence of new
cases of t!pe-' diabetes .
Thiazolidinediones and biguanides wor, b! primaril! reducing insulin resistance* sulphon!lureas
stimulate insulin releaseA all three are used as treatments for t!pe-' diabetes .
89 . Jou are asked by a H8 to re&iew a 1A-year-old girl who presents with primary
amenorrhoea. She appears on e<amination to ha&e minimal body hair but
normal breast de&elopment. ;<amination also re&eals a blind-ended &agina.
(iochemistry re&eals increased 25 normal 3S5 raised estradiol and raised
testosterone le&els .
9hich diagnosis fits best with this history and e<amination ?
5ol!c!stic o+arian s!ndrome
TurnerBs s!ndrome
AshermanBs s!ndrome
Testicular feminisation
Your answer
5regnanc!
This picture fits a diagnosis of testicular feminisation* in which androgen receptors are defecti+e.
This girl* although ha+ing the eternal phenot!pe of a female* will ha+e the IY ,ar!ot!pe. These
indi+iduals are* of course* infertile. 5atients with testicular feminisation do possess testes* and
orchidectom! is ad+ised since there is a high ris, of testicular malignanc! .
Hirls with TurnerBs s!ndrome usuall! ha+e the t!pical TurnerBs phenot!pe of short stature and
webbed nec,. AshermanBs s!ndrome is amenorrhoea due to uterine s!nechiae after infectionA
there is no histor! to suggest this and it can be ruled out on the eamination findings .
90 . A 27-year-old man presents &ia his H8 for re&iew. 5e wants to start a
relationship but is concerned about his small phallus. 5e also has difficulty
becoming aroused. 7n e<amination he is slim and there is gynaecomastia.
4here is a general paucity of body hair his penis is small and he has small
testes.9hich diagnosis fits best with this history and e<amination ?
Testicular feminisation
6linefelterBs s!ndrome
Your answer
.ongenital adrenal h!perplasia
True hermaphroditism
0S -reductase deficienc!
This man is li,el! to ha+e ,ar!ot!pe IIY and has the features of 6linefelterBs s!ndrome.
Indi+iduals with testicular feminisation are phenot!pic females. 6ar!ot!pic females with
congenital adrenal h!perplasia ma! ha+e clitoromegal! and labial fusion* as well as ambiguous
eternal genitalia that ma! appear maleA the! do not* howe+er* ha+e testes .
True hermaphrodites are +anishingl! rare and ma! be II&IY or mosaic in ,ar!ot!peA the!
possess both testes and o+aries and usuall! ha+e male or ambiguous eternal genitalia .
91 . A 5--year-old man presents to the diabetes clinic for re&iew. 5e has had type-
1 diabetes for *0 years. Iecently he has suffered a number of falls which he
describes as attacks where he feels MfaintF and loses his footing. 5e has
suffered from impotence for a number of years and takes antireflu<
medication. 7n e<amination he has a postural drop of *5 mm5g in his blood
pressure.9hich diagnosis fits best with this history and e<amination ?
3iabetic autonomic neuropath!
Your answer
Transient ischaemic attac,s
Arrh!thmia
;imple fainting
;omatisation disorder
This man has a mar,ed postural drop* features of HI tract neuropath! and impotence. 4urther
assessment of his cardio+ascular s!stem is li,el! to re+eal tach!cardia and impaired
cardio+ascular response to the @alsal+a manoeu+re. :ther s!mptoms of HI tract in+ol+ement
ma! include diarrhoea and intractable +omiting. Bladder in+ol+ement ma! result in atonic
bladder and problems with painless urinar! retention and recurrent urinar! tract infections .
92 . A -2-year-old woman presents with difficult-to-treat hypertension. She is
taking ramipril atenolol and bendroflua"ide yet her blood pressure as
measured in the clinic is still 150+100 mm5g. 5er serum potassium
concentration as measured in clinic was 2.G mmol+l. She has been weaned off
her antihypertensi&es for a period of -KA weeks, at this time her renin le&el
was noted to be suppressed and her aldosterone le&el was abo&e normal.
Abdominal ultrasound suggests enlargement of the right adrenal .
9hich of the following diagnoses is most likely to fit with this clinical picture ?
Idiopathic h!peraldosteronism
.onnBs adenoma
Your answer
Hlucocorticoid suppressible h!peraldosteronism
Aldosterone-producing carcinoma
.arcinoid s!ndrome
There is contro+ers! about the commonest cause of h!poreninaemic h!poaldosteronism*
pre+iousl! ?1-E1F were said to be related to an adenoma* but the pre+ailing +iew now is that
=0F ma! be related to adrenal h!perplasia. Hlucocorticoid suppressible h!peraldosteronism and
aldosterone-producing carcinoma ma,e up less than #F of cases each. 5rimar!
h!peraldosteronism is estimated to be responsible for #D'F of cases of h!pertension and is
generall! as!mptomatic. Rarel!* there ma! be muscle cramps associated with h!po,alaemia .
To estimate renin and aldosterone le+els accuratel! it ma! be necessar! to wean the patient off
antih!pertensi+e drugs for a period of a few wee,s* since most classes of antih!pertensi+es
ha+e effects on the reninDangiotensin ais. .T or MRI scanning ma! be used to confirm the
adenoma .
After remo+al of the adenoma* blood pressure is normal in =1F of patients at # !ear. At 0
!earsB postsurger!* 01F of patients remain normotensi+e .
93 . An 1=-year-old girl is referred by her H8 who is concerned that she may ha&e
an underlying endocrine problem. 4he girl had been taken to the H8 by her
mother. She is a good student and has Cust won a place at uni&ersity. She
weighs only *= kg #A stone$ and is 1.7= m #5ft 10 inches$ tall. She is
emaciated her skin is dry and she has e<cessi&e growth of lanugo hair. She
has been amenorrhoeic for G months. 5er cortisol le&el is ele&ated her free 4-
is normal. She has an anaemia and associated reduced white cell and platelet
count.9hich of the following diagnoses is most likely to fit with this clinical
picture ?
AddisonBs disease
2I@
:ccult carcinoma
2!poth!roidism
Anoreia ner+osa
Your answer
Anoreia ner+osa has a female to male preponderance of MC#. It is estimated that around 1.0D
#F of American women between #0 and $1 !ears of age ha+e anoreia* and the numbers are
li,el! to be similar in the 86. The aetiolog! of anoreia is un,nown* but it is li,el! to be an
interaction between en+ironmental and genetic factors* American studies report rates of seual
abuse as high as 01F in anoreic females. 3espite an association with depressi+e and other
ps!chiatric illness* demographic studies show no impairment of cogniti+e le+el nor educational
achie+ement in the group of patients with anoreia ner+osa as compared to the general
population .
Indeed such patients ma! fit the profile of being Vhigh achie+ersV* which is what is being hinted
at in this 7uestion .
T!pical laborator! tests include decreased 4;2* L2* oestrogens and #=-:2 steroids. 4ree T( and
T;2 le+els are usuall! normal. Anaemia with decreased white and platelet cell count ma! also
occur. There ma! be metabolic al,alosis* h!pocalcaemia* h!po,alaemia and h!pomagnesaemia .
Treatment ma! in+ol+e comple ps!chotherap! for a number of !ears* and referral to a
specialist in the field is recommended .
94 . A A0-year-old woman is sent by A:; for endocrine re&iew. During the past 1=
months she has suffered two 6olleFs fractures and a fractured neck of her left
femur. Iesults of thyroid function testing serum protein electrophoresis and
serum parathyroid hormone estimation are all normal. (one densitometry of
the lumbar spine and femoral neck on the non-replaced side re&eal a bone
density within the osteoporotic range.9hich of the following inter&entions
would be most appropriate for her ?
:bser+e and repeat the densitometr! in #' months
Initiate bisphosphonate therap!
.orrect answer
Initiate calcium and +itamin 3 therap!
Initiate 2RT
Your answer
Initiate calcium supplementation
This woman has idiopathic osteoporosis* with normal th!roid function and parath!roid hormone
assa! ruling out h!perth!roidism and h!perparath!roidism. 2er osteoporosis is clearl! of clinical
significance in that she has suffered two fractures of the distal radius and a fractured nec, of a
femur during the past #M months. 2ormone replacement therap! (2RT) would not be ad+ised in
this age group due to the increased cardio+ascular and breast cancer ris,. Bisphosphonate
therap!* which inhibits osteoclast acti+it!* would be the best choice in this patient .
95 . A 5=-year-old woman is taking alendronate for osteoporosis. She &isits the
clinic and is keen to discuss the mechanism of action of this class of drugs as
she has been studying them on the %nternet.9hich of the following options
best describes the mode of action of the bisphosphonate class of agents ?
Inhibits osteoblast acti+it!
;timulates osteoblast acti+it!
;timulates osteoclast acti+it!
Inhibits osteoclast acti+it!
Your answer
Increases the bioa+ailabilit! of +itamin 3
The primar! mode of action of the bisphosphonates is inhibition of osteoclast acti+it!. 9ith the
results from large-scale 2RT studies suggesting an increased ris, of carcinoma or cardio+ascular
disease* the! ha+e become the first-choice therap! for osteoporosis. The! ma! be gi+en orall!*
although some clinicians prefer intermittent intra+enous use. This class of drugs ma! be
associated with an increased ris, of oesophageal ulceration. 9hen the class was de+eloped
there was initial concern o+er prolonged inhibition of bone turno+er o+er a number of !ears* but
this appears to ha+e been unfounded .
96 . A A1-year-old farmer who has long-standing type-1 diabetes is brought to the
clinic by his wife. 5e has been limping for a while and his wife noticed that his
ankle was rather abnormally shaped after he stepped out of the shower.
;<amination of his right ankle re&eals a painless Coint that is warm and
swollen. 4here is crepitus and what appears to be palpable bone debris. ?-ray
re&eals gross Coint destruction and apparent dislocation. 5is 6I8 and white
count are normal the Coint aspiration fluid shows no microbes and historical
re&iew of 5( A1c re&eals that it has rarely been below GP.9hat is the most
likely diagnosis in this case ?
.harcotBs an,le
Your answer
:steom!elitis
:ld healed fracture
Rheumatoid arthritis
:steoarthritis
.harcotBs joint is said to occur in # in e+er! =01 patients with diabetes mellitus* although this is
increased to 0 in e+er! #11 in those with pro+en neuropath!. 2owe+er* a .harcotBs joint does
not just occur in those with diabetes but ma! occur in '1D(1F of patients with s!ringom!elia
and 0D#1F of those with long-standing s!philis (although this condition is now +er! rare).
Management in diabetes includes tight control of blood glucose* appropriate orthotic
inter+ention* and there is e+idence that bisphosphonates ma! slow the pace of joint destruction .
97 . A *2-year-old merchant banker presents for endocrine re&iew. Apart from a
past history of reflu< symptoms her pre&ious medical history is unremarkable.
She gi&es a history of increasing fatigue being sometimes unable to lea&e her
bedroom, she says that she is only able to walk a few steps without feeling
e<hausted. She has lost a little weight and gone on long-term sick lea&e from
her Cob. She feels unable to concentrate has headaches intermittent sore
throats and feels that when she is able to sleep she awakes unrefreshed. 3ull
blood count &iscosity urea and electrolytes li&er function testing thyroid
function testing and a Synacthen test arranged by her H8 ha&e all been
normal. 7n e<amination in the clinic there are no abnormal physical findings in
this normal weight normal height young woman.9hat diagnosis fits best with
this clinical picture ?
AddisonBs disease
2!poth!roidism
.hronic fatigue s!ndrome
Your answer
M!asthenia gra+is
:ccult malignant disease
The pre+alence of chronic fatigue s!ndrome is estimated at #D$ per #111 indi+iduals. The
predominant age at presentation is !oung adulthood to middle age* and there is a slight female
preponderance. ;ufferers often report a short flu-li,e illness at the point of onset and some
scientists belie+e that an initial +iral trigger ma! be responsible. .on+entional in+estigation and
ph!sical eamination pro+es normal. Initial hopes centred on the /psteinDBarr +irus infection
being that trigger* but concrete proof has not !et been demonstrated. /ducation* counselling
and goal setting ha+e pro+ed useful in stimulating reco+er!* and trials ha+e demonstrated
impro+ements with amitript!line use* but man! patients ta,e !ears to return to normal life .
98 . A A2-year-old man presents with e&idence of gynaecomastia. 5e has been
taking long-term digitalis and warfarin therapy for persistent atrial fibrillation.
%n addition he takes another tablet for heartburn. Iesults of tests for
androgens 56H li&er function and thyroid function are all normal .
9hat is the most likely cause of his gynaecomastia ?
9arfarin therap!
3igoin therap!
.orrect answer
4urosemide
Ranitidine
Your answer
;odium bicarbonate
3igitalis is one of a number of drugs that ma! be associated with g!naecomastia. :ther causes
include oestrogens* cannabis* diamorphine* spironolactone* cimetidine* c!proterone*
gonadotrophins and some c!totoics. 9hilst g!naecomastia is occasionall! seen with ranitidine*
this is much less common +ersus digoin or cimetidine. Th!rotoicosis* li+er disease* oestrogen-
producing tumours* 2.H-producing tumours* star+ation&refeeding and carcinoma of the breast
are also associated with g!naecomastia. ;urgical remo+al is occasionall! carried out in !ounger
men .
99 . A 21-year-old uni&ersity student presents for re&iew. She is distressed by the
fact that she is o&erweight and is ha&ing to sha&e or pluck e<cessi&e facial
hair. She also notices that she appears to ha&e more generalised body hair
than other women. 7n further Buestioning you elicit a history that she can
sometimes miss a couple of menses but pregnancy testing is consistently
negati&e. 5er mother apparently had similar problems in her youth took a
while to concei&e and now has type-2 diabetes. 4estosterone is Cust outside
the upper limit of normal range her 25' 3S5 ratio is increased and prolactin is
normal. 9hich diagnosis fits best with this womanFs clinical picture ?
5ol!c!stic o+arian s!ndrome (5.:; (
Your answer
2!perprolactinaemia
Androgen-secreting tumour
.ushingBs disease
T!pe-' diabetes
5.:; is said to occur in $F of adult women. ;!mptoms usuall! begin around the time of
menarche and the diagnosis is often made in adolescence. :besit! is present in around (1F of
sufferers and there is an increased association with t!pe-' diabetes due to insulin resistance.
Biochemical abnormalities characteristicall! include a mildl! raised testosterone le+el and an
increased L2C 4;2 ratio. Treatment includes weight loss or metformin therap!. ;urgical
inter+ention with wedge o+arian resection ma! reduce androgen secretion and s!mptoms. Man!
women just re7uire reassurance and effecti+e local treatment for hirsutism. Those who wish to
concei+e do so with increased fre7uenc! after a trial of metformin therap!* clomifene* or in
some cases gonadotrophins .
100 . A 21-year-old woman presents for re&iew. She is concerned on this occasion
because she has not had a period for 5 months. She is 1.7A m in height and
weighs only -*.7 kg #7 stone$. A pregnancy test is negati&e and thyroid
function testing is normal. 9hich diagnosis fits best with this womanFs clinical
picture ?
Honadotrophin deficienc!
9eight-related amenorrhoea
Your answer
2!perprolactinaemia
5rimar! o+arian failure
5ol!c!stic o+arian s!ndrome
A woman needs to maintain a minimum bod! weight for menstruation* and amenorrhoea ma!
e+en be seen at weights considered to be at the lower end of the normal range. The biochemical
picture ma! be indistinguishable from gonadotrophin deficienc!. Haining bod! weight to abo+e
the 01th centile for height normall! results in the restoration of menstruation* but if this cannot
be achie+ed then oestrogen replacement ma! be considered. Amenorrhoea is often seen in
ballet dancers who maintain a low weight and carr! out periods of etreme ph!sical eercise .
101 . A 75-year-old woman is admitted in an unconscious state. 5er daughter found her
on the floor. 7n e<amination in casualty she is found to ha&e a core temperature of **
>6 and also to be in left &entricular failure. 5er blood glucose le&el is 5.7 mmol+l
random cortisol is ele&ated. (y chance you also catch the twice weekly run of thyroid
function testing and her free 4- is -.- pmol+l. A 64 scan of her brain re&eals no focal
lesion and a cursory assessment re&eals no gross focal neurology. 9hich diagnosis
fits best with this womanFs clinical picture ?
2!pogl!caemia
Your answer
AddisonBs disease
5rofound h!poth!roidism
.orrect answer
Massi+e stro,e
Alcohol ecess
This woman has a greatl! reduced free T( concentration* is h!pothermic* unconscious and has
e+idence of associated heart failure. Mortalit! associated with this condition used to be as high
as 01F* but with modern intensi+e care management* sur+i+al has impro+ed. T$ is usuall! gi+en
+ia a nasogastric (>H) tube or i+ injection at the rate of '.0D0 % g e+er! E h* with con+ersion to
T( after the patient regains consciousness. There is a ris, of precipitating heart failure if larger
doses are gi+en in the initial period. :ther supporti+e measures include o!gen therap!*
h!drocortisone i+ therap! and glucose infusion* but none of these ha+e been effecti+el! pro+en
in a randomised* controlled trial setting .
102 . A A2-year-old woman undergoes subtotal thyroidectomy for goitre. Some 12-
h postsurgery she calls the duty doctor and is found to be suffering from pins
and needles in her hands and carpopedal spasm. An urgent corrected calcium
le&el is found to be 2.1 mmol+l. 9hat is the most likely cause of her
hypocalcaemia and what is the long-term prognosis ?
;he is li,el! to ha+e permanent h!poparath!roidism due to surger!
2er h!pocalcaemia is li,el! to be transient due to local trauma at the time of
surger!
Your
answer
2er h!pocalcaemia is li,el! to be related to an acute fall in th!roid hormone
concentration after th!roid gland remo+al and she will reco+er
2er h!pocalcaemia is probabl! due to a fall in calcitonin after th!roidectom!
2er h!pocalcaemia is probabl! due to coeistent +itamin 3 deficienc! and is
li,el! to respond to treatment with +itamin 3
8p to #1F of patients who undergo subtotal th!roidectom! suffer transient h!pocalcaemia.
Than,full!* it becomes permanent h!poparath!roidism in less than #F of patients. Management
includes calcium chloride infusion (slowl!) under /.H monitoring* which should lead to the rapid
resolution of s!mptoms. /pert th!roid surgeons ha+e now minimised the ris, of permanent
h!poparath!roidism .
103 . Jou re&iew a *2-year-old woman with relapsed Hra&eFs disease. 4S5 is less
than 0.05 m!+l with a free 4-of *2.5 pmol+l. She has se&ere bilateral thyroid
eye disease with marked orbital oedema and proptosis. Jou are considering
radiotherapy as she has failed drug treatment.9hich of the following
statements best fits the management of her thyroid eye disease ?
;!stemic steroids are of no +alue in managing the e!e disease
:rbital irradiation is commonl! used to treat th!roid e!e disease
.orrecti+e e!e muscle surger! should now be considered
Your answer
;he should not be gi+en meth!lcellulose drops as these ma! worsen
oedema
2er th!roid e!e disease ma! be worsened b! radioiodine treatment
.orrect
answer
/acerbation of e!e disease is more common after radioiodine treatment (#0F +s $F on drug
therap! alone). :nl! 0D#1F of these worsening cases threaten sight* but discomfort and
deteriorating cosmetic appearance ma! cause much patient aniet!. ;!stemic steroids and oral
non-steroidal anti-inflammator! drugs (>;AI3s) ma! ease the discomfort and decrease
inflammation in se+ere s!mptoms. :rbital irradiation should be reser+ed for all but the most
se+ere of cases. .orrecti+e e!e muscle surger! should be dela!ed until th!roid e!e disease has
been stable for at least ? months* but ma! be of +alue in impro+ing diplopia .
104 . A *--year-old woman with a long history of type-1 diabetes
microalbuminuria and diabetic eye disease presents for re&iew. She has been
amenorrhoeic for 12 months and recent pregnancy tests ha&e been negati&e.
She has a healthy 1-year-old child but suffered a postpartum haemorrhage
Cust after his birth. 7&er the past few months she has also been increasingly
tired and has noticed thinning of her pubic and a<illary hair. 7n e<amination
she is slim and pale with small breasts and thin or largely absent pubic and
a<illary hair. A645 3S5 25 are all low with 4S5 Cust below the normal range.
)I% of the pituitary gland re&eals an empty sella.9hat diagnosis fits best with
this clinical picture ?
5rolactinoma
9eight-related pituitar! failure
5ol!glandular s!ndrome
;heehanBs s!ndrome
Your answer
Metastatic carcinoma
;heehanBs s!ndrome is well ,nown to occur in women due to postpartum haemorrhage and
h!po+olaemic shoc,* but the ris, of it occurring is increased in women with t!pe-# diabetes who
ha+e micro+ascular disease* and in patients with sic,le-cell anaemia. It is said to occur in # in
#1*111 deli+eries. Initial management includes immediate steroid therap! with later full
endocrine assessment and replacement of pituitar!-dependent hormones (eg th!roine) as
re7uired. 4urther conception ma! be difficult and re7uire pulsed gonadotrophin therap! to
restart o+ulation. 3iabetes insipidus is not usuall! associated with the s!ndrome .
105 . A 2*-year-old woman is admitted to the 6asualty department from her office
for the third time in the space of 2 months after ha&ing suffered a syncopal
attack. 7n Buestioning she admits to feeling &ery tired o&er the past few
months and being di""y on a number of occasions. 7n e<amination she looked
slim and tanned her blood pressure was 110+70 mm5g lying but dropped to
=5+A5 mm5g on standing. A645 is markedly raised free thyro<ine is below
the lower limit of normal and cortisol is low.9hich diagnosis fits best with the
clinical picture ?
2!poth!roidism
5rimar! h!poadrenalism
Your answer
5s!chiatric s!mptoms
2!po+olaemia
2I@
This woman has a mar,ed postural drop* increased pigmentation due to her high A.T2 and a
low cortisol* ma,ing primar! h!poadrenalism the most li,el! diagnosis. 4ree th!roine ma! also
be low at time of diagnosis* but th!roid hormone replacement should not be started as this ma!
worsen the adrenal crisis. Instead* steroid replacement therap! should be started* accompanied
b! fluid replacement* and th!roid function reassessed at a later stage. Autoimmune destruction
of the adrenal glands is responsible for E1F of cases. Tuberculosis accounts for a further #0F
of cases. There is a femaleC male predominance of 'C#* and a pre+alence of around 0 per
#11*111 .
106 . A 5--year-old publican is referred by his H8 for endocrine assessment. 5e is
obese with a ()% of *2 and has hypertension which is poorly controlled on
atenolol ramipril and bendroflua"ide. A recent fasting blood glucose test has
re&ealed type-2 diabetes. 7n e<amination he looks cushingoid with a blood
pressure of 150+G5 mm5g. Jou order a 2--h urinary free cortisol estimation
which turns out to be Cust abo&e the normal range. An o&ernight
de<amethasone suppression test is also unremarkable.9hich diagnosis fits
best with this clinical picture ?
.ushingBs disease
5seudo-.ushingBs
Your answer
;imple obesit!
/ssential h!pertension
5rimar! aldosteronism
:bese patients who consume alcohol to chronic ecess ma! ac7uire a cushingoid appearance. In
this case* this manBs occupation as a publican suggests that he ma! ha+e eas! access to
alcohol. 2is two screening tests for .ushingBs disease D the deamethasone suppression test
and '(-h urinar! free cortisol D are normal* which effecti+el! rules out option A. 2is t!pe-'
diabetes is li,el! to be related to obesit! and his sedentar! lifest!le* although diabetes mellitus
could* of course* also be due to alcohol-induced chronic pancreatitis .
Management in+ol+es lifest!le measures to promote weight loss* and strict control of his alcohol
inta,e. Metformin would be the ideal treatment for his diabetes* although this would be
contraindicated in the presence of continued alcohol ecess .
107 . Jou are asked to re&iew a *A-year-old man who has suffered a myocardial
infarction. 5e is a non-smoker with no past history of note and is not diabetic.
7n admission his total cholesterol was 10.2 mmol+l with triglycerides Cust
abo&e the normal range normal 5D2 and markedly raised 2D2 cholesterol. 5is
father died of a myocardial infarction at the age of -*.9hat is the most likely
cause of his raised cholesterol ?
2eteroz!gous familial h!percholesterolaemia
Your answer
4amilial combined h!perlipidaemia
4amilial h!pertrigl!ceridaemia
;econdar! h!perlipidaemia
Remnant h!perlipidaemia
2eteroz!gous familial h!percholesterolaemia is an autosomal-dominant monogenic disorder
present in # in 011 of the populationA the pre+alence is increased in 4rench-.anadians* 4inns
and ;outh Africans. 2omoz!gous 42 occurs in # in #1
?
patients and is associated with earl!
cardio+ascular death in childhood. .linical features ma! include tendon anthomas and
anthelasma. There is a genetic abnormalit! of the li+er L3L receptor. It is said that 01F of
men with this disorder will die b! the age of ?1 from cardio+ascular disease if untreated. 2igh-
dose statin therap! is the standard therap!* and specialist lipidologist ad+ice is recommended
for these patients .
108 . A 2=-year-old man presents with acute pancreatitis. 5e admits to
occasionally drinking wine but not to e<cess and there ha&e been no
symptoms to suggest gall-bladder disease. 5e suffered a left retinal &ein
thrombosis 2 years ago. 4riglyceride concentration was estimated at 10
mmol+l with normal 5D2 and 2D2 le&els. 9hat is the most likely cause of his
clinical presentation ?
;econdar! h!perlipidaemia
4amilial h!percholesterolaemia
4amilial h!pertrigl!ceridaemia
Your answer
2!polipidaemia
Abetalipoproteinaemia
Trigl!ceride concentrations abo+e ? mmol&l carr! a significant ris, of complications* and this
unfortunate man has suffered both a retinal +ein thrombosis and acute pancreatitis. Inherited
lipoprotein lipase or apoprotein .-II deficienc! ma! be responsible* but these defects are rare
and normall! present in childhood with erupti+e anthomas* lipaemia retinalis* pancreatitis and
retinal +ein thrombosis. ;o-called Jfamilial h!pertrigl!ceridaemiaB presenting in this wa!* with
complications in adulthood and no specific identified genetic defect* is more common. Treatment
in this case would in+ol+e the a+oidance of alcohol and use of a fibrate-t!pe agent .
109 . A 2--year-old man with learning difficulties presents for re&iew. 5e
complains of a sudden deterioration of &ision in his left eye. 5is past history of
note includes a deep &ein thrombosis. 7n e<amination he appears tall and slim
and almost marfanoid in appearance. 5e has a markedly ele&ated urinary
homocysteine. 9hich en"yme defect is most likely to be responsible for this
clinical picture ?
Meth!lene tetrah!drofolate reductase
Your answer
2istidase
2omogentisic acid oidase
Branch-chain ,etoacid deh!drogenase
.!stathionine s!nthetase
.orrect answer
This is the presentation of t!pe-# homoc!stinuria* where a defect in c!stathionine s!nthetase is
responsible. 5atients present with mild to moderate mental handicap* a marfan-li,e s!ndrome
and thrombotic episodes. The sudden +isual deterioration ma! be due to a thrombotic episode
or to the lens dislocation associated with this condition. 2omoc!stinuria ma! also associated
with defects in meth!lene tetrah!drofolate reductase (t!pe-' disease)* but sur+i+ors often ha+e
more se+ere mental retardation and rarel! sur+i+e the neonatal period .
110 . A 5--year-old man with long-standing type-2 diabetes presents for re&iew.
5e has a history of hypertension 20 cigarettes per day smoking habit, recent
5b A1c results ha&e a&eraged =.GP. 5e wore deck shoes without socks on a
recent holiday in Spain and his wife noticed a large ulcer o&er the big toe on
his left foot. 7n e<amination there is ob&ious loss of sensation probing of the
depth of the ulcer elicits no pain. 5is foot appears warm with a bounding
dorsalis pedis pulse and there is some toe clawing.9hat is the likely cause of
his ulcer ?
;imple trauma with no underl!ing patholog!
5eripheral +ascular disease
Local trauma combined with diabetic neuropath!
Your answer
@asculitis
;elf-neglect
;ome #1D#0F of patients with diabetes suffer foot ulceration at some stage during their li+esA
while 01F of all lower limb amputations are performed on patients with diabetes* effecti+e
super+ision ma! pre+ent a number of these from occurring .
This man has the features of neuropath!* although he certainl! has ris, factors for peripheral
+ascular disease. The neuropathic foot is said to be painless* or to ha+e abnormal neuropathic
pain* to be high arched with toe clawing. It is often warm with bounding pulses and ulceration
tends to occur on the plantar surface. 5eripheral ischaemia is said to be associated with rest
pain in a cold and nearl! pulseless foot. There is often gra+it!-dependent reddening of the foot*
which +anishes if the foot is ele+ated* ulceration tends to be painful and often presents in the
heal area. :f course* it is perfectl! possible for the two conditions to coeist* and for a mied
ischaemic&neuropathic patholog! to be the underl!ing cause .
111 . A AG-year-old woman presented &ia her H8 with episodes of facial flushing
and diarrhoea. An ultrasound scan re&ealed multiple hepatic lesions and a 2--
h urine collection re&ealed an ele&ated 5-5%AA. !nfortunately she did not
consent to follow-up and ne<t presented 2 years later. 5er son noticed a
gradual deterioration in her condition so that she appeared unable to cope at
home. 4here was apathy depression and the onset of mild confusion. 4he
diarrhoea is still present at re&iew and now she appears to ha&e
photosensiti&e dermatitis glossitis and angular stomatitis.9hat diagnosis fits
best with her clinical picture ?
5ellagra
Your answer
AlzheimerBs disease
.oeliac disease
Ribofla+in deficienc!
Thiamine deficienc!
In the presence of hepatic metastases* diarrhoea* facial flushing and a raised urinar! 0-2IAA (0-
h!dro!indoleacetic acid)* it is highl! li,el! that this woman has the carcinoid s!ndrome.
8nfortunatel! during the ' !ears that she has been lost to follow-up* it appears that she has
de+eloped pellagra .
5ellagra is caused b! a deficienc! of niacin. >iacin is manufactured +ia an enz!matic pathwa!
that in+ol+es the metabolism of tr!ptophan. Tr!ptophan is also the substrate amino acid used
b! carcinoid tumours to produce 0-h!dro!tr!ptamine. As the carcinoid tumour mass increases*
more and more of the a+ailable tr!ptophan is consumed* and less is a+ailable for niacin
production. /+entuall! patients ma! become niacin deficient* suffering the triad of dermatitis*
diarrhoea and dementia* the features of pellagra .
112 . A -5-year-old woman is referred to the endocrine clinic by her H8 for re&iew.
She has a body mass inde< of *5 hypertension and impaired glucose
tolerance. (y the time she &isits you in clinic she has succeeded in losing * kg
in weight. Jou decide to gi&e her a trial of orlistat and beha&ioural
therapy.9hich of the following best describes the mode of action of orlistat ?
:rlistat is a centrall! acting appetite suppressant
:rlistat is a pancreatic and gastric lipase inhibitor
Your answer
:rlistat reduces hepatic glucose production
:rlistat reduces insulin resistance
:rlistat is a b$-agonist
:rlistat* a pancreatic lipase inhibitor* bloc,s the brea,down and hence absorption of dietar! fat.
In essence* this means that ingested fat continues its passage through the gut. If patients
ta,ing orlistat do not maintain a low-fat diet then the! ma! suffer distressing oil! diarrhoea.
This is wh! it is essential to combine orlistat therap! with an effecti+e patient support
programme. ;ibutramine is a centrall! acting drug that acts on serotoninergic and
noradrenergic pathwa!s to encourage earlier satiet! and the ingestion of smaller meal portions .
113 . A -2-year-old man with long-standing 5%E infection presents for re&iew. 5e
has been taking antiretro&iral therapy for 5 years and has been relati&ely free
of associated disease. Jou notice on e<amination that he appears to ha&e lost
subcutaneous fat on his arms legs and face and has increased deposition of
fat around his abdomen. 5is lipids are also abnormal with a raised triglyceride
le&el and low 5D2 cholesterol.9hat is the most likely cause of this clinical
picture ?
Antiretro+iral-related lipod!stroph!
Your answer
2I@ wasting
2I@-associated malignanc!
Li,el! gastrointestinal patholog!
An inherited insulin-resistance s!ndrome
As the sur+i+al of 2I@-positi+e patients impro+es* more complications of antiretro+iral therap!
(AR@s) are becoming apparent. These include a lipod!stroph!-t!pe s!ndrome characterised b!
the loss of peripheral and facial subcutaneous fat* but increased abdominal and +isceral fat
deposition. There is also an increase in the size of the dorsocer+ical fat pad Jbuffalo-humpB. This
fat redistribution is associated with a picture of abnormalities usuall! associated with insulin
resistance* such as impaired glucose tolerance* low 23L cholesterol and high trigl!cerides.
Hlitazones ma! be of +alue in treating the condition* although some small trials ha+e pro+ed
e7ui+ocal .
114 . Jou are asked by the psychiatrists to re&iew a -2-year-old woman who has
long-standing bipolar disorder for which she takes lithium. During a recent
inpatient stay she appeared to be drinking &ast amounts of water and getting
up many times in the night to urinate. !rea and electrolyte testing re&eals
ele&ated sodium and urea concentrations suggesting possible mild
dehydration. 9ater depri&ation testing re&eals a progressi&ely rising serum
osmolality to abo&e *00 m7sm+kg without increased urine osmolality. (lood
glucose is normal.9hat is the most likely cause of this clinical picture ?
.ranial diabetes insipidus
Your answer
>ephrogenic diabetes insipidus
.orrect answer
5s!chogenic pol!dipsia
3iabetes mellitus
;!ndrome of inappropriate A32 secretion
This woman has nephrogenic diabetes insipidus (3I) secondar! to chronic lithium therap!. There
is usuall! some reco+er! after withdrawing lithium therap! but man! patients suffer some
degree of permanent nephrogenic 3I. Lithium therap! should onl! be discontinued with
ps!chiatric ad+ice* but at least now there are possible substitute medications including +alproate
and lamotrigine. :ther drugs that ma! cause 3I include demecloc!cline and glibenclamide .
115 . A *5-year-old woman is referred by her H8 because of recurrent headaches.
4hese tend to come on at times of stress or e<ercise and appear almost Min a
flashF. She also complains of intermittent palpitations and problems with
sweating. 5e has tried her on a course of antidepressants which only seemed
to make her symptoms worse. 5er 2--h urinary catecholamines are markedly
raised. An )I% scan re&eals a mass in the right adrenal medulla. 5er blood
pressure in clinic is 1-5+G5 mm5g .
9hat is the best management plan ?
8rgent P-bloc,ade
8rgent surger!
8rgent a-bloc,ade* then P-bloc,ade if re7uired* and
surger!
.orrect answer
:bser+ation
P-Bloc,ade followed b! urgent surger!
Your answer
This woman has a phaeochromoc!toma of the right adrenal medulla. The commonest presenting
features are headache (E1F)* palpitations (=1F)* h!perh!drosis (?1F) and h!pertension
(sustained in 00F and paro!smal in (0F of patients). 3rugs that inhibit catecholamine
reupta,e* such as tric!clic antidepressants and cocaine* can eacerbate or unmas, the
s!mptoms of this condition. MRI or MIBH (metaiodobenz!l guanidine) scanning are the
definiti+e methods of localisation. Management in+ol+es a-bloc,ade prior to surger! to a+oid a
h!pertensi+e crisis. The 0-!ear sur+i+al rate is M0F for patients with benign
phaeochromoc!toma* but this falls to (1F in those with malignant disease.
5haeochromoc!tomas are three times more li,el! to be malignant in women .
116 . A 25-year-old woman presents with a lump on the left-hand side of her neck
in the thyroid region. 4hyroid function is normal and uptake scanning re&eals
it to be a cold nodule. 3ine-needle aspiration biopsy re&eals architecture
suspicious of follicular carcinoma of the thyroid, this is confirmed on thyroid
lobectomy .
9hich of the following is the most appropriate management plan ?
:ral th!roine therap!
>o further surger!
Your answer
Total th!roidectom!
Total th!roidectom!* radioiodine therap! with oral th!roine replacement
therap!
.orrect
answer
Radioiodine therap!
The definiti+e management for follicular carcinoma of the th!roid without metastases is total
th!roidectom!* followed b! radioiodine therap!* with th!roine replacement to a T;2-
suppressi+e dose. If metastases are present the! usuall! respond to radioiodine therap!* and
this is added. Metastases are subject to haematological spread* with bone being the commonest
metastatic site. The 0-!ear sur+i+al rate approaches E1F for follicular carcinoma. This is much
higher than for anaplastic carcinoma of the th!roid* which usuall! occurs in the elderl! and onl!
carries a 0-!ear sur+i+al rate of 0F .
117 . A 5=-year-old woman presents with fe&er marked an<iety and agitation
palpitations marked muscle weakness and diarrhoea. 7n e<amination she has
goitre and is in fast atrial fibrillation with a &entricular rate of 1*5 beats per
minute. !rine testing re&eals e&idence of protein white cells and red blood
cells. 4hyroid function testing re&eals a 4S5 of L 0.05 m!+l.9hich of the
following diagnoses best fits this clinical picture ?
8rinar! tract infection
5aro!smal atrial fibrillation
2!poth!roidism
5haeochromoc!toma
Th!roid storm
Your answer
Th!roid storm is an unusual presentation of th!rotoicosis* precipitated b! acute stress in a
pre+iousl! undiagnosed patient. .auses include infection (as in this case)* m!ocardial infarction
or diabetic ,etoacidosis. Inade7uate adherence to therap! in an alread! diagnosed patient ma!
also be responsible. ;!mptoms include fe+er* aniet! and agitation* sweating* mar,ed proimal
muscle wea,ness* tach!cardias* diarrhoea and sometimes heart failure* particularl! in the
elderl! .
Acute general management includes carbimazole therap!* b -bloc,ade* digitalisation of atrial
fibrillation and occasionall! corticosteroids* which ma! inhibit th!roid hormone release. LugolBs
iodine* used in conjunction with carbimazole therap! ma! bring a more rapid resolution .
118 . A 5--year-old thyroto<ic man has been treated with radioiodine.9hat is the
best ad&ice for the patient during the post-radioiodine period ?
Antith!roid drugs should ne+er be ta,en after radioiodine treatment
2e should not ha+e close contact with children under the age of ## !ears for
about ' wee,s after treatment
Your
answer
There is no need to monitor his T;2 le+el
2e will ne+er need further doses of radioiodine
2e ma! ha+e close contact with children within $D( da!s
Antith!roid drugs ma! be recommenced after radioiodine administration* but treatment should
be withdrawn graduall! and guided b! ?DE wee,l! T4Ts. /arl! post-radioiodine h!poth!roidism
ma! be transient. The patientBs T;2 le+el should be monitored e+er! ? months after radioiodine
therap! to determine late h!poth!roidism. 5atients should not ha+e close contact with children
under the age of ## !ears for about ' wee,s after treatmentA this is to pre+ent eposing
children to radioacti+it!. 2owe+er* there is no o+erall ecess ris, of cancer .
119 . A A1-year-old patient with a history of recent thyroto<icosis underwent
maCor surgery a week ago. 5e now presents with altered mental status
tachycardia high-grade fe&er &omiting and cardiac failure. A diagnosis of
thyroid storm #crisis$ is made.9hat is the most important ne<t step in
management ?
Transfer the patient to IT8
Your answer
A+oid chlorpromazine in the treatment of agitation
A+oid carbimazole treatment
5otassium iodide should be gi+en immediatel! before prop!lthiouracil
5eritoneal dial!sis should be started immediatel!
An! acute stressful precipitating factor such as surger! can result in a th!roid storm or crisis.
This can also occur in patients with acute infections* postpartum and when antith!roid drugs are
being withdrawn. Th!roid crisis is associated with a significant mortalit! rate ($1D01F) and is
best managed in an intensi+e care unit where close attention can be paid to cardiorespirator!
status* fluid balance and cooling .
.hlorpromazine can be used to treat agitation and* because of its effect in inhibiting central
thermoregulation* it ma! be useful in treating the h!perp!reia. ;pecific treatment includes
prop!lthiouracil (5T8)* potassium iodide (after starting 5T8)* P-bloc,ers and glucocorticoids.
There is no clinical data comparing 5T8 and carbimazole in this situation. 5lasmapheresis and
peritoneal dial!sis ma! be effecti+e in cases resistant to pharmacological measures .
120 . A 25-year-old woman de&elops hyperthyroidism A weeks after deli&ery. 7n
e<amination she has painless firm enlargement of the thyroid gland .
9hich of the following statements is most correct with regards to the probable
diagnosis ?
The condition is more li,el! in those in whom th!roid peroidase (T5:)
antibodies were positi+e prior to deli+er!
.orrect
answer
9hen followed up* most patients ha+e lifelong h!poth!roidism
It is less common in patients with a histor! of T!pe-# diabetes
Around '1F of women ha+e some degree of th!roid d!sfunction post-
partum
Your
answer
If it is associated with intense fibrosis of the th!roid gland
5ostpartum th!roiditis is th!roid d!sfunction occurring within the first ? monthsB postpartum.
5re+alence ranges from 0 to =F. It de+elops in $1D0'F of women who ha+e positi+e T5:
antibodies. Most patients ha+e a complete remission but some ma! progress to permanent
h!poth!roidism. It is twice as common in patients with t!pe-# 3M .
121 . A 27-year-old woman on bromocriptine for microprolactinoma becomes
pregnant.9hat is the most appropriate management ad&ice ?
.ontinue bromocriptine at the same dose
Your answer
@isual field testing to be done e+er! fortnight
.ontinuing bromocriptine would significantl! increase congenital
abnormalities
;top bromocriptine as soon as pregnanc! is confirmed
.orrect
answer
>eeds cerebral magnetic resonance imaging (MRI) at least twice during
pregnanc!
9hilst bromocriptine is an effecti+e treatment for microprolactinoma in terms of suppression of
prolactin secretion* it ma! interfere with the natural rise in prolactin seen in pregnanc!.
Bromocriptine should therefore be stopped as soon as pregnanc! is confirmed. @isual field
testing and MRI are indicated onl! in the occasional patient who becomes s!mptomatic. 3ata on
children whose mothers recei+ed bromocriptine throughout pregnanc! indicate that the
incidence of congenital abnormalities is negligibleA hence the actual ris, of possible harm is low .
122 . A -1-year-old hea&y smoker presents with a serum sodium le&el of 112
mmol+l. A diagnosis of S%AD5 is confirmed.9hat is the most appropriate initial
management of his fluid balance ?
Immediate normal saline infusion
4luid restriction
Your answer
3esmopressin
Hlucocorticoids
5erform fluid depri+ation test
In patients with the s!ndrome of inappropriate A32 (;IA32) secretion* it is important to restrict
fluids to 011D#111 ml&'( hours. The underl!ing cause must also be detected and treated. If the
problem is not temporar! and long-term fluid restriction is li,el! to be difficult* then
demecloc!cline ma! be effecti+e b! inducing partial nephrogenic diabetes insipidus. In an
emergenc!* saline infusion ma! be re7uiredA howe+er* great care is re7uired as rapid
o+ercorrection of h!ponatraemia ma! cause central pontine m!elinol!sis .
123 . Jou are called to the psychiatric unit to see an 1=-year-old woman with
anore<ia ner&osa. 9hat would you e<pect to see in the results of her
biochemical in&estigations ?
Raised L2* 4;2
/le+ated circulating cortisol
.orrect answer
Low resting growth hormone le+els
Your answer
Increased HnR2
>ormal oestrogen le+els
The t!pical patient with anoreia ner+osa is a woman aged < '0 !ears with weight loss*
amenorrhea and beha+ioural changes. There is a long-term ris, of se+ere osteoporosis.
/ndocrine abnormalities include HnR2 deficienc!* low L2 and 4;2* low oestrogen in women*
raised circulating cortisol* low to normal th!roine* reduced T$* normal T;2 and increased
resting H2 le+els .
124 . A 72-year-old woman recently diagnosed as ha&ing hyperparathyroidism has
a serum calcium concentration of *.2 mmol+l.9hat is the most appropriate
treatment ?
;urger!
Your answer
Long-term obser+ation with regular blood tests
Long-term bisphosphonates
2ormone replacement therap!
3o further tests to see if there is end-organ damage
In primar! h!perparath!roidism* a patient with a mar,edl! ele+ated serum calcium " $ mmol&l
should be referred for surger!* unless this is contraindicated for other reasons. :ther indications
for surger! include impaired renal function* renal stones* nephrocalcinosis* reduced bone
mineral densit! (BM3)* substantiall! ele+ated urinar! calcium ecretion of "#1 mmol&'( hours.
Medical management is onl! indicated if the patient is unsuitable for surger! .
125 . 9hat is the commonest cause of death in patients with &on 5ippelK2indau
disease ?
.erebellar haemangioblastoma
Your answer
Renal carcinoma
.orrect answer
Retinal tumours
M!ocardial infarction
5haeochromoc!toma
@2L disease is an autosomal-dominant condition* with the @2L gene being located on
chromosome $. /stimated pre+alence is #&$M*111. Retinal angiomatosis is the initial
presentation in (1F patients. .erebellar haemangioblastoma is a common initial presentation.
Renal carcinoma is the commonest cause of death* with phaeochromoc!toma occurring in (1F
of patients with @2L .
126 . A *0-year-old man arri&es in A:; in an unconscious state. 5is initial blood
glucose reading by monitor is 2.1 mmol+l.9hat is the most important
immediate management of this patient ?
:ral glucose
Mannitol
50% glucose into a large +ein after first ta,ing a blood sample
Your answer
10 mg glucagon IM
.hec, his serum ethanol concentration
2!pogl!caemia is defined as a plasma glucose concentration < '.0 mmol&l and associated with
s!mptoms of neurogl!copenia. Large-bore +enous access should be achie+ed and blood samples
for serum glucose* li+er function* ethanol* cortisol* insulin* .-peptide* proinsulin and
sulphon!lurea le+els should be sent for testing. If the patientBs Hlasgow .oma ;core (H.;) is <
#$* then '0D01 ml 01F glucose should be gi+en intra+enousl!. Hlucagon (# mg im) ma! be
administered if no intra+enous access can be obtained .
127 . A 2=-year-old man presents with erectile dysfunction. 9hat is the
commonest cause in this age group ?
Alcohol
b-Bloc,ers
3iabetes mellitus
5s!chological factors
Your answer
Testicular tumour
/rectile d!sfunction is present in approimatel! #1F of all men and "01F of men o+er the age
of =1 !ears. .ommon causes are ps!chological factors ('1F)* drugs ('0F) and endocrine
causes. Baseline in+estigations include serum testosterone* prolactin* fasting glucose* L2 and
4;2* th!roid function tests* li+er and renal function* lipids and ferritin .
128 . 9hich of the following is the most likely long-term conseBuence of the
menopause ?
3ecreased thrombotic tendenc!
Less li,elihood of de+eloping ischaemic heart disease (I23 (
Increased possibilit! of de+eloping AlzheimerBs dementia
.orrect answer
Increased bone mineral densit! (BM3 (
Increased insulin sensiti+it!
Your answer
3uring the perimenopausal period there is an accelerated loss of BM3* rendering
postmenopausal women more susceptible to osteoporotic fractures. The! are also more li,el! to
de+elop I23* decreased insulin sensiti+it! and increased thrombotic tendenc!. 9omen are 'D$
times more li,el! to de+elop AlzheimerBs disease than men. It is suggested that oestrogen
deficienc! ma! pla! a role in the de+elopment of dementia .
129 . 9hich of the following in a A--year-old man with diabetes mellitus warrants
urgent ophthalmology referral ?
Bac,ground diabetic retinopath!
@itreous haemorrhage
Your answer
.ataracts
3rusen
>on-proliferati+e changes in the peripher!
Immediate referral is needed for patients with proliferati+e retinopath! (as it carries a (1F ris,
of blindness if untreated and laser treatment reduces this)* rubeosis iridis* +itreous
haemorrhage* ad+anced retinopath! with fibrous tissue* retinal detachment or maculopath! .
130 . %n which of the following coe<isting medical conditions would prescription of
the oral contracepti&e pill #768$ be acceptable ?
;e+ere migraine
2istor! of +enous thrombosis
Li+er disease
5ulmonar! h!pertension
Your answer
2!poth!roidism
.orrect answer
Absolute contraindications for :.5 includes a histor! of heart disease* pulmonar! h!pertension*
histor! of arterial or +enous thrombosis* histor! of cerebro+ascular disease* li+er disease* se+ere
migraine* breast or genital tract cancer and age o+er $0 !ears. 2!poth!roidism is not a
contraindication to ta,ing the :.5 .
131 . A 2=-year-old pregnant woman is detected to ha&e positi&e thyroid
antibodies and is euthyroid.9hich of the following potential conseBuences
should the obstetrician warn of during early pregnancy ?
2igher ris, of spontaneous abortions
.orrect answer
Increased ris, of neonatal h!poth!roidism
Your answer
Increased ris, of pregnanc!-induced h!pertension (5I2 (
Increased fetal size
Increased ris, of th!roid cancer in the mother
A pregnant mother with positi+e th!roid antibodies* but who is euth!roid* has a higher ris, of
spontaneous abortions. There is no ris, of neonatal h!poth!roidism and the ris, of 5I2 is not
increased. The occasional mother will de+elop h!poth!roidism at the end of pregnanc!* so the
T;2 le+el should be chec,ed between 'E and $' wee,sB gestation* then at $ monthsB
postpartum .
132 . A 2A-year-old woman presents with symptoms of flushing diarrhoea and
whee"ing. After e<amination a clinical diagnosis of carcinoid syndrome is
suspected.9hich of the following tests would be the most sensiti&e marker for
carcinoid syndrome ?
8rinar! 0-2IAA
Your answer
Al,aline phosphatase
/chocardiograph!
5lasma chromogranin A
.orrect answer
5lasma gut-hormone profile
The most sensiti+e mar,er for carcinoid s!ndrome is plasma chromogranin A* which has been
found in #11F of patients. Although the specificit! is lower than for 0-2IAA (0-
h!dro!indoleacetic acid)* most tumours with neuroendocrine differentiation are associated with
increased le+els of plasma chromogranin A. This test is not widel! a+ailable for clinical use.
8rinar! 0-2IAA has a sensiti+it! of about =1F and a specificit! of #11F in patients with
carcinoid s!ndrome. Al,aline phosphatase ma! remain normal despite li+er in+ol+ement and a
plasma gut-hormone profile ma! be useful .
133 . 9hat is the most appropriate in&estigation to confirm a diagnosis of
acromegaly biochemically after initial screening ?
Increased IH4-#
Your answer
Random growth hormone (H2) assa!
Insulin tolerance test
Th!roid function test
:ral glucose tolerance test (:HHT) with growth hormone
assa!
.orrect answer
In acromegal!* there is a failure to suppress H2 to < ' m8&l in response to a =0-g oral glucose
load. In contrast* the normal response is H2 suppression to undetectable le+els. Random H2 is
not useful in the diagnosis of acromegal! because* although normal subjects ha+e undetectable
growth hormone le+els throughout the da!* there are pulses of H2 which are impossible to
differentiate from the le+els seen in acromegal!. 4ollowing a TR2 (th!rotrophin-releasing
hormone) test* E1F of patients with acromegal! show increased le+els of H2. IH4-# is useful
for initial screening for acromegal!* as it pro+ides an integrated measure of growth hormone
le+els o+er time .
134 . An 1=-week pregnant woman presents to hospital with failure to gain weight
hyperemesis and persistent tachycardia. She is found to be thyroto<ic with a
suppressed 4S5 of 0.05 .
9hich of the following is the best management step ?
Radioiodine
Immediate surgical referral
Bloc,-and-replace regime
5rop!lthiouracil
Your answer
:bser+e and wait for normalisation
The aim of treatment of th!rotoicosis in pregnanc! is the alle+iation of s!mptoms and
normalisation of tests in the shortest time. Both prop!lthiouracil and carbimazole are effecti+e in
controlling disease in pregnanc!. The use of radioiodine is contraindicated in pregnanc!. ;urger!
is rarel! performed during pregnanc! and is reser+ed for patients who do not respond to
antith!roid drugs. A bloc,-and-replace regime should not be used as this will result in fetal
h!poth!roidism. This is a different scenario to that of h!peremesis and suppressed T;2* which
is seen in the first trimester and usuall! resol+es .
135 . A *--year-old woman presents with --month history of anore<ia and weight
loss e<cess pigmentation and di""iness on standing. %nitial in&estigations
re&eal hyponatraemia and hyperkalaemia.9hich of the following tests will be
most useful to confirm the diagnosis ?
/;R
Th!roid function tests
;erum urea
;hort ;!nacthen test
Your answer
;erum calcium
2!ponatraemia is present in M1F and h!per,alemia in ?0F of patients with primar! adrenal
insufficienc!. Although increased urea* increased /;R and raised T;2 le+els and mild
h!percalcaemia are seen in AddisonBs disease* these are not particularl! useful in confirming the
diagnosis. 2owe+er* failure to respond following a short ;!nacthen test suggests adrenal failure .
136 . A 72-year-old man with a long history of 678D was admitted with
pneumonia. 8rior to admission he had become increasingly confused. 7n
e<amination he was drowsy had a (8 of 1-2+75 mm5g and was clinically
eu&olaemic. 6hest auscultation was consistent with pneumonia. (lood
biochemistry in&estigations re&ealed /a 121 mmol+l 1 *.G mmol+l urea 2.-
mmol+l creatinine A- @mol+l and glucose -.2 mmol+l. 8lasma osmolarity was
2A1 m7smol+kg and thyroid function testing was normal. !rine testing
re&ealed an osmolality of 5A0 m7smol+kg and a sodium concentration of 55
mmol+l .
9hat was the most likely cause of his hyponatraemia ?
AddisonBs disease
Renal failure
.ardiac failure
.irrhosis
;!ndrome of inappropriate A32 secretion (;IA32 (
Your answer
This is a not uncommon conse7uence of pneumonia* particularl! in patients of this age group.
;mall-cell bronchial carcinoma is another cause of ;IA32. 3iagnosis is made on an
inappropriatel! high urine osmolalit! compared to plasma osmolalit!* urinar! sodium
concentration abo+e $1 mmol&l* eu+olaemia and normal th!roid and adrenal function. The
normal potassium le+el here ma,es AddisonBs disease unli,el!. A number of drugs can cause of
;IA32* including chlorpropamide* phenothiazines and carbamazepine. Both fluid restriction and
demecloc!cline precipitate renal diabetes insipidus* which ma! be of assistance in the treatment
of ;IA32 .
137 . A -5-year-old woman is due to undergo a hysterectomy. She has a pre&ious
history of hypertension headaches and panic attacks o&er the past few years.
She currently takes ramipril for her blood pressure. 4he preoperati&e
e<amination re&ealed a blood pressure of 150+=5 mm5g normal renal
function and calcium concentration an ;6H showed mild 2E5. !nfortunately
during the operation as the surgeon attempts to mobilise her uterus her
blood pressure rises to 210+110 mm5g her pulse to 1*0 bpm and she suffers
an acute myocardial infarction.9hat is the most likely cause of her
intraoperati&e hypertension and myocardial infarction ?
/ssential h!pertension
:ccult coronar! arter! disease
8ndiagnosed phaeochromoc!toma
Your answer
M/>-#
Renal arter! stenosis
Hi+en her histor!* she is li,el! to ha+e been suffering paro!sms of catecholamine release.
Mobilisation of the uterus is li,el! to ha+e precipitated an acute release of large amounts of
catecholamines* causing her catastrophic rise in blood pressure and intraoperati+e m!ocardial
infarction. ;tandard therap! for phaeochromoc!toma is aggressi+e S -bloc,ade prior to surger!*
then surgical ecision. The 0-!ear prognosis for benign tumours approaches M0F* but this falls
to around (1F in patients with malignant disease .
138 . A marfanoid-looking 21-year-old man is referred by his H8 for re&iew. 5e
has been suffering from acute headaches and panic attacks and is unable to
continue his course work at uni&ersity. 7n e<amination his blood pressure is
1-=+G= mm5g and you notice a number of neuromas around his lips. 5is 2--h
urinary catecholamine le&els are raised .
9hat diagnosis fits best with this clinical picture ?
Multiple endocrine neoplasia (M/>)-#
Multiple endocrine neoplasia (M/>)-
'a
>eurofibromatosis
Multiple endocrine neoplasia (M/>)-
'b
Your answer
.arcinoid s!ndrome
M/>-'b is characterised b! a marfanoid habitus* +isceral and intestinal ganglioneuromas (which
ma! occur around the lips and tongue)* adrenal tumours* and medullar! th!roid carcinoma*
parath!roid h!perplasia occurs much more rarel! than in M/>-'a. Adrenal tumours leading to
.ushingBs s!ndrome or phaeochromoc!tomas (=1F bilateral) ma! occur. ;creening is
recommended for M/>-'C total th!roidectom! in childhood is often recommended for those with
a ,nown gene defect. ;creening for medullar! th!roid carcinoma is +ia the pentagastrin test.
The best screen for phaeochromoc!toma is '(-h urinar! catecholamine estimation .
139 . A -5-year-old man presents for re&iew at the type-2 diabetes clinic. 5e is on
ma<imal metformin and his 5b A1c is still 7.=P. Jou elect to add in a
pero<isome proliferator-acti&ated receptor #88AI$-gamma agonist
pioglita"one.9hich of the following best describes the mode of action of 88AI-
Q agonists ?
The! act at the 55AR-Q receptor site* promoting binding as a heterodimer
with the retinoid I-receptor to 3>A
Your
answer
The! bind to a promoter region of 3>A as a heterodimer with the retinoid A-
receptor
The! act b! reducing peripheral insulin sensiti+it!
The! act b! stimulating insulin output
The! act +ia receptors at the cell surface
55AR-Q agonists act b! binding to the 55AR-gamma receptor* which binds to a promoter region
of 3>A in tandem with the retinoid I-receptor. This then leads to the upregulation of a number
of enz!mes concerned with lipid metabolism* bringing about a fall in free fatt! acids. The fall in
free fatt! acids promotes a reduction in hepatic insulin resistance* a rise in adiponectin and
impro+ed peripheral insulin sensiti+it!. Retinoid I-receptors are also now under in+estigation for
their possible action in impro+ing insulin sensiti+it!. .urrentl!* there are two 55AR-Q agonists
on the mar,et* rosiglitazone and pioglitazone* both with promising data for long-term gl!caemic
control in t!pe-' diabetes .
140 . A 52-year-old woman is referred by the A:; department after attending with
a 6ollesF fracture the second in the last * years. She underwent a total
hysterectomy at the age of *= for carcinoma. (one densitometry confirms
osteoporosis. 4he calcium and parathyroid hormone assays are normal.9hich
of the following is the most likely concerning her underlying pathophysiology ?
;he has suffered predominantl! cortical bone loss
;he has probabl! suffered an e7ual mi of cortical and trabecular bone loss
;he has probabl! suffered predominantl! trabecular bone loss
Your answer
8nderl!ing h!perparath!roidism will ha+e contributed
8nderl!ing h!poparath!roidism will ha+e contributed
2!po-oestrogenisation* as in this case* is usuall! characterised b! predominantl! trabecular
bone loss. In+olutional bone loss of old age is characterised b! a mied picture of trabecular and
cortical bone loss. In the presence of normal calcium and parath!roid hormone le+els*
parath!roid disease is somewhat unli,el!. 2ormone replacement therap! was formerl! the
mainsta! of treatment* but this has fallen out of fa+our due to increased thromboembolic
disease and the ris, of breast carcinoma. Bisphosphonates are now the fa+oured treatment
modalit! in this group of patients .
141 . A *2-year-old aromatherapist is referred to the endocrine clinic for re&iew.
She has been suffering intermittent tachycardias and panic attacks. 4here is
no significant past medical history. She takes a number of &itamin and mineral
supplements including kelp. 7n e<amination there are no eye signs and no
goitre. 4S5 is L 0.05 m!+l and thyroid antibodies are negati&e.9hat is the
most likely cause of her thyroto<icosis ?
Hra+eBs disease
Toic multinodular goitre
;olitar! toic nodule
/cess iodine ingestion
Your answer
/cess T;2 secretion
6elp is a +er! rich source of iodine. In patients with pre-eisting th!roid h!perplasia or
adenoma* the ingestion of large amounts of iodine ma! precipitate th!rotoicosis. Treatment is
withdrawal of the ,elp with monitoring of th!roid function. Th!rotoicosis affects 'F of women
and 1.'F of men in their lifetimes. Hra+eBs disease is responsible for around E1F of cases of
th!rotoicosisA toic multinodular goitre is the net commonest cause. 5atients with Hra+eBs
disease ma! be offered radioiodine* if suitable* or a period of medical therap! with antith!roid
drugs and th!roine replacement. 5atients who then fail on medical therap! are offered
radiotherap! or surger! .
142 . A 17-year-old student presents with intermittent weakness and feelings of
tiredness. 5er H8 reBuested some blood tests and found her to ha&e a
potassium le&el of 2.= mmol+l and bicarbonate of *2 mmol+l. She is
normotensi&e. Jou arrange a renin and aldosterone le&el and both are
ele&ated. !rinary calcium e<cretion is ele&ated. !rinary diuretic screen is
negati&e .
9hich diagnosis fits best with this clinical picture ?
3iuretic abuse
HitelmanBs s!ndrome
LiddleBs s!ndrome
BartterBs s!ndrome
Your answer
.onnBs s!ndrome
This s!ndrome is characterised b! increased urinar! calcium ecretion* h!po,alaemia* metabolic
al,alosis and raised renin and angiotensin le+els. Age at onset ma! +ar! depending on the
se+erit! of the causati+e mutation* whilst most patients present in childhood* some do present
later. This is not .onnBs s!ndrome because the patient is normotensi+e and both her renin and
angiotensin le+els are raised. BartterBs s!ndrome is due to a mutation in the >a
O
D6
O
D'.l
D
co-
transporter* the AT5-regulated potassium channel* or the ,idne!-specific basolateral chloride
channel. This impairs sodium and chloride reabsorption* and causes an increase in the
production of renal prostaglandin /'. 2!perplasia of the jutaglomerular apparatus is seen on
renal biops!. HitelmanBs s!ndrome is due to a different mutation* and produces a clinical picture
similar to that seen with chronic thiazide administration. LiddleBs s!ndrome is characterised b!
potassium wasting and h!pertension .
143 . A 2=-year-old man presents with hypertension that his H8 is finding difficult
to manage. 4here are a number of metabolic abnormalities and he is
concerned about the possibility of 6onnFs syndrome. 4he blood picture is one
of metabolic acidosis hyperkalaemia and low renin and aldosterone
le&els.9hat diagnosis fits with this clinical picture ?
HordonBs s!ndrome
Your answer
BartterBs s!ndrome
AddisonBs disease
.onnBs s!ndrome
HitelmanBs s!ndrome
HordonBs s!ndrome presents as almost the mirror image of the metabolic abnormalities seen in
BartterBs s!ndrome* and is caused primaril! b! renal sodium retention and +olume epansion.
AddisonBs disease is* of course* associated with h!potension and h!per,alaemia. .onnBs
s!ndrome is primar! h!peraldosteronism and is associated with h!po,alaemia. In general* the
emergenc! treatment of h!per,alaemia is well documented* with acute administration of
calcium to reduce the ris, of arrh!thmias* insulin to dri+e potassium into cells and the use of
ion-echange resins to increase the ecretion of potassium +ia the gastrointestinal tract .
144 . A *5-year-old woman with a strong family history of breast cancer &isits
you because she is keen to start on tamo<ifen for breast cancer
prophyla<is.9hich of the following statements best describes the mode of
action of tamo<ifen ?
It is a progesterone-receptor agonist
It is a progesterone-receptor antagonist
It is an oestrogen-receptor agonist
It is an oestrogen-receptor antagonist
It is a mied oestrogen-receptor antagonist and partial agonist
Your answer
The use of tamoifen after breast cancer is associated with a '0F ris, reduction in metastatic
disease* and is also associated with a reduction in the ris, of primar! breast cancer in high-ris,
women. It is a mied antagonist and partial agonist at the oestrogen receptor. Its partial
agonist action is probabl! reflected in the increased ris, of endometrial carcinoma associated
with chronic tamoifen use. Tamoifen has been considered the treatment of choice* although
positi+e e+idence for other hormone modulators is accumulating .
Additional modern selecti+e oestrogen-receptor modifiers now eist* one eample being
raloifene which is currentl! used for the treatment of osteoporosis .
145 . Jou are asked to urgently re&iew a 5=-year-old woman who presents with a
slowly enlarging hard mass in the anterior neck. 4hyroid ultrasound re&eals
infiltration and biopsy does re&eal dense infiltration of the gland. 3ree 4- is
low and her 4S5 is markedly raised consistent with hypothyroidism. 4hyroid
autoantibodies are negati&e.9hich of the following is the most likely diagnosis
gi&en this clinical picture ?
RiedelBs th!roiditis
Your answer
Th!roid carcinoma
2ashimotoBs th!roiditis
Hra+eBs disease
Toic multinodular goitre
B! the nature of its presentation* RiedelBs th!roiditis is often confused with th!roid carcinoma. It
is characterised b! mar,ed fibrous infiltration of the th!roid gland* the aetiolog! of this being as
!et unidentified. Treatment is with th!roine replacement. 2ashimotoBs th!roiditis is
autoimmune in aetiolog! and is characterised b! l!mphoc!tic infiltration and the presence of
antimicrosomal antibodies. Hra+eBs disease is associated with th!roid-stimulating autoantibodies
and h!perth!roidism. Th!roid ultrasound re+eals infiltration but no discrete mass is identified*
and biops! does re+eal dense infiltration of the gland .
146 . Jou are referred a 15-year-old girl from a family of tra&ellers who has ne&er
pre&iously attended medical care. 5er mother is concerned that she has short
stature and is still to commence her periods. 7n e<amination she is clearly
below the 50th centile for height. She appears to ha&e e<cess skin around her
neck and has poor de&elopment of secondary se<ual characteristics. 4here is
a murmur suggesti&e of aortic stenosis. 6hromosome analysis re&eals a -5 ?'
0 picture. 9hat diagnosis fits best with this clinical picture ?
>oonanBs s!ndrome
TurnerBs s!ndrome
Your answer
Testicular feminisation
.ongenital adrenal h!perplasia
.ongenital l!mphoedema
TurnerBs s!ndrome occurs in between # in '011 and # in 0111 li+e births. Rarel!* girls with
TurnerBs s!ndrome ma! still present late if the! ha+e limited contact with medical ser+ices* as in
this case. In older children* s!mptoms include slow linear growth* short stature* dela!ed
de+elopment of secondar! seual characteristics and absence of menses. There ma! also be
d!spraia* poor spatial awareness or mild intellectual impairment. Associated cardio+ascular
defects ma! include aortic stenosis* bicuspid aortic +al+e or coarctation of the aorta. 5atients
with TurnerBs s!ndrome also ha+e an increased ris, of h!poth!roidism .
>oonanBs s!ndrome presents with a similar phenot!pic picture to TurnerBs s!ndrome but without
the absent I chromosome .
147 . A 7--year-old man who is maintained on metformin for type-2 diabetes
presents to the emergency department acutely unwell. 5e is shocked drowsy
and confused. (lood testing re&eals a metabolic acidosis with an anion gap of
2- mmol+l. 1etones are not significantly ele&ated and random blood glucose
was =.7 mmol+l .
9hat is the mainstay of treatment for this condition ?
Intra+enous insulin therap!
8.2% sodium bicarbonate
1.26% sodium bicarbonate i+
4.1% sodium bicarbonate i+ and reh!dration
Reh!dration
Your answer
The patient has lactic acidosis and re7uires close monitoring and should be admitted to an 238
or IT8 ward. The lactate is the cause of the raised anion gap. The mainsta! of treatment is
reh!dration. 5re+ious commentaries recommended sodium bicarbonate* although no
randomised clinical trials eist to support its use and it ma! be deleterious. 3espite modern fluid
management and aggressi+e treatment of these patients* mortalit! still approaches 01F. It is
important to monitor creatinine in patients on metformin as a raised serum creatinine le+el
significantl! increases the ris, of lactic acidosis. Man! ph!sicians reduce or discontinue
metformin when the serum creatinine le+el is abo+e #$1 Rmol&l in women or #01 mmol&lin men*
although the serum creatinine le+el should alwa!s be considered in the contet of o+erall muscle
mass .
148 . A 5--year-old type-2 diabetic man presents for re&iew.9hich of the following
laboratory test results would be most significantly associated with an
increased incidence of cardio&ascular disease in his case ?
Raised proinsulin le+els
Your answer
3ecreased proinsulin le+els
>ormal or decreased L3L cholesterol
Increased 23L cholesterol le+els
Increased trigl!ceride le+el to #1F abo+e the normal
range
Raised le+els of proinsulin were shown in the .aerphill! .ohort ;tud! to be independentl!
associated with an increased incidence of cardio+ascular disease .
Inter+ention trials with proinsulin were also discontinued after a possible .@ e+ent signal was
seen. The reason behind this association has not !et howe+er been clearl! elucidated .
Raised L3L le+els are an independent cardio+ascular ris, factor and treatment with statins has
been pro+en to reduce cardio+ascular ris, .
Increased 23L cholesterol le+els are associated with re+erse cholesterol transport and increased
processing of cholesterol b! the li+er* and are therefore protecti+e .
Increased trigl!ceride le+els are associated with increased cardio+ascular ris,* but their
importance as a cardio+ascular ris, factor has !et to be pro+en categoricall! +ia an inter+ention
stud! .
Insulin resistance* a root cause of t!pe-' diabetes* is also independentl! associated with an
increased cardio+ascular ris,. 9hether it is a ris, mar,er or a ris, factor in its own right is open
to debate. Results of glitazone inter+ention to reduce insulin resistance in T!pe ' diabetes ha+e
been mied at best* with the 5roacti+e stud! demonstrating perhaps a small benefit in reducing
e+ents* and the R/.:R3 stud! demonstrating at best a neutral effect .
149 . Jou are re&iewing a 52-year-old woman who has a history of type-1
diabetes autoimmune thyroid disease and coeliac disease. 5er daughter has
been researching autoimmune disease on the %nternet and has been learning
about the association between certain 52A types and disease.9hich of the
following 52A subtypes is most strongly associated with autoimmune thyroid
disease or type-1 diabetes ?
2LA-B(=
2LA-B'=
2LA-3R$
Your answer
2LA-A'E
2LA-3R=
2LA-3R$ is associated with diabetes mellitus* autoimmune hepatitis* dermatitis herpetiformis*
Hra+eBs disease* membranous glomerulonephritis* m!asthenia gra+is* AddisonBs disease*
;jWgrenBs s!ndrome and s!stemic lupus er!thematosus (;L/). 2LA-B(= is associated with
congenital adrenal h!perplasia* -B'= with seronegati+e arthropathies* -A'E with schizophrenia
and -3R= with minimal-change disease (nephrotic s!ndrome .(
150 . An obese -=-year-old woman &isits the endocrine clinic. Screening for
endocrine disease including diabetes mellitus hypothyroidism and 6ushingFs
disease is negati&e. She asks for dietary ad&ice particularly about her fat
intake.9hich of the following is the best ad&ice to gi&e her concerning her fat
intake ?
Total fat inta,e should be restricted to less than (1F of total dietar!
energ!
;aturated fats should pro+ide no more than 0F of dietar! energ!
Monounsaturated fats should pro+ide around ?F of dietar! energ!
5ol!unsaturated fats should pro+ide around $F of dietar! energ!
Total fat inta,e should be restricted to less than $1F of total dietar!
energ!
Your answer
The current recommendations in the 86 for fat inta,e are that total fat inta,e should be
restricted to less than $1F of dietar! energ!C that monounsaturated fats should pro+ide around
#'F* pol!unsaturated fats around ?F and that saturated fats should pro+ide no more than
#1F of dietar! energ!. Increased dietar! fat inta,e has an epidemiological association with
cardio+ascular disease* cancers* obesit! and t!pe-' diabetes. /ssential fatt! acid deficienc! ma!
occasionall! occur in patients on long-term parenteral nutrition who are gi+en a mi of protein
and glucose. Alopecia* thromboc!topenia* anaemia and dermatitis ma! occur .
151 . Jou are asked by the respiratory physicians to see a 7--year-old man who
has been admitted from A:; with an abnormal chest ?-ray re&ealing a right
hilar mass. 4here is a history of -0 years of cigarette smoking. 5e is noted to
ha&e a markedly raised corrected calcium le&el of *.25 mmol+l. Apart from a
mildly raised urea le&el due to dehydration his other renal function testing is
normal. 5e also has a normochromic normocytic anaemia with a haemoglobin
of 10.0 mg+dl .
9hat is likely to be the underlying cause of his hypercalcaemia ?
5rimar! h!perparath!roidism
;econdar! h!perparath!roidism
Tertiar! h!perparath!roidism
5seudo-h!perparath!roidism
Raised parath!roid hormone-related protein
Your answer
5arath!roid hormone-related protein (5T2-rp) is a #(( amino acid pol!peptide* the initial
se7uence of which shows some homolog! with the biologicall! acti+e part of 5T2. This is being
produced b! his bronchial carcinoma. .ommon primar! tumours which ma! be associated with
h!percalcaemia include bronchial* breast* renal* prostate carcinomas* m!eloma and l!mphoma.
Treatment in+ol+es reh!dration and intra+enous bisphosphonate therap!. 5atients are often
trialled on prednisolone* but this is rarel! successful .
152 . A 1G-year-old student nurse was admitted after her third collapse in recent
months. She was noted to ha&e a blood sugar of 0.G mmol+l on finger -prick
testing and responded well to intra&enous glucose therapy. Eenous blood
taken at the same time as obtaining &enous access showed a markedly raised
insulin le&el but her 6-peptide le&els were normal.9hat diagnosis fits best
with this clinical picture ?
Insulinoma
Hlucagonoma
:ccult administration of sulphon!lureas
;elf-administration of a short-acting
insulin
Your answer
T!pe-# diabetes
This woman has h!perinsulinaemia and h!pogl!caemia* but her .-peptide le+els are normal.
This pattern is strongl! suggesti+e of the fact that she is self-administering insulin. :n further
7uestioning in this case it was determined that her father has t!pe-# diabetes and she had been
using his Actrapid to induce h!pogl!caemia during times of stress at wor,. If both the insulin
and .-peptide le+els were raised* then it would be worth screening her urine for a sulphon!lurea
assa!. :nl! after this had been ecluded would it be worth pursuing in+estigation for
insulinoma .
153 . A *5-year-old woman &isits you in the paediatric diabetes clinic with her 2-
year-old son who has recently de&eloped type-1 diabetes. 5e has an identical
twin brother and she is concerned about his risk of de&eloping diabetes.9hat
ad&ice would you gi&e regarding his future risk ?
2e has a #11F future ris, of de+eloping t!pe-# diabetes
A trial of low-dose insulin in the unaffected child will reduce his future ris,
of diabetes
HA3 antibodies in the unaffected child are not predicti+e of the ris, of
diabetes
2e has a $1D01F future ris, of de+eloping t!pe-# diabetes
Your
answer
IA-' antibodies in the unaffected child are not predicti+e of the ris, of
diabetes
Twin studies ha+e suggested that the future ris, of diabetes in the unaffected child is $1D01F.
The presence of glutamic acid decarbo!lase (HA3)* islet-cell or IA-' antibodies in the
unaffected child increases the li,elihood that the child will go on to de+elop t!pe-# diabetes.
.hildren of t!pe-# diabetic patients ha+e a slightl! increased ris, of de+eloping the disease ($D
?F ris, where the father is diabetic* 'D$F where the mother is diabetic). Twin studies in t!pe-'
diabetes ha+e suggested that if one identical twin de+elops the disease* then the ris, in the
unaffected twin rises to 01F or greater. Maturit!-onset diabetes of the !oung (M:3Y) is a rare
+ariant of t!pe-' diabetes with a +er! strong familial pattern of inheritance .
154 . Jou are re&iewing a -5-year-old man who has type-2 diabetes. 5e works a
&arying shift pattern as a ta<i dri&er and has not tolerated metformin therapy
due to gastrointestinal side-effects. Jou decide that the postprandial glucose
regulator nateglinide is the most appropriate therapy choice. 9hich of the
following best describes the mode of action of nateglinide ?
It acts b! reducing hepatic glucose output
It acts b! reducing peripheral insulin resistance
It acts b! closure of P -cell calcium channels
It acts b! closure of the P-cell 6DAT5 channel
Your answer
It pre+ents the gastrointestinal absorption of sugars
It acts b! closure of the P-cell 6-AT5 channel promoting release of insulin. It is short acting and
ta,en to coincide with meals. >ateglinide is more epensi+e than sulphon!lureas but offers
ad+antages for shift wor,ers and patients who ma! fast for a period of time as doses can be
s,ipped when meals are missed. In these patient groups there ma! be less incidence of
h!pergl!caemia. It appears that the shorter action of duration appears to result in less weight
gain compared to traditional sulphon!lureas* and in theor! these agents should result in Xless
strainY on the P-cell .
155 . A 7--year-old man is admitted to A:; in an acute confusional state. 5is
serum sodium concentration is 105 mmol+l and because he has hea&ily
nicotine-stained fingers S%AD is suspected .
9hich of the following biochemical findings best supports this diagnosis ?
Random plasma cortisol concentration of 0(E nmol&l
Random urine osmolalit! of $E1 mmol&,g
.orrect answer
Random urine sodium concentration of =' mmol&l
;erum osmolalit! of '$1 mmol&,g
Your answer
;erum urea concentration of '.# mmol&l
A diagnosis of ;IA3 (s!ndrome of inappropriate antidiuresis* also ,nown as the s!ndrome of
inappropriate secretion of antidiuretic hormone) is often made without all the criteria being
satisfied. .linicall!* there should be no e+idence of fluid o+erload (oedema) or deh!drationA
normal adrenal and renal function must be demonstrableA serum osmolalit! must be decreased
and* criticall!* the urine osmolalit! should be greater than that of the serum (establishing that
there is impaired water ecretion). The cortisol concentration in this case* gi+en that the patient
is sic,* effecti+el! ecludes adrenal failure and satisfies one criterion. 8rea concentration is lowC
this is li,el! to be because of water o+erload .
5atients with ;IA3 are usuall! in sodium balance and sodium ecretion is appropriate for inta,e.
@er! low urine sodium would suggest that h!ponatraemia was due to sodium depletion .
156 . 3ollowing a head inCury a 2--year-old patient de&elops polyuria and
polydipsia and is suspected of ha&ing cranial diabetes insipidus. 5e undergoes
a water depri&ation test.9hich one of the following responses would most
indicate a positi&e #abnormal$ result ?
A rise in plasma osmolalit! to $1' mmol&,g during water depri+ation
4ailure to concentrate the urine during water depri+ation* but achie+ement of
urine osmolalit! of ='1 mmol&,g following the administration of desmopressin
.orrect
answer
4ailure to concentrate the urine either with water depri+ation or following the
administration of desmopressin
Your
answer
4ailure to concentrate the urine with a plasma osmolalit! of 'E1 mmol&,g at
the end of the period of water depri+ation
9eight loss of 0F during the in+estigation
;ignificant weight loss* an increase in plasma osmolalit! and a failure of urinar! concentration
during water depri+ation are all diagnostic of diabetes insipidus (3I) but do not differentiate
between this being cranial or nephrogenic in origin. In nephrogenic 3I* howe+er* there is
t!picall! no response to desmopressin* whereas this causes the urine to become concentrated in
cranial 3I. A low plasma osmolalit! at the end of the in+estigation* with persistentl! dilute urine*
suggests prior water loading or continued water inta,e. >ote that cranial 3I can be transient
following head injur! .
157 . A -G-year-old woman presents complaining of fatigue and weight loss. She
was successfully treated medically for a prolactinoma * years ago and it is
suspected that she might now ha&e );/-1.9hich of the following underlying
conditions is she most likely to ha&e ?
Medullar carcinoma of th!roid
5arath!roid h!perplasia
Your answer
5ancreatic islet-cell tumour
5ituitar! adenoma
5haeochromoc!toma
5arath!roid h!perplasia is seen in the +ast majorit! of patients with multiple endocrine
neoplasia t!pe # (M/>-#)* and the h!percalcaemia that the! cause is the most common
presenting feature of the condition. 2!percalcaemia ma! be disco+ered incidentall! or because it
is s!mptomatic* but the s!mptoms are often non-specific. Islet-cell tumours (most fre7uentl!
gastrinomas) are present in about (1F of cases. 5ituitar! tumours occur in about $1F of
patientsC o+er a half are prolactinomas. Medullar!-cell carcinomas of the th!roid and
phaeochromoc!tomas occur in M/>-' but not in M/>-# .
158 . A 51-year-old man presents with weight loss and is found on e<amination to
ha&e hepatomegaly. 5e has facial telangiectasia. !rinary 5-5%AA le&els are
found to be ele&ated.Hi&en the diagnosis of carcinoid syndrome which
additional clinical feature is the most likely to be present ?
Abdominal pain
3iarrhoea
.orrect answer
2!pertension
;,in rash
Your answer
9heezing
3iarrhoea and flushing* occurring separatel! or together* are the most fre7uent presenting
features of carcinoid s!ndrome (approimatel! half of all patients). Abdominal pain occurs in
about #1F. 9heezing is a recognised feature of the condition but is uncommon* and a s,in rash
(pellagra) due to niacin deficienc! (niacin s!nthesis ma! be decreased in carcinoid s!ndrome) is
rare. 2!pertension is not a feature of the condition .
159 . A 7*-year-old woman presents with weight loss and is found to ha&e a serum
calcium concentration of *.22 mmol+l. A skeletal sur&ey is normal. /on-
metastatic hypercalcaemia is suspected. ;ndoscopy re&eals a gastric
carcinoma .
Secretion of which of the following substances by the tumour is most likely to be
responsible ?
.alcitonin
.alcitriol (#*'0-dih!dro!cholecalciferol (
:steoclast-acti+ating c!to,ines
Your answer
5arath!roid hormone
5arath!roid hormone-related peptide
.orrect answer
In patients with solid tumours* non-metastatic h!percalcaemia is most fre7uentl! the result of
secretion of parath!roid hormone-related peptide (5T2r5) b! the tumour* and is reported in
association with gastric cancer. >on-parath!roid secretion of 5T2 itself is +er! rare. 8nregulated
production of calcitriol has been described in patients with l!mphoma but is rare. ;ecretion of
calcitonin* either ectopicall! or b! medullar!-cell carcinoma of the th!roid* is clinicall! silentC it
does not perturb calcium homeostasis. ;ecretion of osteoclast-acti+ating c!to,ines is an
important cause of h!percalcaemia in patients with m!eloma* but this is a relati+el! uncommon
malignanc! .
160 . A 55-year-old man is found incidentally to ha&e hypercalcaemia during a
routine health screen .
9hich one of the following biochemical findings would be most suggesti&e of
this being caused by primary hyperparathyroidism rather than any other cause
of hypercalcaemia ?
/le+ated '(-h urinar! calcium ecretion
Your answer
/le+ated serum al,aline phosphatase acti+it!
Low serum concentration of calcitriol (#*'0-dih!dro!cholecalciferol (
>ormal serum phosphate concentration
;erum 5T2 concentration within the normal range
.orrect answer
Although parath!roid hormone (5T2) concentrations are often increased in patients with
h!perparath!roidism* the! are not alwa!s so. 5T2 secretion should be suppressed b!
h!percalcaemia from an! other cause* so that a 5T2 +alue in the normal range is consistent
with the diagnosis. 8rinar! calcium ecretion is increased in man! causes of h!percalcaemia
(ecept familial h!pocalciuric h!percalcaemia)* including h!perparath!roidism. ;erum al,aline
phosphatase acti+it! can also be ele+ated with h!percalcaemia* regardless of the cause (with
the eception of m!eloma). ;erum phosphate concentrations tend to be reduced in
h!perparath!roidism (5T2 is phosphaturic) and calcitriol concentrations to be increased (5T2
stimulates the formation of this hormone .(
161 . A 57-year-old woman is brought to A:; after sustaining a head inCury. A skull
?-ray shows there is no fracture but that there is erosion of the anterior
clinoid processes of the pituitary fossa suggesting a pituitary tumour.9hich
of the following hormones is most likely to be being secreted in e<cess ?
Adrenocorticotrophin (A.T2 (
4ollicle-stimulating hormone (4;2 (
Hrowth hormone
5rolactin
Your answer
Th!roid-stimulating hormone (T;2 (
5rolactinomas are the most fre7uentl! occurring functional pituitar! tumoursA howe+er* prolactin
secretion ma! be increased in patients with other pituitar! tumours because of the interruption
of the inhibitor! effect of dopamine secreted b! the h!pothalamus. A.T2-secreting and growth
hormone-secreting tumours both occur less fre7uentl! and T;2- and 4;2-secreting tumours are
uncommon. >on-functional tumours can occur at an! age but are more fre7uent in the elderl! .
162 . A -2-year-old patient complains of se&ere fatigue following surgery for a
pituitary tumour and is put on growth hormone replacement.9hich of the
following is a well-recognised effect of this treatment ?
3ecrease in serum lipoprotein(a) concentration
Increase in fasting serum trigl!ceride concentration
Increase in fat mass
Increase in lean bod! mass
Your answer
Increase in serum total cholesterol concentration
Hrowth hormone replacement in adults with deficienc! of the hormone has numerous effects*
including an increase in +italit! and o+erall 7ualit! of life. Lean bod! mass tends to increase but
bod! fat decreases* often to a greater etent. ;erum total- and L3L-cholesterol concentrations
and trigl!cerides tend to decrease but lipoprotein (a) concentration ma! increase .
163 . A *7-year-old man with a diagnosis of hypogonadotrophic hypogonadism is
being followed in the endocrine clinic. 5e does not desire fertility at
present.9hich would be the most appropriate treatment at this stage ?
:ral testosterone replacement
5ulsatile subcutaneous administration of gonadotrophin-releasing hormone
(HnR2 (
Regular injections of human chorionic gonadotrophin
Regular injections of human menopausal gonadotrophin (2MH (
Regular testosterone injections
Your
answer
If fertilit! is not re7uired* there is no need to stimulate spermatogenesis with HnR2 or
gonadotrophinsC onl! testosterone replacement is re7uired. Testosterone undecanoate is
a+ailable for oral use* but fre7uent dosage is re7uiredA and* because of poor absorption* the
plasma concentrations that can be achie+ed are often subnormal. Testosterone injections*
implants or patches are more reliable .
164 . A ---year-old man is surprised to find that he cannot easily get his feet into a
pair of shoes that he last wore 5 years ago. 5e goes to buy a new pair and is
told that his si"e has increased. 5e trawls the %nternet for an e<planation and
deciding that he may ha&e acromegaly consults his H8. 4he H8 has not seen
him for se&eral years and thinks his appearance has changed so refers him to
the endocrine clinic .
9hich of the following would be the most useful first-line test for in&estigating him ?
Hlucose tolerance test with measurement of growth hormone
Insulin h!pogl!caemia test (insulin tolerance test* insulin stress test (
Measurement of serum growth hormone during sleep
Measurement of serum growth hormone following eercise
Measurement of serum insulin-li,e growth factor-# (IH4-#)
concentration
Your answer
In health! indi+iduals* growth hormone secretion is suppressed following the administration of
glucose. 4ailure of suppression is diagnostic of ecessi+e growth hormone secretion. As an initial
screening test howe+er* it ma! be more practical to measure serum IH4-# and to follow this
with an :HTT if the results are abnormal. An ad+antage of IH4-# is that le+els are relati+el!
stable o+er time. The insulin h!pogl!caemia test* measurement of growth hormone during sleep
and measurement following eercise* ma! be used in the in+estigation of suspected growth
hormone insufficienc! .
165 . )I imaging indicates a microadenoma in a *A-year-old man presenting with
features of acromegaly and pro&en to ha&e e<cessi&e growth hormone
secretion.9hich of the following would usually be the treatment of choice ?
Medical treatment with a growth hormone-receptor antagonist
Your answer
Medical treatment with a somatostatin analogue
Radiotherap!
Transfrontal surger!
Trans-sphenoidal surger!
.orrect answer
;urger! is generall! regarded as the first-line treatment for patients with acromegal!. The
trans-sphenoidal route is preferred ecept with large tumours* which ma! re7uire a transfrontal
approach. Radiotherap! is sometimes used if surger! does not reduce growth hormone
concentrations to acceptable le+els. Medical treatment is used principall! as an adjunct to
surger!. 3opamine agonists (eg cabergoline) ha+e been superseded b! somatostatin agonists
(eg octreotide) and growth hormone-receptor antagonists (eg peg+isomant) are now becoming
a+ailable .
166 . A -G-year-old woman is in&estigated to determine the cause of 6ushingFs
syndrome following the demonstration of hypercortisolaemia by a high 2--
hour urinary cortisol e<cretion. 5er midnight plasma cortisol concentration is
ele&ated. 5er 0G00-h plasma cortisol concentration falls by A0P following a
high-dose de<amethasone suppression test, plasma cortisol concentration
increases by 25P following intra&enous corticotrophin-releasing hormone
#6I5$.9hich is the most likely diagnosis ?
Adrenal adenoma
Your answer
Adrenal carcinoma
Bilateral adrenal h!perplasia
.ushingBs disease (pituitar!-dependent .ushingBs s!ndrome (
.orrect answer
/ctopic secretion of A.T2
Incomplete suppression of cortisol following high-dose deamethasone (' mg ?-hourl! for (E h)
is t!pical of .ushingBs diseaseC there is t!picall! no suppression in patients with other causes of
.ushingBs s!ndrome. An increase in cortisol following the administration of .R2 is also
suggesti+e of .ushingBs disease* but with other causes there is usuall! no response.
:ccasionall!* carcinoid tumours secrete A.T2 and cause similar responses in these tests to
those seen in .ushingBs disease. This* howe+er* is a rare cause of .ushingBs s!ndrome .
167 . A 50-year-old woman with hypertension that has been difficult to control
with drugs is found to ha&e hypercortisolaemia.9hich one of the following
clinical findings would most suggest that ectopic secretion of A645 is the
cause of the condition ?
Hl!cosuria
9orsening h!pertension
2!po,alaemia
Muscle wasting
Your answer
9eight loss
.orrect answer
Hl!cosuria* h!pertension* h!po,alaemia and muscle wasting can occur with .ushingBs s!ndrome
from an! cause. The most common manifestation is centripetal fat deposition* often with weight
gain* but weight loss suggests there is an underl!ing malignanc! .
168 . An 1=-year-old woman complains of a 2-month history of &ague ill health and
nausea. She has had se&eral episodes of di""iness and is found to ha&e
postural hypotension.9hich of the following in&estigations is reBuired to best
demonstrate that these features are the result of adrenal failure ?
Measurement of earl! morning and midnight plasma cortisol
concentrations
Measurement of earl! morning plasma A.T2 (corticotrophin)
concentration
Measurement of '(-h urinar! cortisol ecretion
Your answer
:+ernight deamethasone suppression test
;hort A.T2 (;!nacthen) stimulation test
.orrect
answer
A diagnosis of adrenal failure is demonstrated b! a failure of the plasma cortisol concentration
to increase in response to A.T2. 3emonstration of a high plasma A.T2 concentration indicates
primar! rather than secondar! (to pituitar! insufficienc!) adrenal failure. Random
measurements of cortisol are onl! of +alue in sic, patients* in whom cortisol concentrations
should normall! be high. Measurement of the diurnal +ariation in secretion (which is lost)* the
cortisol response to deamethasone (decreased) or urinar! cortisol ecretion (increased) are
used in the diagnosis of .ushingBs s!ndrome but ha+e no place in the diagnosis of adrenal
failure .
169 . A 17-year-old young woman is admitted to A:; ha&ing collapsed at a ra&e.
She is in a shocked state and unable to gi&e a coherent history but is found to
ha&e a card in her purse that indicates that she is on steroids for adrenal
failure. A clinical diagnosis of an addisonian crisis is made and a blood sample
is taken for cortisol measurement. 3inger prick glucose testing re&eals a () of
*.- mmol+l.9hich of the following should be gi&en the most priority in her
management ?
Intra+enous glucose infusion
5arenteral administration of h!drocortisone
Your answer
Replacement of mineralocorticoid
Resuscitation with intra+enous ph!siological saline and h!drocortisone
.orrect answer
Treatment of an! precipitating factor
All these measures (with the eception of mineralocorticoid replacement) are re7uired in an
addisonian crisis* glucocorticoid and fluid replacements the priorit!* often gi+en simultaneousl!
in clinical practice* but the immediate priorit! is fluid resuscitation. Mineralocorticoid
replacement ma! be re7uired long term* but is unnecessar! in the acute setting because the
large doses of h!drocortisone used suppl! mineralocorticoid acti+it! .
170 . A 2--year-old man is found to ha&e hypertension during an e<amination for
life assurance purposes. 7&er the ne<t few months this is demonstrated to
fluctuate considerably in se&erity and pro&es difficult to control.9hich of the
following additional features would most suggest that a phaeochromocytoma
is causing his hypertension ?
3iarrhoea
4lushing
2eadache
Your answer
Muscle wea,ness
Tremor
Approimatel! E1F of patients with phaeochromoc!toma complain of headaches* which are
often paro!smal. Tremor is much less common. 5allor* not flushing* is seen in those with a
phaeochromoc!toma and patients ma! complain of constipation. 4lushing and diarrhoea are
features of the carcinoid s!ndrome. Approimatel! one-7uarter of patients eperience muscle
wea,ness* which can also be present in primar! aldosteronism* another endocrine cause of
h!pertension .
171 . A *G-year-old man with untreated hypertension has a plasma potassium
concentration of 2.= mmol+l. 9hen measured at 0G00 h with the patient
supine his plasma aldosterone concentration is ele&ated and renin acti&ity is
low. 9hen measured at 1200 h with the patient upright the plasma
aldosterone concentration increases by 20P. 8lasma cortisol concentration
was also measured at both times and found to be decreased by 50P .
9hat is the most likely diagnosis ?
Adrenal adenoma secreting aldosterone
Bilateral adrenal h!perplasia
.orrect answer
LiddleBs s!ndrome
;econdar! aldosteronism
;teroid ##P-h!dro!lase deficienc!
Your answer
Adrenal adenomas that secrete aldosterone (now thought to be the less li,el! cause of
h!poreninaemic h!peraldosteronism) are usuall! sensiti+e to A.T2* and aldosterone secretion
falls during the da! as A.T2 secretion (and that of cortisol) declines. 9ith bilateral adrenal
h!perplasia* howe+er* the aldosterone concentration is usuall! higher when the patient is erect
than when supine. 2!pertension is also a feature of LiddleBs s!ndrome and steroid ##P
-h!dro!lase deficienc!* but aldosterone concentrations are low. In secondar! aldosteronism*
aldosterone secretion is increased secondar! to an increase in renin secretion* and plasma renin
acti+it! is normal or increased .
172 . A 52-year-old woman with primary hypothyroidism is being treated with
thyro<ine replacement the dose of which is being titrated against the results
of biochemical thyroid function tests. 4wo weeks after the last increase in
dose the results of thyroid function tests are' free thyro<ine 2= pmol+l
#normal GK2A$, 4S5 1- m!+l #normal 0.2K5.0). 9hich #if any$ would be the
most appropriate ne<t step in the management of this patient ?
A decrease in the dose of th!roine
A further increase in the dose of th!roine
Your answer
>o change in dose at this time
.orrect answer
Nuestion patientBs compliance with medication
Replace th!roine with triiodoth!ronine
It is too soon after the last change in dose for a new stead! state to ha+e been achie+edC it is
usual to wait for a month before re+iewing the response. 9hen th!roine replacement is
started* the T;2 concentration often falls to normal more slowl! than free th!roine increases.
4ree th!roine concentrations in clinicall! euth!roid patients on th!roine replacement are often
high normal or e+en slightl! ele+ated* reflecting the lac, of production of triiodoth!ronine (T$)
b! the th!roidC peripheral metabolism of th!roine is the onl! source of T$. Irregular medication
could eplain these results but should not be considered until sufficient time has elapsed for the
data reliabl! to reflect the effects of replacement .
Triiodoth!ronine is usuall! onl! used in +er! se+ere h!poth!roidism* when a rapid response to
treatment is re7uired .
173 . A 20-year-old woman presents with an<iety and weight loss with increased
appetite. 4hyroto<icosis is suspected and &arious in&estigations are
performed.9hich of the following findings would most suggest that she has
Hra&esF disease ?
2igh /;R
2igh serum triiodoth!ronine (T$) concentration but normal th!roine (T()
concentration
2igh titre of th!roid peroidase autoantibodies
Your
answer
Low th!roid upta,e of technecium-MMm
>ormal serum T;2 concentration
Antibodies to th!roid peroidase and th!roglobulin are found in the serum of the majorit! of
patients with Hra+esB disease* in which radioisotope upta,e is t!picall! increased. A low upta,e
occurs in subacute (de Nuer+ainBs) th!roiditis (in which the er!throc!te sedimentation rate
(/;R) is t!picall! ele+ated). /le+ated T$ with a normal T( (JT$ toicosisB) can occur earl! in the
course of th!rotoicosis from an! cause. The high concentrations of th!roid hormones suppress
T;2 secretion to +er! low le+els in th!rotoicosis due to th!roid diseaseC detectable (not alwa!s
ele+ated) T;2 in th!rotoicosis suggests that this is caused b! a pituitar! adenoma secreting
T;2 .
174 . A *5-year-old woman had a febrile infection associated with a painful
swelling in her neck a week ago. 5er thyroid function tests show e&idence of
thyroto<icosis and her ;SI is raised.9hat is the most likely diagnosis ?
;poradic goitre
2ashimoto th!roiditis
Your answer
4ibromatosis
3e Nuer+ainBs th!roiditis
.orrect answer
4ollicular th!roid carcinoma
;ubacute (or de Nuer+ainBs) th!roiditis is due to th!roid infection b! an! of a number of +iruses*
especiall! param!o+iruses (mumps)* cosac,ie+iruses* influenza +iruses* adeno+iruses and
echo+iruses .
The most prominent s!mptom is pain in the th!roid* often radiating to the ears. A small* tender
goitre can be palpated that is usuall! diffuse* but there can be as!mmetrical in+ol+ement.
;!stemic upset with fe+er is +ariable but sometimes profound* and s!mptoms of a prodromal
+iral infection se+eral wee,s earlier ma! be recalled. There is a granulomatous th!roid
inflammation with follicular destruction* and the release of th!roid hormones often results in a
transient th!rotoicosis* lasting for #D( wee,s. .ontinuing th!roid destruction then leads to a
phase of h!poth!roidism once stored hormone is depleted. This lasts (D#' wee,s before
euth!roidism is restored* but relapses occur in #1D'1F of cases. ;ometimes onl! one phase of
th!roid disturbance is seen .
.onfirmation of the clinical diagnosis is made b! finding an ele+ated er!throc!te sedimentation
rate (/;R) and low or absent radioiodine upta,e b! the th!roid. Th!roid function re7uires
continuous monitoring as the disease e+ol+es. Mild cases ma! resol+e spontaneousl! with
aspirin as s!mptomatic treatment* but most patients benefit from prednisolone (1D?1 mg dail!*
which rapidl! alle+iates the pain. The dose is tapered o+er ?DE wee,s* depending largel! on
s!mptoms. 5ropranolol ma! be useful for th!rotoic s!mptoms* and temporar! th!roine
replacement is sometimes needed during the h!poth!roid phase .
175 . A A0-year-old woman presented complaining of a A.* kg #1 stone$ weight
loss polyuria and depressi&e mood for = weeks. 4he following laboratory
results are obtained' calcium *.- mmol+l phosphate 1.1 mmol+l parathyroid
hormone 5 ng+l#reference L A0 ng+l$.9hat is the most likely cause for her
complaints ?
5rimar! h!perparath!roidism
Your answer
;econdar! h!perparath!roidism
2!percalcaemia due to cancer
.orrect answer
@itamin 3 deficienc!
2!perth!roidism
The causes of h!percalcaemia can be classified according to whether serum parath!roid
hormone concentrations are ele+ated (i.e. primar! h!perparath!roidism) or low (i.e. not due to
a parath!roid tumour). 5rimar! h!perparath!roidism and malignanc! are the most common
causes and account for more than M1F of patients with h!percalcaemia .
A detailed clinical histor! and eamination will usuall! help to differentiate between these two
diagnoses. In primar! h!perparath!roidism* the h!percalcaemia is often less than $ mmol&l*
as!mptomatic and ma! ha+e been present for months or !ears. 2owe+er* in malignanc!* the
patients are usuall! acutel! ill* often with neurological s!mptoms* the h!percalcaemia is more
than $ mmol&l and the cancer (eg lung* breast or m!eloma) is often readil! apparent .
176 . A 17-year-old young woman has been referred by her gynaecologist. She has
been complaining of amenorrhoea for 5 months although no gynaecological
abnormality has been found. She feels well and is &ery acti&e but her weight
has decreased from A1 kg to -* kg in the last A months. 5er height is 1A= cm.
7n e<amination her (8 is G0+A0 mm5g heart rate A- bpm.9hat is the most
likely diagnosis ?
.onnBs s!ndrome
.rohnBs disease
Anoreia ner+osa
Your answer
2!perth!roidism
3iabetes mellitus
5atients with anoreia ner+osa acti+el! maintain an undul! low bod! weight. 4or diagnostic
purposes* an Jundul! low bod! weightB ma! be defined as a weight at least #0F below that
epected for the personBs age* height and se* or as a bod! mass inde below #=.0.
Amenorrhoea (in postmenarchal women who are not ta,ing an oral contracepti+e) is almost
alwa!s present in these patients .
177 . A patient has been referred by her H8 because she has been complaining of
freBuent episodes of sweating and palpitations associated with a low blood
glucose le&el. 4he family history re&eals a brother with type-1 diabetes. A
blood test shows the following results' glucose 1.1 mmol+l insulinN 500
pmol+l #reference 15K100 pmol+l$ and 6-peptide of L 0.2 nmol+l #reference
0.2K1.- nmol+l$.9hat is the most likely diagnosis ?
T!pe-# diabetes mellitus
Insulinoma
5ancreatic carcinoma
4actitious insulin-induced h!pogl!caemia
Your answer
Maturit!-onset diabetes of the !oung (M:3Y (
4actitious insulin-induced h!pogl!caemia is as common in pre+iousl! health! subjects as in
insulin-dependent diabetics and is due to the deliberate* but concealed* injection of insulin. The
histor! suggests insulinoma* but this is eliminated b! the laborator! results that re+eal high
plasma insulin and low .-peptide (and proinsulin) concentrations during h!pogl!caemia. In
longstanding factitious h!pogl!caemia* and in insulin-treated diabetics* insulin antibodies ma!
be present in the plasma. Although once considered a strong pointer to factitious
h!pogl!caemia* the presence of insulin antibodies should nowada!s suggest autoimmune insulin
s!ndrome .
178 . A 55-year-old obese patient with hypercholesterolaemia #2D2 5.2 mmol+l
5D2 1 mmol+l$ well-controlled type-2 diabetes and hypertension has been on
a low cholesterol diet for the last A months. 5is latest 2D2 le&el is -.= mmol+l
and triglycerides are within the normal range.%n terms of primary pre&ention
what is the ne<t therapeutic step in his management ?
Add pioglitazone
Add gemfibrozil
Add clopidogrel
Add statins
Your answer
Add ezetimibe
The latest /uropean guidelines suggest that L3L cholesterol should be < (.0 mmol&l rather than
< 0 mmol&l in secondar! pre+ention* while e+idence from primar! pre+ention suggests that
similar targets should also appl! to primar! pre+ention. ;tatins are the medication of choice. As
targets are reduced and further e+idence from the lowDdoseDhigh-dose statin comparator trials
are announced* it is li,el! that targets will fall further and that combination therap! for
h!perlipidaemia will become routine as is the case in h!pertension. The challenge will still
remain with implementation* but the benefits of satisfactor! ris,-factor control are that both
morbidit! and mortalit! from cardio+ascular disease will be decreased .
179 . 50 - year-old woman has been diagnosed with a pericardial effusion. 9hich
endocrine disease is most likely to be associated with this finding ?
2!perth!roidism
2!poth!roidism
Your answer
5haeochromoc!toma
.hronic renal insufficienc!
2!pogonadism
.linicall! silent pericardial effusion is common in untreated h!poth!roidism* it ma! be seen in
h!perth!roidism but incidence is less. The effusion itself has a high cholesterol content* which
ma! produce an unusual secondar! pericarditis with cholesterol deposits of Jgold paintB
appearance. The pericardial effusion +er! rarel! needs to be treated in its own right* and
subsides when th!roid replacement therap! is gi+en .
180 . 9hat is the most reliable in&estigation for determining the &olume of the
thyroid gland ?
Th!roid isotope upta,e technetium-MMm
8ltrasound
.orrect answer
I-ra! of the nec,
Th!roid isotope upta,e iodine-#'0
5/T scan
Your answer
8ltrasound is useful in determining th!roid size accuratel!. Th!roidal upta,e of radioisotopes
(especiall! technetium-MMm) is indicated if destructi+e th!roiditis is suspected as a cause of
goitre. :therwise* the major role for imaging is to ensure there is no tracheal compression or
intrathoracic&retrosternal component in a patient with suggesti+e s!mptoms* and a .T scan is
then the preferred in+estigation .
181 . A *0-year-old man presents with a *-month history of deteriorating physical
performance at work associated with dysarthria and clumsiness. 7n
e<amination he looks anaemic has hepatomegaly and 1ayserK3leischer rings
in the cornea.9hat would be the most important in&estigation to support the
suspected clinical diagnosis ?
.T scan of the brain
MRI scan of the brain
Li+er biops!
;erum ceruloplasmin le+el
Your answer
.R5 le+el
;erum ceruloplasmin can be measured enz!maticall! (copper oidase)* b! radial
immunodiffusion or b! re+erse passi+e haemagglutination. The acti+it! and concentration of this
gl!coprotein is reduced or absent (less than '11 mg&l) in M0F of patients with 9ilsonBs disease.
In the absence of 6a!ser-4leischer rings or neurologic abnormalities* a li+er biops! for
7uantitati+e copper determination would be essential to establish the diagnosis of 9ilsonKs
disease* but this would li,el! not be necessar! here gi+en the t!pical clinical picture .
182 . A *0-year-old man presents with a *-month history of deteriorating physical
performance at work associated with dysarthria and clumsiness. 7n
e<amination he looks anaemic has hepatomegaly and 1ayser-3leischer rings
in the cornea.9hat is the most likely diagnosis ?
2epatitis . infection
9ilsonBs disease
Your answer
Alcohol abuse
Motor neurone disease
;ubacute sclerosing panencephalitis
9ilsonBs disease ma! present in childhood* adolescence or earl! adulthood. ;!mptoms and signs
ma! be clinicall! undetectable under 0 !ears of age* and few present after the age of $0 !ears*
although diagnosis o+er 00 !ears has been reported. In M1F of patients* the disease presents
with ju+enile hepatic disease or with neurological&ps!chiatric manifestations. In large studies of
patients with 9ilsonBs disease* initial manifestations wereC hepatic ((1F)A neurological ($1F)A
ps!chiatric (#1F)A haematological (#'F)A and renal (#F). ;ome '0F of patients ha+e two or
more organs in+ol+ed (usuall! li+er and brain) at the initial assessment. 6a!serD4leischer rings
are almost pathognomonic of 9ilsonBs disease .
183 . A *0-year-old man presents with a *-month history of deteriorating physical
performance at work associated with dysarthria and clumsiness. 7n
e<amination he looks anaemic has hepatomegaly and 1ayserK3leischer rings
in the cornea.Hi&en the likely diagnosis what would be the most important
treatment option ?
Blood transfusion
d-5enicillamine
Your answer
.iclosporin
Azathioprine
Interferon-alpha
3-5enicillamine is effecti+e in remo+ing copper from patients with 9ilsonBs disease. The
optimum time for treatment is in the earl! stages* and all patients with 9ilsonBs disease should
be treated* e+en if as!mptomatic. Treatment is lifelong* unless the patient undergoes li+er
transplantation. The aim of treating a patient with 9ilsonBs disease is to reduce toic copper
le+els in the bod! tissues. This can be achie+ed b! increasing the urinar! ecretion of copper. A
negati+e copper balance should be monitored carefull! since* with an increase in urine and
faecal copper ecretion that eceeds copper inta,e* increased urinar! copper ma! reflect
increased plasma non-ceruloplasmin copper (copper in its most damaging form .(
The effect of an! therap! should be regularl! monitored b! clinical and radiological assessment*
and b! biochemical monitoring of abnormal li+er enz!mes and li+er function .
184 . A patient was referred by his H8 because of a borderline fasting glucose. Jou
arrange for a glucose tolerance test to clarify the diagnosis. 3or this test how
much glucose is dissol&ed in 250 ml of water ?
10 g
50 g
75 g
Your answer
100 g
150 g
After an o+ernight fast* the subject drin,s =0 g of anh!drous glucose dissol+ed in '01 ml water*
+enous blood is sampled at baseline and ' hours later. 4ood inta,e should be normal during the
preceding few da!sC poor nutrition can cause dela!ed h!pergl!caemia with a raised '-hour
+alue (the JlagB cur+e .(
185 . A patient with longstanding type-2 diabetes was found to ha&e a urinary
albumin e<cretion rate of -00 mg+l. 5is diabetes is well controlled and he is
normotensi&e. 9hat additional drug should he be prescribed ?
Aspirin
Atenolol
Lisinopril
Your answer
.lopidogrel
5ioglitazone
In patients with t!pe-# diabetes with a mean entr! blood pressure of #''&== mm2g* a
combined anal!sis of one /uropean and one American stud! (total n Z ''0) showed an adjusted
ris, reduction of ?$F (M0F .I* #?DE(FA p Z 1.1#=) for the de+elopment of clinical
nephropath! comparing the A./ inhibitor captopril #11 mg&da! with placebo. Three smaller
studies in normotensi+e t!pe-' patients ha+e reported a similar reduction in the rate of
de+elopment of clinical nephropath!. Thus bloc,ade of the reninDangiotensin s!stem b! an!
means appears to confer benefit .
186 . A patient presents with truncal obesity insulin resistance and
dyslipidaemia.9hat additional clinical feature might you e<pect to be present ?
Asthma
Renal failure
2!pertension
Your answer
.ancer
:phthalmoplegia
Metabolic s!ndrome I is the term gi+en to the co-occurrence of insulin resistance and glucose
intolerance (ranging from mild to o+ert t!pe-' diabetes)* with truncal obesit!* d!slipidaemia
(raised trigl!cerides and a high L3LC 23L ratio) and h!pertension .
187 . A patient has been complaining of a 2-month history of intermittent flush
associated with tachycardia and whee"ing. 4here ha&e also been episodes of
profuse watery diarrhoea.Hi&en the suspected diagnosis what would be the
most appropriate in&estigation ?
3eamethasone suppression test
8rinar! catecholamine collection
8rinar! 0-h!dro!indoleacetic acid collection
Your answer
Abdominal ultrasound
24 - hour ambulator! blood pressure recording
The biologicall! acti+e metabolite characteristicall! produced b! metastatic carcinoid tumours is
0-h!dro!tr!ptamine (0-2T* serotonin)* s!nthesised from the amino acid tr!ptophan. 0-2T
pla!s a part in the pathogenesis of some of the s!mptoms of the carcinoid s!ndrome*
particularl! the diarrhoea and bronchoconstriction. It is metabolised to 0-h!dro!indoleacetic
acid (0-2IAA)* which accounts for M0F of the urinar! ecretion of 0-2T .
188 . A 55-year-old woman has been complaining of a A-month history of weight
gain. She is otherwise well and takes no medication. 7n e<amination her ()%
is 2= (8 170+100 mm5g she has a round red face and a slight atrophy of her
arm muscles. Ienal function test and urinalysis are normal.9hat is the ne<t
step in obtaining the diagnosis ?
3eamethasone suppression test
Your answer
8rinar! catecholamine collection
8rinar! 0-h!dro!indoleacetic acid collection
Abdominal ultrasound
.T scan abdomen
Low-dose deamethasone suppression tests the diurnal circadian rh!thm .
In normal subjects* the cortisol concentration begins to rise at 1'11 h* pea,ing at 10$1D1M$1 h.
.oncentrations drop during the morning and reach a nadir at around '(11 h (cortisol < 01
nmol&l). Blood is ta,en for A.T2 and cortisol at 1M11 h and '(11 h. The finding of a normal
circadian rh!thm +irtuall! rules out the presence of acti+e .ushingKs s!ndrome .
189 . A patient with poorly diet-controlled type-2 diabetes mellitus needs to be
started on medication. 9hich concomitant condition would be a
contraindication for starting metformin ?
2!pertension
Respirator! insufficienc!
Your answer
2!perlipidaemia
2!perth!roidism
Adipositas
Lactic acidosis is best ,nown in diabetic patients as a rare* but often fatal* complication of the
biguanide metformin* which acts mainl! b! inhibiting hepatic gluconeogenesis. The ris, is much
increased in patients with respirator! insufficienc! .

You might also like